You are on page 1of 94

ReSA - THE REVIEW SCHOOL OF ACCOUNTANCY

CPA Review Batch 43  May 2022 CPALE  7 Feb 2022  6:00 PM - 9:00 PM

MANAGEMENT ADVISORY SERVICES FIRST PRE-BOARD EXAMINATION

INSTRUCTIONS: Select the correct answer for each of the questions. Mark only one
answer for each item by shading the box corresponding to the letter of your choice on
the answer sheet provided. STRICTLY NO ERASURES ALLOWED. Use pencil no. 2 only.
Set A
1. Which is NOT among the typical strategic analysis to operating profit?
C a. Cost effect of growth component
b. Cost effect of price-recovery component
c. Revenue effect of productivity component
d. Revenue effect of price-recovery component

2. Using the high-low method of cost segregation, Chinese Zodiac Company has
developed a monthly variable cost function of Y = 2.5X (where X is based on the
number of machine hours) for the factory overhead costs of its lone product based
on the following information:
Machine Hours Factory Overhead
Month 1 (Low) 200 P 1,700
Month 2 (High) ??? P 1,800

The number of machine hours for month 2:


B a. Must have been 250 hours
b. Must have increased by 20% from month 1
c. Must have increased by 40% from month 1
d. Cannot be determined from the given information

3. If a firm uses variable costing, fixed manufacturing overhead will be included


B a. Only on the balance sheet
b. Only on the income statement
c. On both the balance sheet and income statement
d. On neither the balance sheet nor income statement

4. Tiger Corporation uses a standard cost system. The following information was
provided for the period that just ended:
Actual price per kilogram P 3.00
Actual kilograms of material used 31,000
Actual hourly labor rate P 18.10
Actual hours of production 4,900 labor hrs.
Standard price per kilogram P 2.80
Standard kilograms per completed unit 6 kilograms
Standard hourly labor rate P 18.00
Standard time per completed unit 1 hr.
Actual total factory overhead P 34,900
Fixed factory overhead P 18,000
Standard fixed factory overhead rate P 1.20 per labor hour
Standard variable factory overhead rate P 3.80 per labor hour
Maximum plant capacity 15,000 hours
Plant operated during the period 10,000 hours
Units completed during the period 5,000

The direct materials cost variance is:


B a. P 3,400 unfavorable
b. P 9,000 unfavorable
c. P 9,000 favorable
d. P 3,400 favorable

5. If economic activity slows down, total costs could easily decline in which of
the following categories?
A a. Variable costs and discretionary fixed costs
b. Variable costs and committed fixed costs
c. Committed fixed costs only
d. Variable costs only

6. If sales and expenses both rise by P 100,000


D a. Asset turnover will decrease
b. Residual income will increase.
c. Return on investment will increase.
d. Return on investment will be unchanged.

Page 1 of 12 0915-2303213  resacpareview@gmail.com


MANAGEMENT ADVISORY SERVICES
ReSA Batch 43 - May 2022 CPALE Batch
07 Feb 2022  6:00 PM to 9:00 PM MAS First Pre-Board Exam

7. An actual cost system differs from a normal cost system in that an actual cost
system
B a. Assigns overhead as it occurs during the manufacturing cycle
b. Assigns overhead at the end of the manufacturing process
c. Does not use an Overhead Control account
d. Does not assign overhead at all

8. Horse Co. presents the following operating data for its manufacturing operations:
Unit selling price P 250
Unit variable cost P 100
Total fixed costs P 840,000
The company has decided to increase the wages of hourly workers which will
increase the unit variable cost by 10%. Increases in the salaries of factory
supervisors and property taxes for the factory will increase fixed costs by 4%.
If sales prices are held constant, the next break-even point for Horse will be:
A a. increased by 640 units
b. increased by 400 units
c. decreased by 640 units
d. increased by 800 units

9. The opportunity cost of making a component part in a factory with no excess


capacity is the:
D a. Fixed manufacturing cost of the component
b. Variable manufacturing cost of the component
c. Cost of the production given up in order to manufacture the component
d. Net benefit foregone from the best alternative use of the capacity
required

10. Based on past experience, Dragon Company has the following expected pattern of
collections on credit sales: 70 percent collected in the amount of sale, 15
percent in the month after the month of sale, and 14 percent in the second month
after the month of sale. The remaining 1 percent is never collected. At the end
of May, Dragon Company has the following accounts receivable balances:
From April sales P 21,000
From May sales 48,000

Dragon’s expected sales for June are P150,000. What were total sales for April?
B a. P 150,000
b. P 140,000
c. P 72,414
d. P 70,000

11. Line management typically includes


C a. IT Managers
b. HR Managers
c. Manufacturing Managers
d. Management-Accounting Managers

12. Consider the following production and cost data for two products, L1 and C2:
Product L1 Product C2
Contribution margin per unit ....... P 130 P 120
Machine set-ups needed per unit .... 10 set-ups 8 set-ups

The company can only perform 65,000 machine set-ups each period due to limited
skilled labor and there is unlimited demand for each product. What is the largest
possible total contribution margin that can be realized each period?
C a. P 845,000
b. P 910,000
c. P 975,000
d. P 1,820,000

13. The number of variables used in simple regression analysis is:


B a. One
b. Two
c. Two or more
d. Three or more

Page 2 of 12 0915-2303213  resacpareview@gmail.com


MANAGEMENT ADVISORY SERVICES
ReSA Batch 43 - May 2022 CPALE Batch
07 Feb 2022  6:00 PM to 9:00 PM MAS First Pre-Board Exam

14. Standard costs are LEAST useful for:


B a. Controlling costs
b. Income determination
c. Measuring efficiencies
d. Forming a basis for price setting

15. Rabbit Corporation uses the total cost concept of product pricing. Below is cost
information for the production and sale of 60,000 units of its sole product.
Rabbit desires a profit of 21% rate of return on invested assets of P 600,000.

Fixed factory overhead cost P 37,500


Fixed selling and administrative costs 7,500
Variable direct materials cost per unit 4.50
Variable direct labor cost per unit 1.88
Variable factory overhead cost per unit 1.13
Variable selling and administrative cost per unit 4.50

What is the unit selling price for the company's product?


B a. P 13.92
b. P 14.86
c. P 14.97
d. P 15.00

16. Managerial finance


A a. Involves tasks such as budgeting, financial forecasting, cash
management, and funds procurement.
b. Devotes the majority of its attention to the collection and
presentation of financial data.
c. Involves the design and delivery of advice and financial products.
d. Recognizes funds on an accrual basis.

Items 17 to 19 are based on the following information


The following information is available for Ox Company for its first year of
operations:

Sales in units 5,000


Production in units 8,000
Manufacturing costs:
Direct labor P 3 per unit
Direct material P 5 per unit
Variable overhead P 1 per unit
Fixed overhead P 100,000
Net income (absorption method) P 30,000
Sales price per unit P 40

17. If Ox Company had used variable costing, what amount of income before income
taxes would it have reported?
A a. (P 7,500)
b. P 30,000
c. P 67,500
d. cannot be determined from the information given

18. What was the total amount of Selling, General and Administrative expense incurred
by Ox Company?
A a. P 62,500
b. P 30,000
c. P 6,000
d. can’t be determined from the information given

19. If Ox Company were using variable costing, what would it show as the value of
ending inventory?
C a. P 120,000
b. P 64,500
c. P 27,000
d. P 24,000

Page 3 of 12 0915-2303213  resacpareview@gmail.com


MANAGEMENT ADVISORY SERVICES
ReSA Batch 43 - May 2022 CPALE Batch
07 Feb 2022  6:00 PM to 9:00 PM MAS First Pre-Board Exam

20. A company that maintains a raw material inventory, which is based on the following
month's production needs, will purchase less material than it uses in a month
where
C a. Sales exceed production
b. Production exceeds sales
c. Planned production exceeds the next month's planned production
d. Planned production is less than the next month's planned production

21. Rooster Company has two divisions – North and South. The divisions have the
following revenues and expenses:
NORTH SOUTH
Sales ........................ P 550,000 P 500,000
Variable costs ............... 275,000 200,000
Direct fixed costs ........... 180,000 150,000
Allocated corporate costs .... 170,000 135,000
Net income (loss) ............ (75,000) 15,000

The management of Rooster is considering the elimination of the North Division.


If the North Division were eliminated, the direct fixed costs associated with
this division could be avoided. However, corporate costs would still be P 305,000
in total. Given these data, the elimination of the North Division would result
in an overall company net income (loss) of:
D a. P 15,000
b. (P 60,000)
c. (P 75,000)
d. (P 155,000)

22. Management Accounting


D a. Provides information about the company as a whole
b. Reports information that has occurred in the past that is verifiable
and reliable
c. Provides information that is generally available only on a quarterly
or annual basis
d. Focuses on estimating future revenues, costs and other measures to
forecast activities and their results

23. Sheep Company developed following standards for its main product lamps:
Standard Inputs Expected for Standard Price Expected
Each Unit of Output per Unit of Output
Direct materials 20 pounds P 2 per pound
Direct labor 6 hours P 8 per hour

During January, production of 100 lamps was expected, but 110 lamps were actually
completed. Direct materials purchased and used were 2,100 pounds at an actual
price of P 2.20 per pound. Direct labor cost for the month was P 5,310 and actual
pay per hour was P 9. What was the direct-labor efficiency variance for the month
of January?
A a. P 560 favorable
b. P 560 unfavorable
c. P 630 favorable
d. P 630 unfavorable

24. Controllability and variable costs are different in that


C a. Managers have more influence over variable costs than over
controllable costs
b. Variable costs may be with production or administrative, whereas
controllable costs are only production-related costs
c. Managers have controllability over more than just variable costs
d. Variable costs are only short-run costs

25. In a decentralized company in which divisions may buy goods from one another,
the transfer pricing system should be designed primarily to
D a. Increase the consolidated value of inventory
b. Allow division managers to buy from outsiders
c. Minimize the degree of autonomy of division managers
d. Aid in the appraisal and motivation of managerial performance

Page 4 of 12 0915-2303213  resacpareview@gmail.com


MANAGEMENT ADVISORY SERVICES
ReSA Batch 43 - May 2022 CPALE Batch
07 Feb 2022  6:00 PM to 9:00 PM MAS First Pre-Board Exam
26. Operating decisions primarily deal with
B a. Satisfying shareholders
b. The best use of scarce resources
c. Acquiring equipment and buildings
d. How to obtain funds to acquire resources

27. The Manila Division of Dog Corporation has the following segment information:
Assets available for use P 2,500,000
Target rate of return 12%
Residual income P 300,000

What was Manila Division's return on investment?


C a. 12%
b. 10%
c. 24%
d. 20%

28. When the number of units manufactured increases, the most significant change in
unit cost will be reflected as a(n)
D a. increase in the fixed element
b. decrease in the variable element
c. increase in the mixed element
d. decrease in the fixed element

29. Snake Company has provided you with the following budget information for April:
Cash collections P 876,000
April 1 cash balance 23,000
Cash disbursement 978,600

Snake has a policy of maintaining a minimum cash balance of P 20,000 and borrows
only in P 1,000 increments. How much will Snake borrow in April?
D a. P 79,600
b. P 80,000
c. P 99,000
d. P 100,000

30. Variable costing regards fixed manufacturing overhead as


D a. a product cost
b. an unexpired cost
c. an inventoriable cost
d. a charge against sales

31. Rat Company uses a two-way analysis of overhead variances. Selected data for the
March production activity are as follows:
Actual variable OH incurred P 196,000
Variable OH rate per MH P 6
Standard MHs allowed 33,000
Actual MHs 32,000

Assuming that budgeted fixed overhead costs are equal to actual fixed costs, the
controllable variance for March is
A a. P 2,000 F
b. P 4,000 U
c. P 4,000 F
d. P 6,000 F

32. Yin-Yang Corporation is composed of three operating divisions. Overall, the Yin-
Yang Corporation has a return on investment of 20%. Division A has a return on
investment of 25%. If Yin-Yang Corporation evaluates its managers on the basis
of return on investment, how would the Division A manager and the Yin-Yang
Corporation president react to a new investment that has an estimated return on
investment of 23%?
Division A Manager Ying-Yang Corporation President
C a. accept accept
b. accept reject
c. reject accept
d. reject reject

Page 5 of 12 0915-2303213  resacpareview@gmail.com


MANAGEMENT ADVISORY SERVICES
ReSA Batch 43 - May 2022 CPALE Batch
07 Feb 2022  6:00 PM to 9:00 PM MAS First Pre-Board Exam
33. Relevant revenues and costs focus on:
D a. Last year's net income
b. Monies already earned and/or spent
c. Activities that occurred in the past
d. Differences between the alternatives being considered

34. The material purchases budget tells a manager all of the following except the
D a. Quantity of material to be purchased each period
b. Quantity of material to be consumed each period
c. Cost of material to be purchased each period
d. Cash payment for material each period

Items 35 and 36 are based on the following information


The following information relates to financial projections of Pig Company:
Projected sales 60,000 units
Projected variable costs P 2.00 per unit
Projected fixed costs P 50,000 per year
Projected unit sales price P 7.00

35. How many units would Pig need to sell to earn a profit before taxes of P 10,000?
D a. 25,714
b. 10,000
c. 8,571
d. 12,000

36. If Pig Company achieves its projections, what will be its degree of operating
leverage?
B a. 6.00
b. 1.20
c. 1.68
d. 2.40

37. In a linear programming model, the objective function and the resource
constraints have the same
D a. Constants
b. Coefficients
c. Dependent variables
d. Independent variables

38. The West Division of the Feng Shui Company has the following statistics for its
most recent operations:
Assets available for use (Market Value) P 3,600,000
Assets available for use (Book Value) P 2,000,000
West Division's return on investment 25%
West Division's residual income 200,000
Return on investment (entire Feng Shui Company) 20%
If cost of capital is 10% & tax rate is 40%, what is Economic Value-Added (EVA)?
D a. P 150,000
b. P 90,000
c. P 0
d. P (60,000)

39. As the economy becomes more and more depressed, a company's management decides
to slash spending on research and development. What is the likely effect of this
action on net income? Net income will be
A a. higher this period and lower in future periods
b. higher this period and higher in future periods
c. lower this period and higher in future periods
d. lower this period and lower in future periods

40. Favorable volume variances may be harmful when:


C a. machine repairs cause work stoppages
b. supervisors fail to maintain an even flow of work
c. production in excess of normal capacity cannot be sold
d. there are insufficient sales orders to keep the factory operating at
normal capacity

Page 6 of 12 0915-2303213  resacpareview@gmail.com


MANAGEMENT ADVISORY SERVICES
ReSA Batch 43 - May 2022 CPALE Batch
07 Feb 2022  6:00 PM to 9:00 PM MAS First Pre-Board Exam

41. The primary reason for implementing a new budgeting system is


B a. Managers’ need for more information in making better purchasing
decisions.
b. The relative expected benefits of the new system exceed its costs.
c. Managers’ need for more information for variance analysis.
d. The expected benefits exceed the costs of the old system.

42. The following information regarding fixed production costs from a manufacturing
firm is available for the current year:
Fixed costs in the beginning inventory P 16,000
Fixed costs incurred this period 100,000

Which of the following statements is FALSE?


C a. The maximum amount of fixed production costs that this firm could
deduct using absorption costs in the current year is P 116,000.
b. The minimum amount of fixed costs that firm could deduct for the
current year under variable costing is P 100,000.
c. Using variable costing, this firm will deduct no more than P 16,000
for fixed production costs.
d. If this firm produced substantially more units than it sold in the
current year, variable costing will probably yield a lower income than
absorption costing.

43. In microeconomics, which situation would there be inelastic demand?


B a. A 2% price increase results in a 1% increase in the quantity demanded
b. A 2% price increase results in a 1% decrease in the quantity demanded
c. A 1% price increase results in a 2% increase in the quantity demanded
d. A 1% price increase results in a 2% decrease in the quantity demanded

44. Materials and labor cost standards are generally based on:
D a. Expected actual conditions, anticipated prices, and desired efficiency
levels
b. Theoretical conditions, present price levels, and desired efficiency
levels
c. Capacity conditions, anticipated prices, and desired efficiency levels
d. Normal conditions, present price levels, and desired efficiency levels

45. Monkey Corporation manufactures hats that sell for P 10 per unit. This is its
sole product and it has projected the break-even point at 50,000 units in the
coming period.

If fixed costs are projected at P 100,000, what is the projected variable cost
ratio?
A a. 80 percent
b. 20 percent
c. 40 percent
d. 60 percent

46. The greatest degree of control for committed fixed costs is exerted
C a. in the post-investment audit.
b. during the life of the investment.
c. prior to acquisition.
d. by equipment operators.

47. Budgeted sales for the first six months for Earth Corp. are listed below:
January February March April May June
Units 6,000 7,000 8,000 7,000 5,000 4,000

Earth Corp. has a policy of maintaining an inventory of finished goods equal to


40 percent of the next month’s budgeted sales. If Earth Corp. plans to produce
6,000 units in June, what are budgeted sales for July?
C a. 3,600 units
b. 1,000 units
c. 9,000 units
d. 8,000 units

Page 7 of 12 0915-2303213  resacpareview@gmail.com


MANAGEMENT ADVISORY SERVICES
ReSA Batch 43 - May 2022 CPALE Batch
07 Feb 2022  6:00 PM to 9:00 PM MAS First Pre-Board Exam
48. Expected value in decision analysis is
A a. An arithmetic mean using the probabilities as weights
b. A standard deviation using the probabilities as weights
c. The standard deviation divided by the coefficient of variation
d. A measure of the difference between best possible outcome and outcome
of the original decision

49. Which of the following is not true about an imposed budget?


C a. It uses the knowledge of top management as it relates to resource
availability
b. It reduces the budgeting process time frame
c. It increases the feeling of teamwork
d. It enhances coordination

50. Water Company has developed standard overhead costs based on a capacity of 180,000
machine hours as follows:
Standard costs per unit
Variable portion 2 hours @ P 3 = P 6
Fixed portion 2 hours @ P 5 = 10
P 16

During November, 85,000 units were scheduled for production, but only 80,000 units
were actually produced. The following data relate to November:
 Actual machine hours used were 165,000.
 Actual overhead incurred totaled P 1,378,000 (P 518,000 variable plus P 860,000
fixed).
 All inventories are carried at standard cost.

The fixed overhead volume variance for November was


D a. P 60,000 U
b. P 60,000 F
c. P 100,000 F
d. P 100,000 U

51. In evaluating the profit center manager, the income from operations should be
compared:
B a. across profit centers
b. to budget or past performance
c. to the competition's net income
d. to the total company earnings per share

52. Fire Company produces and sells product with the following unit costs:
Prime costs P 10
Variable indirect manufacturing costs 6
Fixed indirect manufacturing costs 4
Variable marketing costs 8
Fixed marketing costs 3
Total unit cost P 31

The company is considering whether to continue producing the product or simply


buy it from an outside supplier. It has found a supplier which can produce the
product per the company’s specifications.

If the company buys the product, variable marketing cost would be reduced by
60%, but fixed marketing costs would remain the same.

What is the maximum unit price that the company would be willing to pay the
supplier without decreasing its operating income?
B a. P 19.20
b. P 20.80
c. P 24.00
d. P 31.00

53. A firm that is successful in meeting its output goal for a period is said to be
B a. efficient
b. effective
c. profitable
d. exercising cost containment measures

Page 8 of 12 0915-2303213  resacpareview@gmail.com


MANAGEMENT ADVISORY SERVICES
ReSA Batch 43 - May 2022 CPALE Batch
07 Feb 2022  6:00 PM to 9:00 PM MAS First Pre-Board Exam
54. Mr. Lee is an experienced and fully trained shipbuilder based in the Middle East.
Due to significant economic change in supply and demand conditions for
shipbuilding in Mr. Lee's own country, the shipyard he worked for has closed and
he was made redundant.

There was no other local demand for his skills within his own region and he would
have to move to another country to obtain a similar employment, and could only
find similar work locally through undertaking at least a year's retraining in a
related engineering field.

Which of the following describes the type of unemployment that Mr. Lee has been
affected by?
D a. Cyclical unemployment
b. Marginal unemployment
c. Frictional unemployment
d. Structural unemployment

55. The separation of fixed and variable costs is necessary for all of the following
purposes except:
A a. absorption costing and net income analysis
b. direct costing and contribution margin analysis
c. break-even and cost-volume-profit analysis
d. differential and comparative cost analysis

Items 56 & 57 are based on the following information


The Motor Division of Chinese Zodiac Corporation uses 5,000 carburetors per month
in its production of automotive engines. It presently buys all of the carburetors
it needs from two outside suppliers at an average cost of P 100. The Carburetor
Division of Chinese Zodiac manufactures the exact type of carburetor that the
Motor Division requires. The Carburetor Division is presently operating at its
capacity of 15,000 units per month and sells all of its output to a foreign car
manufacturer at P 106 per unit. Its cost structure (on 15,000 units) is:
Variable production costs P 70
Variable selling costs 10
All fixed costs 10

Assume that the Carburetor Division would not incur any variable selling costs
on units that are transferred internally.

56. What is the maximum of the transfer price range for a transfer between the two
divisions?
B a. P 106
b. P 100
c. P 90
d. P 70

57. What is the minimum of the transfer price range for a transfer between the two
divisions?
C a. P 70
b. P 90
c. P 96
d. P 106

58. The flexible budget is


A a. Appropriate for any relevant level of activity
b. Not appropriate when costs and expenses are affected by fluctuations
in volume limits
c. Appropriate for control of direct materials and direct labor but not
for control of factory overhead
d. Appropriate for control of factory overhead but not for control of
direct materials and direct labor

59. From the viewpoint of financial management, profit maximization as a company


goal often fails because it ignores all, EXCEPT
B a. cash flows available to stockholders
b. earnings per share
c. the timing of returns
d. risk

Page 9 of 12 0915-2303213  resacpareview@gmail.com


MANAGEMENT ADVISORY SERVICES
ReSA Batch 43 - May 2022 CPALE Batch
07 Feb 2022  6:00 PM to 9:00 PM MAS First Pre-Board Exam
60. The coefficient of determination indicates:
A a. The percentage of explained variance in the dependent variable
b. Causal relationships among costs and other factors
c. The linear relationship between two variables
d. Whether several variables fluctuate

Items 61 and 62 are based on the following information


Inventory balances and manufacturing cost data for the month of January for
Fortune Company. Under Fortune’s cost system, any over-applied or under-applied
overhead is closed to the cost of goods sold account at the end of the calendar
year.
Inventories: Beginning Ending
Direct materials P 15,000 P 20,000
Work-in-process 7,500 10,000
Finished goods 32,500 25,000

Month of January
Cost of goods manufactured P 257,500
Factory overhead applied 75,000
Direct materials used 95,000
Actual factory overhead 72,000

61. What was the amount of direct material purchases during January?
D a. P 90,000
b. P 95,000
c. P 97,500
d. P 100,000

62. How much direct labor cost was incurred during January?
C a. P 85,000
b. P 87,500
c. P 90,000
d. P 93,000

63. Which classification of costs shall be least relevant in a responsibility


accounting system?
D a. Controllable cost vs. non-controllable cost
b. Discretionary cost vs. committed cost
c. Direct cost vs. indirect cost
d. Product cost vs. period cost

64. The following operating data refer to Lucky Company’s 6-day workweek:
Sum of the hours 174
Sum of the costs 225
Sum of the hours x costs 3,414
Sum of the squared value of costs 4,259

Which equation shall be used under the least-squares method?


D a. 3,414 = 225 a + 4,259 b
b. 3,414 = 174 a + 4,259 b
c. 225 = 174 a + 3,414 b
d. 225 = 6 a + 174 b

65. Which of the following would least likely cause an unfavorable materials quantity
(usage) variance?
A a. Labor that possesses skills equal to those required by the standards
b. Scheduling of substantial overtime
c. A mix of direct materials that does not conform to plan
d. Materials that do not meet specifications

66. Happiness Company recently reported a profit of P 5,000 when the unit variable
cost of its lone product is P 3.00. If the unit selling price is P 5.00, then
how much is the margin of safety?
B a. P 10,000
b. P 12,500
c. P 15,000
d. Cannot be determined from the given information

Page 10 of 12 0915-2303213  resacpareview@gmail.com


MANAGEMENT ADVISORY SERVICES
ReSA Batch 43 - May 2022 CPALE Batch
07 Feb 2022  6:00 PM to 9:00 PM MAS First Pre-Board Exam

67. Pricing decisions do not usually consider


A a. Costs of competitors products
b. Total product absorption costs
c. Prices of competitors products
d. Elasticity of demand for the product

68. The following information is for Yin-Yang Company:


Product A
Revenue P 4.00
Variable Cost P 1.00

Product B
Revenue P 6.00
Variable Cost P 2.00
Total fixed costs are P 40,000.

What is the break-even point, assuming the sales mix consists of 10


units of Product A and 5 units of Product B?
C a. 2,000 units of B and 4,000 units of A
b. 2,025 units of B and 4,050 units of A
c. 4,000 units of B and 8,000 units of A
d. 4,025 units of B and 8,050 units of A

69. Which of the following is least likely to be classified as an investment


center in a responsibility accounting system?
A a. Ayala Mall car park ticket outlet
b. Recto branch of a popular fast-food restaurant
c. Selecta product division of Unilever Philippines
d. Convenience store owned by a large chain organization

70. Two market structures that are imperfectly competitive are oligopoly and
monopolistic competition. What is the difference between an oligopoly and
monopolistic competition?
A a. Oligopoly is about the number of firms while monopolistic
competition is about the variety of products
b. Monopolistic competition is about the number of firms while
oligopoly is about the variety of products
c. Oligopoly has no barriers to entry while monopolistic competition
has low entry barrier
d. Monopolistic competition has no barriers to entry while oligopoly
has low entry barrier

- END –

Page 11 of 12 0915-2303213  resacpareview@gmail.com


MANAGEMENT ADVISORY SERVICES
ReSA Batch 43 - May 2022 CPALE Batch
07 Feb 2022  6:00 PM to 9:00 PM MAS First Pre-Board Exam

Page 12 of 12 0915-2303213  resacpareview@gmail.com


ReSA - THE REVIEW SCHOOL OF ACCOUNTANCY
CPA Review Batch 43  May 2022 CPALE  9 Feb 2022  6:00 PM – 9:00 PM

TAXATION FIRST PRE-BOARD EXAMINATION

INSTRUCTIONS: Select the correct answer for each of the questions. Mark only one
answer for each item by shading the box corresponding to the letter of your choice on
the answer sheet provided. STRICTLY NO ERASURES ALLOWED. Use pencil no. 2 only.

1. Which of the following is not a nonresident citizen?


D a. A citizen of the Philippines who establishes to the satisfaction of
the Commissioner the fact of his physical presence abroad with a
definite intention to reside therein.
b. A citizen of the Philippines who leaves the Philippines during the
taxable year to reside abroad, either as an immigrant or for
employment on a permanent basis.
c. A citizen of the Philippines who works and derives income from sources
abroad and whose employment thereat requires him to be physically
present abroad most of the time during the taxable year.
d. A citizen of the Philippines who goes on a business trip abroad and
stays therein most of the time during the year.

2. First statement: The husband and wife shall compute their individual income tax
separately based on their respective total taxable income.
Second statement: If any income cannot be definitely attributed to or identified
as income exclusively earned or realized by either of the spouses, the same shall
be divided equally between the spouses for the purpose of determining their
respective taxable income.
A a. Both statements are correct
b. Both statements are incorrect
c. Both statements are incorrect
d. Only the second statement is correct

3. Which of the following is not subject to tax as a corporation?


D a. Business partnerships
b. Joint stock companies
c. Insurance companies
d. General professional partnership

4. First statement: All corporations, whether domestic or foreign, shall be taxed


at 20% effectively July, 2020 if their net tax taxable income does not exceed
P5,000,000 and their total assets do not exceed P100,000,000.
Second statement: Effective January 1, 2021, non-resident foreign corporations
shall be taxed at 25% on their net income from sources within the Philippines.
B a. Both statements are correct
b. Both statements are incorrect
c. Only the first statement is correct
d. Only the second statement is correct

5. One of the following is not a requisite of a taxable income.


D a. There must be gain.
b. The gain must be realized or received.
c. The gain must not be excluded by law from taxation.
d. The gain must be that of resident or nonresident citizen.

6. A property was received as donation from Charlwin when its fair market value was
P300,000. Charlwin in turn received this property as donation from Jose when
its fair market value was P350,000. This property was purchased by Marceliano
for P200,000 and was donated to Jose. The property was sold for P500,000.

How much gain (loss) shall be recognized from the sale?


A a. P300,000
b. P200,000
c. P150,000
d. None of the choices

7. If an individual performs services for a creditor who in consideration thereof


cancels the debt, the cancellation of indebtedness may amount to a:
D a. gift.
b. capital contribution.
c. donation inter vivos.
d. payment of income.

Page 1 of 14 0915-2303213  resacpareview@gmail.com


TAXATION
ReSA Batch 43 - May 2022 CPALE Batch
09 Feb 2022  6:00 PM to 9:00 PM TAX First Pre-Board Exam
8. It is important to know the source of income for tax purposes (i.e., from within
and without the Philippines) because:
A a. some individuals and corporate taxpayers are taxed on their
worldwide income while others are taxable only upon income from
sources within the Philippines.
b. the Philippines imposes income tax only on income from sources
within.
c. some individual taxpayers are citizens while others are aliens.
d. export sales are not subject to income tax.

9. Gains, profits and income from the sale of real property are from sources
within the Philippines if:
A a. the real property sold is located in the Philippines.
b. the real property is sold in the Philippines.
c. the real property sold is located outside the Philippines.
d. the real property sold is owned by a resident citizen.

10. The term “capital asset” includes:


B a. stock in trade or other property included in the taxpayer’s inventory.
b. real property not used in the trade or business of taxpayer.
c. property primarily for sale to customers in the ordinary course of
his trade or business.
d. property used in the trade or business of the taxpayer and subject
to depreciation.

11. The widow of your best friend has just been paid P1,000,000 on account of the
life insurance policy of the deceased husband. She asks you whether she shall
declare the amount for income tax purposes or for estate tax purposes.
First advice: The proceeds of the life insurance paid to the beneficiary upon
the death of the insured are exempt from income tax and need not be declared for
income tax purposes.
Second advice: The proceeds of life insurance will have to be declared for estate
tax purposes if the designation of the beneficiary is revocable, otherwise, they
need not be declared.
A a. Both advices are right
b. First advice right; second advice wrong
c. Both advices are wrong
d. First advice wrong; second advice right

12. Ms. Elisya Montenegro entered her short story in a literary contest. She won in
the Short Story category, and received P500,000 for her prize. What was the tax
consequence of the literary prize?
B a. Exempt from income tax
b. Subject to final withholding tax
c. Subject to Section 24 (A)
d. Not subject to any internal revenue tax

13. Under this system, the amount of income tax withheld by the withholding agent is
constituted as a full and final payment of the income tax due from the payee on
the said income.
B a. Creditable withholding tax
b. Final withholding tax
c. Global tax system
d. Schedular tax system

14. Which of the following passive income is not subject to 20% final withholding
tax. As a general rule, which of the following passive income of resident citizen?
D a. Interest from any peso bank deposit
b. Yield or any other monetary benefit from deposit substitutes and
from trust funds and similar arrangements
c. Prizes (except prizes amounting to P10,000 or less)
d. Royalties on books, as well as other literary works and musical
compositions

Page 2 of 14 0915-2303213  resacpareview@gmail.com


TAXATION
ReSA Batch 43 - May 2022 CPALE Batch
09 Feb 2022  6:00 PM to 9:00 PM TAX First Pre-Board Exam
15. Which of the following fringe benefits shall be subject to the fringe benefit
tax?
D a. Benefits given to the rank-and-file employees, whether granted under
a collective bargaining agreement or not
b. Fringe benefits required by the nature of, or necessary to the trade,
business or profession of the employer
c. Fringe benefit given for the convenience or advantage of the employer
d. Membership fees, dues and other expenses borne by the employer for
the managerial and supervisory employee in social and athletic clubs
and similar organizations

16. First statement: The fringe benefit tax is imposed only when the employer is a
corporation.

Second statement: The fringe benefit tax shall be withheld and paid by the
employer in accordance with the provision of Section 57 (A).
D a. Both statements are correct
b. Both statements are incorrect
c. Only the first statement is correct
d. Only the second statement is correct

17. De minimis benefit pertaining to monetized value of vacation and sick leave
credits paid to government officials and employees is limited to:
D a. 10 days
b. 15 days
c. 20 days
d. None of choices

18. The gross estate of this decedent shall only be comprised of properties
situated in the Philippines.
D a. Filipino residing in the Philippines
b. American residing in the Philippines
c. Filipino residing in the USA
d. American residing in the USA

19. Mr. Julian Cruz procured a life insurance upon his own life. He designated his
estate’s executor as an irrevocable beneficiary. For estate tax purposes, the
proceeds of life insurance is:
A a. included in the gross estate of Mr. Julian Cruz because when the
executor of the estate is a beneficiary the proceeds are included in
the gross estate regardless of the designation.
b. not included in the gross estate of Mr. Julian Cruz because the
designation of the beneficiary is irrevocable.
c. included in the gross estate of Mr. Julian Cruz because proceeds of
life insurance are always subject to estate tax.
d. not included in the gross estate because, as a rule, proceeds of life
insurance are generally not subject to estate tax.

20. Mr. Rigoberto Collado, a citizen and resident of Puerto Rico, dies during the
year. Puerto Rico does not impose transfer taxes on properties of decedent not
residing therein. He left the following properties among others:
Shares of stock, San Miguel Corporation, Manila
House and lot, Puerto Rico
Leasehold on a condominium unit, Philippines
Contract for public works, Philippines

The executor of his estate in Philippines asked you what properties are to be
included in his Philippine gross estate. What answer will you give him?
B a. Include all the properties.
b. Include contract for public works and leasehold on condominium unit
only.
c. Include all properties except shares of stock and house and lot
d. Include all properties except house and lot in Puerto Rico.

Page 3 of 14 0915-2303213  resacpareview@gmail.com


TAXATION
ReSA Batch 43 - May 2022 CPALE Batch
09 Feb 2022  6:00 PM to 9:00 PM TAX First Pre-Board Exam
21. A property was transferred mortis causa. The following data were gathered from
the transaction:
Fair market value, time of transfer – P500,000
Fair market value, time of death – P300,000
Consideration received when transferred – P350,000

How much shall be included in the gross estate?


D a. P500,000
b. P300,000
c. P150,000
d. None

22. How much is the allowable standard deduction for non-resident alien decedent?
C a. Five Million Pesos (P5,000,000)
b. Three Million Pesos (P3,000,000)
c. Five Hundred Thousand Pesos (P500,000)
d. None, not allowed to deduct standard deduction

23. The following selected data were taken from the Estate of Ed Sados:
Claim against an insolvent person
(fully uncollectible) P 500,000
Claim against a person who absconded
(fully uncollectible) 300,000
Claim against an insolvent person
(20% collectible) 100,000

How much should be deducted from the gross estate?


B a. P880,000
b. P580,000
c. P100,000
d. P 80,000

24. An unmarried decedent died leaving properties he inherited 4 ½ years ago which
had fair market value of P800,000 at the time of his death (650,000 at the time
of inheritance, and unpaid mortgage of P50,000 paid by the present decedent).
Other properties in his gross estate had fair market value of P1,000,000. The
total expenses, losses indebtedness, taxes and transfer for public purpose
amounted to P300,000.

How much was the vanishing deduction?


D a. P 500,000
b. P 225,000
c. P 200,000
d. P 100,000

25. For purposes of availing of a family home deduction to the extent allowable, a
person may constitute:
B a. as many family homes as possible.
b. only one family home.
c. one family home for each spouse.
d. one family for each child.

26. One of the following is not a distinction between donation inter vivos and
donation mortis causa.
C a. Donation inter vivos takes effect during the lifetime of the grantor
while donation mortis causa takes effect after the death of the
grantor.
b. Donation inter vivos is subject to donor’s tax while donation mortis
causa is subject to estate tax.
c. Donation inter vivos requires a public document while donation mortis
causa may not require a public document.
d. Donation inter vivos is valued at fair market value at the time the
property is given while donation mortis causa is valued at the fair
market value at the time of the death of the grantor.

Page 4 of 14 0915-2303213  resacpareview@gmail.com


TAXATION
ReSA Batch 43 - May 2022 CPALE Batch
09 Feb 2022  6:00 PM to 9:00 PM TAX First Pre-Board Exam
27. Mr. Primitivo Primero died and was survived by his wife and two (2) children,
Faye and Faith. After getting her share in the conjugal property, the surviving
spouse renounced her share in the hereditary estate in favor of Faith to the
exclusion of Faye. Was the renunciation subject to donor’s tax?
A a. Yes, because the renunciation was made categorically in favor of
identified heir to the exclusion or disadvantage of the other co-
heirs.
b. No. because the renunciation was considered a general renunciation.
c. Yes, because , as a rule, renunciation of share in the hereditary
estate is always subject to donor’s tax.
d. No, because, as a rule, the surviving spouse cannot renounce her
share in the hereditary estate.

28. Mr. J. Santos donated P500,000 to AST Foundation, a philanthropic organization.


The organizations total administrative expenses amounted to P5,000,000 out of
its expenses of P20,000,000. How much was the exempt gift to the philanthropic
organization?
C a. P6,000,000
b. P5,000,000
c. P500,000
d. None

29. First statement: For VAT purposes, a taxable person is any person liable to pay
the VAT, whether registered or registrable in accordance with the Tax Code.

Second statement: The status of a “VAT-registered person” as a VAT-registered


person shall continue until the cancellation of such registration.
A a. Both statements are correct
b. Both statements are incorrect
c. Only the first statement is correct
d. Only the second statement is correct

30. Which of the following sales of residential property, shall be subject to VAT
beginning January 1, 2021?
A a. Sale of residential lot the value of which does not exceed P1,500,000
b. Sale of residential house and lot and other residential dwellings
with selling price of not more than P2,000,000.
c. Sale of real properties not primarily held for sale to customers or lease
in the ordinary course of trade or business
d. Sale of real property utilized for socialized housing

31. Which of the following is not included in the term “gross selling price”?
B a. Total amount of money or its equivalent paid by the purchaser
b. Value-added tax passed on by the seller to the buyer
c. Excise tax
d. None of the choices

32. Which statement is incorrect? VAT on importation of goods:


D a. is imposed on an importation for sale or for use in business.
b. is imposed on an importation for personal use.
c. shall be paid prior to removal from customs custody.
d. may not be available as input tax.

33. For VAT purposes, capital goods or properties, also known as depreciable
assets, refer to:
I – goods or properties with estimated useful life greater than one (1)
year
II – treated as depreciable assets under Section 34 (F) of the Tax Code
III – used directly or indirectly in the production or sale of taxable
goods or services
A a. Yes to I, II and III
b. Yes to II and III only
c. Yes to I and II only
d. Yes to I only

Page 5 of 14 0915-2303213  resacpareview@gmail.com


TAXATION
ReSA Batch 43 - May 2022 CPALE Batch
09 Feb 2022  6:00 PM to 9:00 PM TAX First Pre-Board Exam

34. A VAT-registered person shall issue for every sale, barter or exchange of goods
or properties a:
A a. VAT invoice.
b. VAT official receipt.
c. VAT credit certificate.
d. VAT refund.

35. Which of the following shall not be considered livestock for VAT-exempt
transactions purposes?
D a. Cows
b. Bulls and calves
c. Sheep
d. Race horses

36. All of the following cooperatives enjoy VAT exemption on their sales or
receipts except:
D a. agricultural cooperatives.
b. credit and multi-purpose cooperatives.
c. non-agricultural, non-electric and non-credit cooperatives.
d. electric cooperatives.

37. Effective July 1, 2020 to June 30, 2023, the percentage tax rate on persons
exempt from VAT shall be:
A a. 1%.
b. 2%.
c. 3%.
d. 5%.

38. Mr. Jaime Rodriguez is the owner of a small variety store. His gross sales in
any one year do not exceed the VAT threshold amount. He is not VAT-registered.
The following data are taken from the books of the variety store for the quarter
ending March 31, 2020:
Merchandise inventory, December 31, 2019 P 100,000
Gross sales 450,00
Purchases from VAT-registered suppliers 350,000

How much is the percentage tax due and payable?


B a. P22,500
b. P13,500
c. P 9,000
d. P 4,500

39. The following data were provided by an air carrier:


Freight and cargo fees (cargo originating from the
Philippines in a continuous and uninterrupted flight to
Japan, passage documents sold in Japan) P5,000,000
Mail fees (mail originating from Japan in a continuous and
uninterrupted flight to the Philippines, passage documents
sold in the Philippines) 4,000,000
Advance payments for cargo originating from the Philippines
in a continuous and uninterrupted flight to Singapore
(passage documents sold in Singapore) 3,000,000
Receipts from sales of tickets to passengers originating from
the Philippines in a continuous and uninterrupted flight to
Hongkong (tickets sold in the Philippines) 2,000,000
Expenses, Philippines 1,500,000

How much is the common carrier’s tax due from the air carrier assuming it is an
international carrier doing business in the Philippines?
C a. P420,000
b. P300,000
c. P240,000
d. P210,000

Page 6 of 14 0915-2303213  resacpareview@gmail.com


TAXATION
ReSA Batch 43 - May 2022 CPALE Batch
09 Feb 2022  6:00 PM to 9:00 PM TAX First Pre-Board Exam
40. Ms. N is a caterer and a videoke bar operator. In a taxable period, she had the
following data, tax not included:
Sales:
From operations of the Truluv Catering Service:
Cash sales P400,000
Accounts receivable (catering) 250,000
Credit card sales 243,000
From operations of the Dude Videoke Bar:
Cash sales 1,360,000
Credit card sales 624,200
Payments for catering service, based on gross
receipts (80% of which are to VAT taxpayers) 60%

How much is the amusement tax payable?


A a. P357,156
b. P297,630
c. P198,420
d. None of the choices

SITUATIONAL

Ms. Leni operates a convenience store while she offers bookkeeping services to her
clients. In 2018, her gross sales amounted to P800,000.00, in addition to her receipts
from bookkeeping services of P300,000.00. She already signified her intention to be
taxed at 8% income tax rate in her 1st quarter return. How much is the income tax
liability for the yea?

41. Will she qualify to be taxed at 8%?


A a. Yes, because her total gross sales and/or gross receipts and other
non-operating income do not exceed the VAT threshold and she
signifies her intention to be taxed at 8% income tax rate in her
first quarter return.
b. No, because while her total gross sales and/or gross receipts and other
non-operating income do not exceed the VAT threshold and she
signifies her intention to be taxed at 8% income tax rate in her
first quarter return, her income comes from different sources.
c. Yes, because individual taxpayers are always given the option to be
taxed at 8% income tax rate regardless of their total gross sales
and/or gross receipts and other non-operating income.
d. No, because individual taxpayers whose total gross sales and/or
gross receipts and other non-operating income are always subject
to tax under Section 24 (A).

42. How much is her taxable income for the year?


B a. P1,100,000
b. P 850,000
c. P 800,000
d. P 300,000

43. How much is the income tax due?


B a. P88,000
b. P68,000
c. P64,000
d. P10,000

44. Assuming Ms. Leni above, failed to signify her intention to be taxed at 8% income
tax rate on gross sales in her 1st Quarter Income Tax Return, and she incurred
cost of sales and operating expenses amounting to P600,000.00 and P200,000.00,
respectively, or a total of P800,000.00. How much is the income tax?
D a. P88,000
b. P68,000
c. P64,000
d. P10,000

Page 7 of 14 0915-2303213  resacpareview@gmail.com


TAXATION
ReSA Batch 43 - May 2022 CPALE Batch
09 Feb 2022  6:00 PM to 9:00 PM TAX First Pre-Board Exam
45. Using the same information in the immediately preceding number, to what
business tax will she be liable if Ms. Leni failed to signify her intention to
be taxed at 8% income tax rate on gross sales in her 1st Quarter Income Tax
Return?
B a. Value-Added Tax
b. Percentage Tax on Person Exempt From VAT
c. Amusement Tax
d. Excise Tax

SITUATIONAL

A proprietary educational institution has the following data for the calendar year
2021:
Gross receipts, related activities P 15,000,000.00
Cost of services, related activities 6,000,000.00
Allowable deductions from related activities 3,250,000.00
Gross receipts, unrelated activities 18,000,000.00
Cost of services, unrelated activities 5,000,000.00
Allowable deductions from unrelated activities 2,000,000.00
Payments, first three (3) quarters 2,000,000.00

46. How much is net taxable income from related activities?


D a. P16,750,000.00
b. P15,750,000.00
c. P11,000,000.00
d. P 5,750,000.00

47. How much is the net taxable income from unrelated activities?
C a. P16,750,000.00
b. P15,750,000.00
c. P11,000,000.00
d. P 5,750,000.00

48. What corporate tax rate will it be subject to?


A a. 25%
b. 20%
c. 10%
d. 1%

49. How much is the income tax payable?


B a. P4,187,500.00
b. P2,187,500.00
c. P1,675,000.00
d. P1,350,000,00

50. How much is the output VAT, if any?


D a. P3,960,000.00
b. P2,160,000.00
c. P2,160,000.00
d. None, VAT-Exempt

SITUATIONAL

The decedent is a married man with a surviving spouse with the following raw data:
Conjugal real and personal properties (including an automobile
purchased during the marriage using common fund for the
exclusive use of the surviving spouse) P 14,000,000
Conjugal family house 9,000,000
Exclusive family lot 1,000,000
Exclusive properties (including the P3,000,000 exclusive
properties of the surviving spouse) 8,000,000
Conjugal ordinary deductions (including funeral expenses of
P100,000 and judicial expenses of P200,000) 2,300,000
Exclusive ordinary deductions (excluding the P500,000 unpaid
mortgage on the exclusive property of the surviving spouse) 1,000,000
Medical expenses 200,000

Page 8 of 14 0915-2303213  resacpareview@gmail.com


TAXATION
ReSA Batch 43 - May 2022 CPALE Batch
09 Feb 2022  6:00 PM to 9:00 PM TAX First Pre-Board Exam
51. Using BIR Form No. 1801, what value shall be reflected in Schedule 1A (Details
of Family Home)?
A a. P10,000,000
b. P 5,500,000
c. P9,000,000
d. P1,000,000

52. Using BIR Form No. 1801, how much shall be reflected in Line 35 (ordinary
deductions)?
C a. P3,800,000
b. P3,300,000
c. P3,000,000
d. P2,300,000

53. Using BIR Form No. 1801, how much shall be reflected in Line 37D (total special
deductions)?
B a. P15,000,000
b. P10,500,000
c. P10,000,000
d. P 6,000,000

54. Using BIR Form No. 1801, how much shall be reflected in Line 40/16 (net taxable
estate)?
B a. P4,300,000
b. P5,000,000
c. P7,300,000
d. P8,500,000

55. Using BIR Form No. 1801, how much shall be reflected in Line 18 (estate tax due)?
B a. P256,000
b. P300,000
c. P440,000
d. P515,200

SITUATIONAL
Excellent Products, Inc. processes canned fruits and canned sardines. The following
VAT-exclusive selected data for the first quarter of the 2021 calendar year are taken
from its books:
Sale of canned fruits P2,000,000
Sale of canned sardines 3,000,000
Purchase of tomatoes and onions from farmers
(for canned sardines) 500,000
Purchase of corn oil from a supplier
(for canned sardines) 300,000
Purchase of fresh fish from fishermen
(for canned sardines) 400,000
Purchase of fresh fruits from farmers
(for canned fruits) 200,000
Purchase of refined sugar from an agricultural
cooperative that manufactures refined sugar
(for canned fruits) 100,000
Purchase of refined sugar from a refined sugar
refinery (for canned fruits) 50,000
Purchase of packaging materials for the both products 800,000
Purchase of labels (for both products) 150,000
Total bills for trucking services in bringing
canned products to warehouse only P50,000 was paid) 100,000
Monthly VAT payments, previous two months 300,000

56. Using BIR Form No. 2550Q, what shall be reflected in Line 19A (Total
Sales/Receipts)?
A a. P5,000,000
b. P3,000,000
c. P2,000,000
d. None of the choices

Page 9 of 14 0915-2303213  resacpareview@gmail.com


TAXATION
ReSA Batch 43 - May 2022 CPALE Batch
09 Feb 2022  6:00 PM to 9:00 PM TAX First Pre-Board Exam
57. Using BIR Form No. 2550Q, what shall be reflected in Line 19B (output tax due)
using 12% rate?
A a. P600,000
b. P500,000
c. P360,000
d. P240,000

58. Using BIR Form No. 2550Q, what shall be reflected in Line 20D (presumptive
input tax), if any?
C a. P56,000
b. P32,000
c. P20,000
d. None. Not allowed

59. Using BIR Form No. 2550Q, what shall be reflected in Line 24 (total allowable
input tax)?
A a. P182,000
b. P176,000
c. P162,000
d. P156,000

60. Using BIR Form No. 2550Q, what shall be reflected in Line 27 (Tax Still
Payable) using 12% rate?
D a. P418,000
b. P370,000
c. P346,000
d. P118,000

SITUATIONAL

Mr. Nguyen Gandaipen, a non-resident Thai, during the current year donates on January
15, 2022 a brand-new car in the Philippines valued at P1,200,000 and house and lot in
Thailand valued at P2,500,000 to his legitimate son who is getting married in the
Philippines. The son agrees to pay the unpaid tax of P120,000 on the car and the
unpaid mortgage of P500,000 on the house and lot.

61. Using BIR Form No. 1800, what shall be reflected in line 27 (Total Gifts In This
Return)?
C a. P3,700,000
b. P2,500,000
c. P1,200,000
d. None of the choices

62. Using BIR Form No. 1800, what shall be reflected in line 33 (total deductions
allowed)?
B a. P130,000.
b. P120,000.
c. P10,000.
d. none.

63. Using BIR Form No. 1800, what shall be reflected in line 38/14 (total net gift
subject to tax)?
D a. P2,830,000
b. P1,200,000
c. P1,080,000
d. P 830,000

64. Using BIR Form No. 1800, what shall be reflected in line 18 (tax payable)?
D a. P214,200
b. P142,800
c. P64,800
d. P49,800

Page 10 of 14 0915-2303213  resacpareview@gmail.com


TAXATION
ReSA Batch 43 - May 2022 CPALE Batch
09 Feb 2022  6:00 PM to 9:00 PM TAX First Pre-Board Exam
65. Where shall the donor file the donor’s tax return?
C a. Revenue District Office having jurisdiction over the place in the
Philippines where the son is getting married
b. Revenue District Office having the jurisdiction over the place in the
Philippines where the son’s wife resides
c. Philippine Embassy or Philippine Consulate in Thailand
d. None of the choices

SITUATIONAL

The following items (66 – 70) are considered BONUS ITEMS for purposes of determining the score of the
TAX First Pre-Board Exam. The given information for these items were uploaded only around 7:35 PM of
February 9. Some examinees have already submitted the Google Quiz form before 7:35 PM of February 9
while some examinees were able to learn about the uploaded information only AFTER submitting the form.
Ube Paspas Bus is a common carrier by land. It is VAT-registered. During quarter
ending March 31, 2021, it has the following gross receipts:
Transport of passengers P1,000,000
Transport of goods 1,500,000
Transport of cargoes 500,000

66. How much is the common carrier’s tax payable?


D a. P 120,000
b. P 90,000
c. P 60,000
d. P 30,000

67. When shall the Percentage Tax Return be filed?


A a. April 25, 2021
b. April 24, 2021
c. April 21, 2021
d. April 15, 2021

68. How much is the Value-Added Tax?


B a. P 360,000
b. P 240,000
c. P 120,000
d. None. VAT threshold not exceeded

69. When shall the Quarterly VAT be filed?


A a. April 25, 2021
b. April 24, 2021
c. April 21, 2021
d. April 15, 2021

70. Assuming it is not VAT-registered, how much shall be the total percentage
taxes?
C a. P90,000
b. P60,000
c. P50,000
d. P30,000

- END –

Page 11 of 14 0915-2303213  resacpareview@gmail.com


TAXATION
ReSA Batch 43 - May 2022 CPALE Batch
09 Feb 2022  6:00 PM to 9:00 PM TAX First Pre-Board Exam

ANSWERS & SOLUTIONS/CLARIFICATIONS

6. Selling price P500,000


Less: Basis (basis to the last owner who did not acquire the 200,000*
property by donation)
Gain from sale P300,000

23. Claim against an insolvent person (fully uncollectible P500,000


Claim against an insolvent person (80% x 100,000) 80,000
Total P580,000

24. Value to take P 650,000


Less: Mortgage paid 50,000
Initial basis 600,000
Less: Proportional deduction (P600,000/P1,800,000 X P300,000) 100,000
Final basis 500,000
Rate 20%
Vanishing deduction P 100,000

28. The gift was exempted because the administrative expenses of the donee-institution did not
exceed 30% of the total expenses (5,000,000/P20,000,000 = 25%).

38. Gross sales P450,000


Tax rate 3%
Percentage tax 13,500
Effective July 1, 2020 until June 30, 2023, the rate shall be 1%.

39. Freight and cargo fees (cargo originating from the Philippines in a continuous
and uninterrupted flight to Japan, passage documents sold in Japan) P5,000,000
Advance payments for cargo originating from the Philippines in a continuous
and uninterrupted flight to Singapore (passage documents sold in Singapore) 3,000,000
Gross receipts 8,000,000
Tax rate 3%
Common carrier’s tax P 240,000

40. Cash sales P1,360,000


Credit card sales 624,200
Gross receipts (Dude Videoke Bar) P1,984,200
Tax rate 18%
Amusement tax P 357,156

42. Gross sales – Convenience store P800,000.00


Gross receipts - Bookkeeping 300,000.00
Total gross sales/receipts 1,100,000.00
Less: Amount allowed as deduction 250,000.00
Taxable income P850,000.00

43. Taxable income P850,000.00


Tax rate 8%
Tax due P 68,000.00

44. Gross sales – Convenience store P 800,000.00


Gross receipts - Bookkeeping 300,000.00
Total gross sales/receipts 1,100,000.00
Less: Cost of sales 600,000.00
Gross income 500,000.00
Less: Operating expenses 200,000.00
Taxable income P 300,000.00
Tax due 250,000.00 Exempt
50,000.00 x 20% 10,000.00 P 10,000.00

Aside from income tax, Ms. Leni is likewise liable to pay business tax.

Page 12 of 14 0915-2303213  resacpareview@gmail.com


TAXATION
ReSA Batch 43 - May 2022 CPALE Batch
09 Feb 2022  6:00 PM to 9:00 PM TAX First Pre-Board Exam

45. Effective January 1, 2018 to December 31, 2022


If the taxable income is:
Over But not over The tax shall be Plus Of excess over
P 250,000 0%
P 250,000 400,000 20% P 250,000
400,000 800,000 P 30,000 25% 400,000
800,000 2,000,000 130,000 30% 800,000
2,000,000 8,000,000 490,000 32% 2,000,000
8,000,000 2,419,000 35% 8,000,000

Related activities Unrelated activities Total


Gross receipts P15,000,000.00 P18,000,000.00 P33,000,000.00
Less: Cost of services 6,000,000.00 5,000,000.00 11,000,000.00
Gross income 9,000,000.00 13,000,000.00 22,000,000.00
Less: Allowable deductions 3,250,000.00 2,000,000.00 5,250,000.00
Net taxable income (46)P 5,750,000.00 (47) P11,000,000.00 16,750,000.00
Tax rate* (48) 25%
Income tax due 4,187,500.00
Less: Payments, first 3 quarters 2,000,000.00
Income tax payable (49)P2,187,500.00

51. Conjugal family house P9,000,000


Exclusive family lot 1,000,000
Total P10,000,000

52. Conjugal ordinary deductions (2,300,000 – 300,000) P2,000,000


Exclusive deductions 1,000,000
Total P3,000,000

53. Standard deduction P5,000,000


family home
Family house (1/2 x 9,000,000) 4,500,000
Family lot (full) 1,000,000
Total 5,500,000
Maximum 10,000,000
Lower 5,500,000
Total P10,500,000

Exclusive Common Total


34 Gross estate P6,000,000 P23,000,000 P29,000,000
35 Less: Ordinary deductions 1,000,000 2,000,000 3,000,000
36 Estate after ordinary deductions 5,000,000 21,000,000 26,000,000
37 Less: Special deductions
37A Standard deduction 5,000,000
37B Family home 5,500,000
37C Others (specify) -
37D Total special deductions 10,500,000
38 Net estate 15,500,000
39 Less: Share of surviving spouse (1/2 x
21,000,000*) 10,500,000
40/16 Net taxable estate 5,000,000 (54)
17 Applicable tax rate 6%
18 Estate tax due P 300,000 (55)

56. Sale of canned fruits P2,000,000


Sale of canned sardines 3,000,000
Total P5,000,000

57. Gross sales P5,000,000


Tax rate 12%
Output tax P600,000

58. Presumptive input tax on purchase of tomatoes and onions (500,000 x 4% = P20,000))

Page 13 of 14 0915-2303213  resacpareview@gmail.com


TAXATION
ReSA Batch 43 - May 2022 CPALE Batch
09 Feb 2022  6:00 PM to 9:00 PM TAX First Pre-Board Exam

59. Presumptive input tax on purchase of tomatoes and onions (500,000 x 4%) P20,000
Passed-on VAT on purchase of corn oil (300,000 x 12%) 36,000
Passed-on VAT on purchase of refined sugar from sugar refinery (50,000 x 12%) 6,000
Passed-on VAT on purchase of packaging materials (800,000 x 12%) 96,000
Passed-on VAT on purchase of labels (150,000 x 12%) 18,000
Passed-on VAT on trucking services (50,000 x 12%) 6,000
Total P182,000

60. 19B Output tax P600,000


24 Total allowable input tax 182,000
25 Net VAT payable 418,000
Less: 26A Monthly VAT payments, previous two months 300,000
27 Tax Still Payable P118,000

61. Car, Philippines P1,200,000

62. Unpaid tax on car assumed by the donee P120,000

63. 27 Total Gifts In This Return P1,200,000


Less: Deductions
Unpaid tax on car assumed by the donee
33 Total Deductions Allowed 120,000
34 Total net gifts in this return 1,080,000
Add: 35 Total prior net gifts during the calendar year -
36 Total net gifts 1,080,000
37 Less: Exempt gift 250,000
38/14 Total net gift subject to tax P 830,000

64. 38/14 Total net gift subject to tax 830,000


15 Applicable donor’s tax rate 6%
16 Total donor’s tax due 49,800
Less: 17Tax credit/payments -
18 Tax payable P49,800
Add:19 Penalties -
20 Total amount payable P49,800

65. 27 Total Gifts In This Return P1,200,000


Less: Deductions
Unpaid tax on car assumed by the donee
33 Total Deductions Allowed 120,000
34 Total net gifts in this return 1,080,000
Add: 35 Total prior net gifts during the calendar year -
36 Total net gifts 1,080,000
37 Less: Exempt gift 250,000
38/14 Total net gift subject to tax 830,000
15 Applicable donor’s tax rate 6%
16 Total donor’s tax due 49,800
Less: 17Tax credit/payments -
18 Tax payable P49,800
Add:19 Penalties -
20 Total amount payable P49,800

66. Gross receipts (transport of passengers) P1,000,000


Tax rate 3%
Common carrier’s tax P 30,000

68. Gross receipts (transport of goods) P1,500,000


Gross receipts (transport of cargoes 500,000
Total 2,000,000
Tax rate 12%
Value-added tax P 240,000

70. Common carrier’s tax (transport of passengers)


(1,000,000 x 3%) P30,000
Percentage on Persons Exempt from VAT (transport of goods and cargoes)
(2,000,000 x 1%) 20,000
Total P50,000

Page 14 of 14 0915-2303213  resacpareview@gmail.com


ReSA - THE REVIEW SCHOOL OF ACCOUNTANCY
CPA Review Batch 43  May 2022 CPALE  8 Feb 2022  6:00 PM – 9:00 PM

AUDITING FIRST PRE-BOARD EXAMINATION

INSTRUCTIONS: Select the correct answer for each of the questions. Mark only one
answer for each item by shading the box corresponding to the letter of your choice on
the answer sheet provided. STRICTLY NO ERASURES ALLOWED. Use pencil no. 2 only.
1. A practitioner's report on agreed-upon procedures should contain which of the
following statements?
a. The procedures performed were those agreed to by the specified parties
identified in the report.
b. Sufficiency of procedures is the responsibility of the practitioner.
c. All classification codes appeared to comply with such performance
documents.
d. Nothing came to my attention as a result of applying the procedures.
2. The following, except one, is always present in assurance engagements. Select
the exception:
a. The issuance of a written report.
b. The consideration of internal control.
c. The presence of written assertions which is the responsibility of
another party.
d. Independence of mind and in appearance on part of the auditor.
3. While this type of engagement involves the application of audit skills and
techniques and the gathering of evidence, it does not ordinarily involve an
assessment of accounting and internal control systems, tests of records, and of
responses to inquiries by obtaining corroborating evidence through inspection,
observation, confirmation, and computation.
a. Compilation
b. Review
c. Agreed-upon procedures
d. Consultancy
4. Which of the following groups is considered a subgroup ordinarily charged with
assisting the board of directors in fulfilling its oversight responsibilities?
a. Audit committee.
b. Secured creditors.
c. Internal auditors.
d. Senior management.
5. Governmental auditing often extends beyond examinations leading to the expression
of opinion on the fairness of financial presentation and includes audits of
efficiency, economy, effectiveness, and also
a. Accuracy
b. Compliance
c. Evaluation
d. Internal control.
6. Which of the following statements correctly defines the term reasonable
assurance?
a. A substantial level of assurance to allow an auditor to detect a
material misstatement.
b. A significant level of assurance to allow an auditor to detect a
material misstatement.
c. An absolute level of assurance to allow an auditor to detect a
material misstatement.
d. A high, but not absolute, level of assurance to allow an auditor to
detect a material misstatement.
7. According to the IFAC Code of Ethics for Professional Accountants, audit teams
are required to be independent of the audit client during the engagement period
and during which other period?
a. The fiscal year following the period covered by the financial
statements.
b. The period covered by the financial statements.
c. The calendar years that include any part of the period covered by the
financial statements.
d. The two years prior to the period covered by the financial statements.
8. Which of the following types of risks most likely would increase if accounts
receivable are confirmed 3 months before year end?
a. Inherent.
b. Control.
c. Detection.
Page 1 of 20 0915-2303213 
resacpareview@gmail.com
AUDITING
ReSA Batch 43 - May 2022 CPALE Batch
08 Feb 2022  6:00 PM to 9:00 PM AUD First Pre-Board Exam
d. Business.
9. An auditor is unable to obtain absolute assurance that misstatements due to fraud
will be detected for all of the following except
a. Employee collusion.
b. Falsified documentation.
c. Need to apply professional judgment in evaluating fraud risk factors.
d. Professional skepticism.
10. The following statements relate to the Board of Accountancy. Which statement is
incorrect?
a. The Board consists of a Chairman and six members.
b. The Chairman and members are appointed by the President of the
Philippines upon recommendation of PRC.
c. No person shall be appointed a member of the Board unless he is
natural-born citizen of the Philippines, a duly registered CPA and
has been in the practice of accountancy for at least ten years.
d. The Professional Regulation Commission may remove from the Board any
member whose certificate to practice has been removed or suspended.
11. In accordance with PRC Resolution No. 254 Series of 2017, an individual born on
March 2, 1996 has applied for accreditation with the BOA to practice public
accounting was approved on April 30, 2019. The registration shall expire on
a. September 30, 2022
b. March 2, 2022
c. December 31, 2022
d. April 30, 2022
12. The following were the ratings of examinees who took Licensure Examination for
CPAs (LECPA) in October 2022.
S1 S2 S3 S4 S5 S6
Examinee 1 95% 95% 95% 95% 95% 95%
Examinee 2 75% 75% 75% 75% 75% 75%
Examinee 3 82% 73% 74% 74% 74% 74%
Examinee 4 64% 74% 85% 85% 85% 85%
Examinee 5 75% 75% 75% 74% 74% 74%
Examinee 6 74% 74% 74% 74% 74% 74%
Examinee 7 100% 100% 100% 64% 74% 74%
Examinee 8 64% 95% 86% 74% 75% 77%
Examinee 9 100% 100% 100% 31% 32% 75%
Examinee 10 100% 74% 74% 74% 74% 74%
Of the ratings presented above, how many examinees obtained a conditional status
in the Board Exam?
a. 4
b. 2
c. 3
d. 5
13. How should differences of opinion between the engagement partner and the quality
control reviewer be resolved?
a. By adhering to industry best practices.
b. By following the firm's policies and procedures.
c. By accepting the recommendations of the client's audit committee.
d. By issuing a disclaimer of opinion and reporting the issue to those
charged with the entity's governance.
14. In accordance with SRC Rule No. 68 (Revised 2019), what is the minimum number of
years in which an auditor should retain the audit documentation following the
report release date?
a. 10 years
b. 7 years
c. 5 years
d. 3 years
15. An entity requests that a CPA change an audit engagement to a review engagement.
If the accountant agrees to the change, how, if at all, should the accountant’s
review report be modified?
a. The accountant should issue the review report without mentioning the
change in engagement.
b. The accountant should include in the review report a disclaimer of
an audit opinion.
Page 2 of 20 0915-2303213  resacpareview@gmail.com
AUDITING
ReSA Batch 43 - May 2022 CPALE Batch
08 Feb 2022  6:00 PM to 9:00 PM AUD First Pre-Board Exam
c. The accountant should include in the review report the circumstances
that resulted in the change in engagement.
d. The accountant should include in the review report a reference to the
original engagement but not the reason for the change.

16. The primary objective of an auditor when considering the acceptance of an initial
audit engagement of a non-issuer is to
a. Establish whether the preconditions for an audit are present.
b. Agree with management on the timing of tests at interim and year end.
c. Limit the auditor's responsibility if management fails to provide
written representations.
d. Specify the degree to which management intends to rely on the
auditor's testing of internal controls.
17. When an auditor of a parent is also the auditor of a component, then each of the
following factors would ordinarily influence the decision to obtain a separate
engagement letter from the component, except:
a. The legal requirements regarding the appointment of the auditor.
b. Whether a separate audit report is to be issued on the component.
c. Whether there has been any turnover of the component's board members.
d. The degree of independence of the component management from the parent
entity.
18. An auditor who performed analytical procedures that compared current-year
financial information to the comparable prior period noted a significant increase
in net income. Given this result, which of the following expectations of recorded
amounts would be unreasonable?
a. A decrease in costs of goods sold as a percentage of sales.
b. A decrease in accounts payable.
c. A decrease in retained earnings.
d. A decrease in notes payable.

19. Which of the following results of analytical procedures would most likely
indicate possible unrecorded liabilities?
a. Current ratio of 2:1 as compared to 5:1 for the prior period.
b. Ratio of accounts payable to total current liabilities of 4:1,
compared to 6:1 for the prior period.
c. Accounts payable turnover of 5, compared to 10 for the prior period.
d. Accounts payable balance increase greater than 10 percent over the
prior period.

20. Under which of the following circumstances would an auditor be considered to be


using the work of a specialist?
a. The auditor engages a lawyer to interpret the provisions of a complex
contract.
b. The auditor makes inquiries of the client's lawyer regarding pending
litigation.
c. A tax expert employed by the auditor's CPA firm reviews the client's
tax accruals.
d. The client engages an outside computer service organization to
prepare its payroll.
21. Based on new information gained during an audit of an entity, an auditor
determines that it is necessary to modify materiality for the financial
statements as a whole. In this circumstance, which of the following statements
is accurate?
a. The auditor is required to reperform audit procedures already
completed on the audit using the revised materiality.
b. The auditor should consider disclaiming an opinion due to a scope
limitation.
c. The revision of materiality at the financial statement levels will
not affect the planned nature and timing of audit procedures, only
the extent of those procedures.
d. Materiality levels for particular classes of transactions, account
balances, or disclosures might also need to be revised.

Page 3 of 20 0915-2303213  resacpareview@gmail.com


AUDITING
ReSA Batch 43 - May 2022 CPALE Batch
08 Feb 2022  6:00 PM to 9:00 PM AUD First Pre-Board Exam
22. Which of the following statements is correct regarding the predictability of
analytical procedures in a financial statement audit?
a. Relationships involving only balance sheet accounts tend to be more
predictable than relationships involving income statement accounts.
b. Relationships involving income statement accounts tend to be more
predictable than relationships involving only balance sheet accounts.
c. Relationships involving transactions subject to management discretion
tend to be more predictable than automated transactions.
d. Relationships in a dynamic environment tend to be more predictable
than relationships in a stable environment.
23. Management's responses to inquiries can be corroborated by each of the following,
except:
a. Visits to the entity's premises and plant facilities.
b. Inspection of documents and internal control manuals.
c. Preparation of the summary of unadjusted differences.
d. Observation of entity activities and operations.
24. In performing interviews and examining documents related to preliminary work in
a financial statement audit of an entity, an auditor identifies a business risk
associated with plans for a new product line. What should the auditor do as a
result?
a. Modify the scope of the engagement to include an analysis of the
budget for the new product line and consider the new risk in
conjunction with other risks after the budget items have been
analyzed.
b. Analyze the newly identified risk in conjunction with economic
circumstances related exclusively to the new product line and
consider whether there is an immediate consequence for the risk of
material misstatement for affected classes of transactions.
c. Modify the financial statement disclosures to include the newly
identified risk if it is likely that the new product line will have
an adverse effect on the company’s profitability.
d. Analyze the newly identified risk in conjunction with other known
business risks and consider whether there is an immediate consequence
for the risk of material misstatement at various levels of the audit.
25. Which of the following statements correctly describes the "top-down approach"
used during an audit of internal control over financial reporting?
a. Begin reviewing balance sheet accounts and then review income
statement accounts.
b. Begin reviewing income statement accounts and then review balance
sheet accounts.
c. Begin by understanding the overall risks to internal control over
financial reporting at the financial statement level.
d. Begin by understanding the overall risks to internal control over
financial reporting at the general ledger level.
26. Manual controls would most likely be more suitable than automated controls for
which of the following?
a. Large, unusual, or nonrecurring transactions.
b. High-volume transactions that require additional calculations.
c. Situations with routine errors that can be predicted and corrected.
d. Circumstances that require a high degree of accuracy.
27. Which of the following most accurately describes the process of a walkthrough?
a. Testing and documenting the results of tests of selected controls.
b. Inspection of selected documents, records, and internal control
documentation.
c. Observation of an entity's activities and operations.
d. Following a transaction from its origination until it is reflected
in the financial statements.
28. Which of the following activities by small business clients best demonstrates
management integrity in the absence of a written code of conduct?
a. Emphasizing ethical behavior through oral communication and
management example.
b. Developing and maintaining formal descriptions of accounting
procedures.

Page 4 of 20 0915-2303213  resacpareview@gmail.com


AUDITING
ReSA Batch 43 - May 2022 CPALE Batch
08 Feb 2022  6:00 PM to 9:00 PM AUD First Pre-Board Exam
c. Documenting internal control procedures using flowcharts rather than
narratives.
d. Reporting regularly to the board of directors about operations and
finances.
29. If an auditor is obtaining an understanding of an entity’s information and
communication component of internal control, which of the following factors
should the auditor assess?
a. The integrity and ethical values of top management.
b. The philosophy and operating style of management to promote effective
internal control over financial reporting.
c. The classes of transactions in the issuer’s operations that are
significant to the issuer’s financial statements.
d. The oversight responsibility over financial reporting and internal
control by the board or audit committee.
30. Which of the following situations represents a limitation, rather than a failure,
of internal control?
a. A jewelry store employee steals a small necklace from a display
cabinet.
b. A bank teller embezzles several hundred dollars from the cash drawer.
c. A purchasing employee and an outside vendor participate in a kickback
scheme.
d. A movie theater cashier sells reduced-price tickets to full-paying
customers and pockets the difference.
31. When planning an engagement to examine the effectiveness of the entity’s internal
control, a practitioner would be least likely to consider which of the following
factors?
a. Preliminary judgments about the effectiveness of internal control.
b. The extent of recent changes in the entity and its operations.
c. The type of available evidential matter pertaining to the
effectiveness of the entity’s internal control.
d. The evaluation of the operating effectiveness of the controls.
32. How would an auditor most appropriately respond to a heightened assessed risk of
material misstatement?
a. By obtaining a management representation
b. By performing analytical procedures, but not substantive procedures,
at period end.
c. By assigning more experienced staff or those with specialized skills
to high-risk areas.
d. By performing tests of controls at interim-and period-end dates.
33. All else being equal, as the level of materiality decreases, the amount of
evidence required will:
a. Remain the same
b. Increase
c. Decrease
d. Change in an unpredictable fashion
34. An auditor with the CPA firm of Winston and Churchill is working to understand
a client’s inventory procurement system. In hopes of assessing the control risk
present in this system, the auditor is reviewing a flowchart created by company
employees. One symbol has a diamond shape. What does that symbol represent?
a. A document within the system
b. A decision made within the system
c. A process carried out within the system
d. The input of information within the system
35. Prior to commencing field work, an auditor usually discusses the general audit
strategy with the client's management. Which of the following details do
management and the auditor usually agree upon at this time?
a. The specific matters to be included in the communication with the
audit committee.
b. The minimum amount of misstatements that may be considered to be
significant deficiencies.
c. The schedules and analyses that the client's staff should prepare.
d. The effects that inadequate controls may have over the safeguarding
of assets.

Page 5 of 20 0915-2303213  resacpareview@gmail.com


AUDITING
ReSA Batch 43 - May 2022 CPALE Batch
08 Feb 2022  6:00 PM to 9:00 PM AUD First Pre-Board Exam
PROBLEM 1:
You are auditing the financial statements of Spurs Corp. as of and for the period ended
December 31, 2021. Spurs Corp. is a supplier of school and office supplies and is
keeping records under cash basis. The following were derived from the cash records of
the company:
Total collections from customers including recovery of P3,556,000
previously written-off accounts
Total payments to suppliers of merchandise 1,876,000
Total cash payments for operating expenses 950,000
Total cash refund collected on goods returned to suppliers 15,000
Total cash refund payment on goods returned by customers 28,000

Audit notes:
a. Outstanding customer invoices at the beginning and at the end of the year were
at P215,000 and P298,000, respectively. Total sales returns was at P149,000
(including refunds) while total sales discount taken by customers was at P92,000.
Total customer invoices written off during the year was at P32,000 while P19,000
of the previously written-off accounts were recovered and collected during the
year.
Customer advances at the end of the year was at P53,000.
b. Outstanding supplier invoices at the beginning and at the end of the year were
at P184,000 and P159,000, respectively. Total purchase returns was at P120,000
(including refunds) while total purchase discount taken by the company was at
P74,000.
c. Merchandise inventory balances at the beginning and at the end of the year were
at P154,000 and P211,000, respectively.
d. Accrual and deferrals at the beginning and at the end of the year were as follows:
Beginning Ending
Accrued operating expenses 24,000 34,000
Prepaid operating expenses 19,000 12,000

Requirements:
36. What is the accrual basis Gross Sales?
a. 3,812,000 c. 3,831,000
b. 3,780,000 d. 3,840,000
37. What is the accrual basis Cost of Sales?
a. 1,789,000 c. 1,779,000
b. 1,836,000 d. 1,794,000
38. What is the accrual basis Operating Expenses?
a. 947,000 c. 933,000
b. 953,000 d. 967,000

PROBLEM 2:
You are auditing for the first time the financial statements of Lakers Inc. for the
period ended December 31, 2021. Lakers Inc. which is the exclusive distributor of
Daikin Appliances in the Philippines has started its operations in 2019.
Your examination revealed the following:
a. Net income as reported by the client and dividends declaration and payment for
each year are as follows:
2019 2020 2021
Net income 987,0000 1,259,000 1,980,000
Dividends declared and paid - 500,000 1,000,000
b. The following were omitted at each year end:
2019 2020 2021
Accrued salaries expense 5,000 8,000 12,000
Accrued utilities expense 14,000 - 7,000
Unused office supplies - 4,000 9,000
Unearned rent income - 6,000 2,000
Accrued royalty income 3,000 - 1,000

Page 6 of 20 0915-2303213  resacpareview@gmail.com


AUDITING
ReSA Batch 43 - May 2022 CPALE Batch
08 Feb 2022  6:00 PM to 9:00 PM AUD First Pre-Board Exam
c. Goods received at the end of each year, recorded as purchases only upon payment
the following year. These goods were appropriately included in the physical count
at each year end.
2019 2020 2021
Invoice price - 25,000 32,000
d. Collections from customers as at each year end for goods delivered the following
year. Sales was recorded upon collection.
2019 2020 2021
24,000 - 41,000
e. Major repairs costs incurred at the beginning of 2020 amounted to P210,000. The
amount was charged as outright repairs and maintenance expense. The repairs
however was necessary to extend the remaining useful life of the related equipment
from 4 years to 7 years. The company uses straight-line method of depreciating
its properties.
Requirements:
39. What is the adjusted net income in 2019?
a. 947,000 c. 944,000
b. 917,000 d. 954,000
40. What is the adjusted net income in 2021?
a. 1,931,000 c. 1,901,000
b. 1,892,000 d. 1,879,000
41. What is the retrospective adjustment to the retained earnings beginning 2021 as
a result of your audit?
a. 145,000 credit c. 115,000 credit
b. 35,000 debit d. 65,000 debit
42. What is the effect of all the errors to the total assets as of December 31,
2021?
a. 10,000 understated c. 190,000 understated
b. 150,000 understated d. 160,000 understated

PROBLEM 3:
You were assigned to audit the shareholders’ equity transactions and account balances
of your audit firm’s client Celtics Corp. The company has the following items in the
stockholders’ equity portion of its statement of financial position as of December 31,
2021, after all necessary year- end closing entries:
9% Cumulative preference shares, P50 par value,
110,000 shares issued and outstanding; Each preference share is P5,500,000
convertible to 3 ordinary shares
Ordinary shares, P25 par value, 265,000 shares issued; 6,625,000
Subscribed preference shares, net of P350,000 subscription receivable 1,650,000
Share premium from preference shares 3,190,000
Share premium from ordinary shares 2,300,000
Share premium from treasury stock transactions – ordinary shares 22,000
Retained Earnings 5,500,000
Treasury stock – ordinary shares, 10,000 shares (380,000)
Total Stockholders’ Equity ?
Your test of details of shareholders’ equity transactions during the year (2021)
revealed the following:
a) On April 30, the company received P1.8M cash for issuing P1M, 12% bonds payable and
20,000 ordinary shares. The bonds which pay annual interest every December 31,
were quoted in the market at 104 (excluding accrued interest). Ordinary shares were
currently selling in the market at P32 per share. The transactions was recorded by
the company as:

Cash 1,800,000
Bonds payable 1,000,000
Ordinary shares 500,000
Share premium from ordinary shares 300,000
b) On June 1, the company reissued 15,000 shares of its treasury shares ordinary shares
in lieu of an equipment with a fair market value of P520,000. The company originally

Page 7 of 20 0915-2303213  resacpareview@gmail.com


AUDITING
ReSA Batch 43 - May 2022 CPALE Batch
08 Feb 2022  6:00 PM to 9:00 PM AUD First Pre-Board Exam
reacquired 30,000 treasury shares as one bloc in 2020 at P38 per share. The company
recorded the reissuance in the books as:
Equipment 570,000
Treasury shares 570,000
c) On September 1, the company retired 5,000 shares of its ordinary shares from the
treasury. The company recorded the retirement as:
Ordinary shares 125,000
Retained earnings 65,000
Treasury shares 190,000
All ordinary shares, with the exception of those issued on June 30 of the current
year were issued during the company’s initial public offering at P33 per share.
d) On December 31, the company issued share rights to ordinary shareholders. Four share
rights plus P40 shall entitle the holder to acquire ordinary shares. Share rights
are exercisable up to 2 years from date of issuance. This transaction has not been
recorded yet per books as of the balance sheet date.
Requirements:
43. What is the necessary adjusting journal entry related to the share issuance on
April 30 (item a)?
a. Dr. Share premium from ordinary shares 80,000
Cr. Bonds payable 80,000
b. Dr. Share premium from ordinary shares 80,000
Cr. Bonds payable 40,000
Cr. Interest expense 40,000
c. Dr. Interest expense 40,000
Cr. Share premium from ordinary shares 40,000
d. Dr. Interest expense 40,000
Cr. Bonds payable 40,000
44. What is the adjusting journal entry related to the treasury shares reissue on
June 1?
a. Dr. Retained earnings 50,000
Cr. Equipment 50,000
b. Dr. Share premium from treasury shares 50,000
Cr. Equipment 50,000
c. Dr. Share premium from treasury shares 22,000
Dr. Retained earnings 28,000
Cr. Equipment 50,000
d. Dr. Share premium from ordinary shares 28,000
Dr. Share premium from treasury shares 22,000
Cr. Equipment 50,000
45. What is the necessary adjusting journal entry related to the treasury shares
retirement on September 1?
a. Dr. Share premium from treasury shares 65,000
Cr. Retained earnings 65,000
b. Dr. Share premium from ordinary shares 25,000
Cr. Retained earnings 25,000
c. Dr. Share premium from ordinary shares 65,000
Cr. Retained earnings 65,000
d. Dr. Share premium from ordinary shares 40,000
Cr. Retained earnings 40,000
46. Assuming that 80% of the share rights issued on December 31, 2021 were
subsequently exercised, the entry to record the exercise shall require a:
a. Credit to Share premium on ordinary shares at P765,000.
b. Credit to Ordinary shares at P1,325,000.
c. Credit to Ordinary shares at P1,200,000.
d. Credit to Share premium on ordinary shares at P795,000
47. Assuming that 30,000 of the preference shares were subsequently converted to
ordinary shares, the entry to record the conversion shall require a:
a. Credit to Share premium on ordinary shares at 870,000.
b. Credit to Ordinary shares at P1,500,000.
c. Debit to Share premium on preference shares at P120,000.
d. Credit to Share premium on ordinary shares at P120,000
Page 8 of 20 0915-2303213  resacpareview@gmail.com
AUDITING
ReSA Batch 43 - May 2022 CPALE Batch
08 Feb 2022  6:00 PM to 9:00 PM AUD First Pre-Board Exam
PROBLEM 4:
On January, 2019 Thunder Corp. grants each of its 100 employees in the sales department
share options. The share options will vest at the end of 2021, provided that the
employees remain in the entity’s employ and provided that the sales increase by at
least 100% by 2021. Actual sales in 2018 (base year) is 2M units. If the sales volume
increase by an average of 100% to 120% by 2021, each employee will receive 200 options
each. If sales volume increase by 121%-150% by 2021, each employee will receive 300
options each. If sales volume increase by more than 150% by 2021, each employee will
receive 400 options each. Five options plus P120 shall entitle the holder to acquire
one ordinary shares (P100 par) at any time up to December 31, 2023.
On the grant date, the company estimates that the share options have a fair value of
P21 per option. There has been a 30% average increase in annual sales for the past
three years and that the company expects the same pattern during the vesting period.
The following information are deemed relevant for your analysis:
Actual employees Estimated additional Actual
Year leaving the employees who will leave Sales
company by the end 2021 (Units)
2019 4 5 2,500,000
2020 2 4 3,500,000
2021 9 - 5,100,000
Requirements:
48. What is the compensation expense in 2019?
a. 127,400 c. 191,100
b. 254,800 d. 134,400
49. What is the compensation expense in 2021?
a. 378,000 c. 363,000
b. 238,000 d. 336,000
50. Assuming that 40% of the options granted to employees were exercised, the entry
to record the exercise shall require a credit share premium at:
a. 320,000 c. 280,000
b. 340,000 d. 312,000
51. Your audit program of auditing shareholders’ equity balances would necessarily
include a procedure to give reference to market quotations in an attempt to
gather evidence regarding valuation assertion in which of the following
shareholder’s equity balances?
a. 20% share dividends payable.
b. Treasury share.
c. Ordinary share warrants outstanding.
d. Subscriptions receivable balance.

PROBLEM 5:
Your examination of the retained earnings account of Jazz Corporation in line with
your audit its financial statements for the period ended December 31, 2021 revealed
the following:
a. The general ledger of the company’s retained earnings included the following
entries:
Debit Credit
Beginning balance 5,290,000
a. Write-off of worthless inventories (no 135,000
allowance has been provided in the prior period)
b. 15% share dividend declaration 1,500,000
c. Understatement in 2020 accrued salaries expense 40,000
d. Loss from reissue of treasury shares 100,000
e. Gain from early retirement of bonds payable 54,000
f. Gain from retirement of preference shares 46,000
g. Impairment loss on an equipment 400,000
h. Revaluation surplus on Land and Building 2,000,000
i. Net income per books 1,760,000

b. The understatement in the 2020 accrued salaries expense has already been adjusted
by the client per books as a debit to retained earnings and a credit to salaries
expense.
Page 9 of 20 0915-2303213  resacpareview@gmail.com
AUDITING
ReSA Batch 43 - May 2022 CPALE Batch
08 Feb 2022  6:00 PM to 9:00 PM AUD First Pre-Board Exam

c. The 15% share dividend declaration is based on 100,000 shares outstanding. Par
value of shares is at P100. The prevailing fair value of shares on the declaration
date was at P120.
d. The loss from the reissue of treasury shares arose from reissuance of 10,000
treasury at P130 per share. The treasury shares were originally reacquired at
P140 per share. Share premium from treasury share transactions had a balance of
P20,000. There were no more treasury shares by the end of the year.
e. The company has been employing the first-in-first-out inventory costing method
since it started its operations in 2019. In 2021 however, the management believes
that changing the cost formula to the average method will show a more relevant
and faithfully represented information. The change is yet to be reflected in the
company’s records. Furthermore, records revealed the following inventor balances
under the two methods:
Dec. 31, 2019 Dec. 31, 2020 Dec. 31, 2021
First-in-first-out 219,000 410,000 388,000
Average 250,000 444,000 425,000
Requirements:
52. What is the correct net income for 2021?
a. 1,279,000 c. 1,282,000
b. 1,316,000 d. 1,228,000
53. What is the retained earnings, beg as restated in 2021?
a. 5,284,000 c. 5,250,000
b. 5,324,000 d. 5,115,000
54. What is the correct retained earnings, end in 2021?
a. 4,726,000 c. 4,986,000
b. 4,652,000 d. 4,686,000

PROBLEM 6:
On December 31, 2018, Trail Blazer Co. issued share appreciation rights to 200 of its
employees. The rights will vest at the end of 3 years provided the employees remain
with the company and provided further that the average annual earnings growth rate is
at least 10% over the vesting period. The following are the approved terms of the
said share appreciation rights:
• If the average annual earnings growth rate is 10 to 20%, each employee will
receive 100 share appreciation rights.
• If the average annual earnings growth rate is 21 to 40%, each employee will
receive 150 share appreciation rights.
• If the average annual earnings growth rate is more than 40%, each employee will
receive 200 share appreciation rights.
On the grant date, each share appreciation right is determined to have a fair value of
P12. Trail Blazer Co. expects an average annual growth growth rate of 35% over the 3-
year vesting period.
The following information are available from the company’s records:
Fair Market
Actual Total Value of the
earnings Estimated share
Year growth rate resignations appreciation
for the year rights
2019 35% 20 P15
2020 40% 25 18
2021 48% 32* 24
*actual
Requirements:
55. Assuming that the estimates regarding the share appreciation rights remained
the same by the end of 2020, what is the salaries expense in 2020?
a. 135,000 c. 180,000
b. 315,000 d. 220,000
56. What is the salaries expense in 2021?
a. 315,000 c. 421,200
b. 398,200 d. 491,400

Page 10 of 20 0915-2303213  resacpareview@gmail.com


AUDITING
ReSA Batch 43 - May 2022 CPALE Batch
08 Feb 2022  6:00 PM to 9:00 PM AUD First Pre-Board Exam
PROBLEM 7:
You were assigned to audit the trade and other liability account balances of your audit
firm’s client Suns Corporation for the calendar year ended December 31, 2021. The
accountant of the client provided you the following lead schedule of its trade and
other current liability accounts:
Accounts payable – trade P918,600
Accrued operating expenses 124,300
Warranties payable 222,750
Audit notes:
a. Purchases cut-off results:
December Entries:
Receiving Receiving Invoice Particulars
Report Report Amount
No. Date
2519 Dec. 28 P12,900 From consignor
2521 Dec. 30 20,100 FOB Destination
2522 Jan. 2 14,500 FOB Destination (In-transit as of Dec. 31)
2523 Jan. 2 17,400 FOB Shipping Point (In-transit as of Dec. 31)
2524 Jan. 3 20,400 FOB Seller’s Warehouse
January Entries:
Receiving Receiving Invoice Particulars
Report Report Amount
No. Date
2525 Jan. 3 11,800 FOB Seller’s Warehouse (In-transit as of Dec. 31)
2526 Jan. 4 9,100 FOB Shipping Point (In transit as of Dec. 31)
2527 Jan. 5 24,500 FOB Destination
2528 Jan. 5 8,400 Bill and Hold Agreement executed in Dec.
Note a) Physical count of goods was rendered on all warehouse on December 30.
Thus all goods received on or before December 30 has been included in the physical
count which amounted to P870,700.
Note b) Receiving report number 2520 corresponds to goods received in December
29. The purchase invoice of the supplier is yet to be received by the client.
The cost of goods purchased was at P5,500.
b. The company sells its products with an accompanying 2-year assurance-type
warranty against manufacturer’s defects. The balance of the warranties payable
was the accrual made by the company in the prior year. Adjustment is yet to made
for the current year-end warranties payable. The following information were
deemed relevant for your analysis:
2020 2021
Unit Sales 11,000 15,000
Selling price of main product P900 P950
Actual warranty costs paid 74,250 205,200
Note a) The company estimates that 20% of the units sold shall be returned for
warranty in the year of sale and another 40% shall be returned for warranty in
the year following the year of sale.
Note b) Unit repairs costs is estimated at P45 per unit in parts and labor.
Requirements:
57. What is the adjusted balance of the accounts payable trade as of December 31?
a. 905,600 c. 897,200
b. 926,000 d. 900,100
58. What is the correct inventory balance as of December 31?
a. 901,600 c. 892,600
b. 910,000 d. 904,500
59. What is the correct warranties payable as of December 31, 2021?
a. 297,000 c. 422,550
b. 405,000 d. 199,800

60. Which of the following would an auditor most likely perform in auditing trade
payables of a merchandising audit client?
a. Send confirmation letters to a sample of suppliers who has significant
account balances outstanding as of the balance sheet date.

Page 11 of 20 0915-2303213  resacpareview@gmail.com


AUDITING
ReSA Batch 43 - May 2022 CPALE Batch
08 Feb 2022  6:00 PM to 9:00 PM AUD First Pre-Board Exam
b. Select a sample of suppliers’ invoices outstanding as of the balance
sheet date and vouch to the receiving report and purchase orders.
c. Review entries to the cash disbursement journals subsequent to the
balance sheet date and ascertain that payments to invoices
outstanding as of the balance sheet date are reported as accounts
payable as of the balance sheet date.
d. Select a sample of purchase orders and trace them to the receiving
reports.

PROBLEM 8:
The following information are deemed relevant in relation to your audit of the current
liabilities of Rockets Corp. as of December 31, 2021:
a. A reconciliation of the company accounts payable general ledger to its subsidiary
ledger appears below:
Balance per general ledger P459,000
Check issued to supplier on December 30, dated January 4 23,000
Check issued to supplier on December 30, dated December 30 (19,000)
Check issued to supplier on January 3, dated December 30 (15,000)
Invoice price of goods received on January 5, 20,000
Purchase in-transit as of Dec. 31, FOB shipping point 25,000
Purchase returns in December, credit memos received in January 5 (4,000)
Debit memos in suppliers account where there is no right of offset 10,000
Balance per subsidiary ledger P499,000

b. The company reported total appliance sales at P25M. The company sold 1,000 units
of its product in the current year. Each product sold is accompanied by a one-
year service-type warranty contract. The warranty service can be sold separately
at P2,500 per service contract. During the current year, the company spend
P800,000 in warranty service-related costs and is expected to incur P400,000
more to complete all service warranty contracts.
Requirements:
61. What is the adjusted balance of the accounts payable trade as of December 31?
a. 484,000 c. 463,000
b. 494,000 d. 514,000
62. What is the balance of unearned income from the service-type warranty as of
December 31?
a. 1,515,152 c. 833,333
b. 757,576 d. 1,666,667

PROBLEM 9:
Heat Corporation’s Salaries payable balance as of December 31, 2021 amounted to
1,502,179 and is composed of:
Accrued compensated absences 1,169,600
Accrued profit sharing bonus 332,579

a. The accrued compensated absences balance was the accrual made in the prior year.
No adjusting entry has been made by the end of the year to reflect the correct
accrual for the year. The following information are deemed relevant for your
analysis:
b.
Prior to 2019 leaves carried over 2021 120 days
2019 leaves carried over 2021 400
2020 leaves carried over 2021 1,200
Leaves exercised in 2021 1,380
Additional leaves earned in 2021 1,200
Note a: Leaves earned for the current year can be carried over up to three years,
thereafter the leaves shall expire.
Note b: There has been a 25% increase in salary rate in 2021.
Note c: From the leaves exercised in 2021, 100 days were earned prior to 2019.
c. The accrued profit sharing bonus per books was computed at 15% of the company’s
net income after bonus and after 30% income tax.

Page 12 of 20 0915-2303213  resacpareview@gmail.com


AUDITING
ReSA Batch 43 - May 2022 CPALE Batch
08 Feb 2022  6:00 PM to 9:00 PM AUD First Pre-Board Exam
Requirements:
63. What is the adjusted balance of the accrued compensated absences as of December
31?
a. 1,309,000 c. 1,033,600
b. 1,129,000 d. 1,292,000
64. What is the adjusted balance of the accrued profit-sharing bonus as of December
31?
a. 320,948 c. 344,209
b. 354,648 d. 344,567
65. What is the adjusted net income after tax?
a. 2,364,320 c. 2,139,656
b. 2,294,727 d. 2,297,113
66. Verification of the legitimacy or propriety of year-end unpaid bonuses to
officers and employees can be accomplished by comparing the recorded accrued to
the amount _________. This is necessary to gather evidence regarding _________
assertion on the year-end accrued bonuses.
a. In the expense account; Completeness
b. Use in the prior period; Existence
c. Authorized in the minutes of board meetings; Existence
d. Paid in the subsequent period; Completeness

PROBLEM 10:
You were assigned to audit the various non-trade liabilities of Bucks Inc. as of
December 31, 2021. The non-trade liabilities included the following:
10%, Convertible bonds payable 4,000,000
12%, Loans payable, Due December 31, 2025 2,000,000
Audit notes:
a. The 10% convertible bonds payable were issued in July 1, 2019 for (P4.4M
cash) when the prevailing market rate of interest for similar securities
without the conversion option was at 8%. The bonds which pay semi-annual
interest every June 30 and December 31 shall mature on June 30, 2022. Each
P1,000 bonds is convertible into 5, P100 par value ordinary shares.
The company recorded the issuance as a debit to cash at 4.4M, a credit to
bonds payable at face value 4M and a credit to additional paid-in capital
from bond conversion option at P400,000.
On December 31, 2021, 1M of the convertible bonds were converted. The same
is yet to be recorded by the client.
b. The company has an outstanding non-cancellable lease agreement with
Milwaukee Leasing Inc. for a building. The lease still has a 4-year
remaining term and that the company still has four P300,000 annual payment
at the end of each year starting next year. The company is no longer in
need of the property as it already has abandoned any operations where the
leased property is located. The company is not allowed to sub-lease the
facility and has a lease termination clause at P1,000,000. The prevailing
market rate of interest as of December 31, 2021 is at 10%. The lease
agreement has been deemed onerous by the management.

c. The 12% Loans payable was to ABC Banking Inc. The loan agreement included
a clause wherein Bucks Inc. should maintain a working capital ratio of 2:1
as at each balance sheet date the loan is outstanding. As of December 31,
2021 however, the company is in violation of this covenant as its working
capital ratio is only 1.5:1. On February 1, before the financial statements
were authorized for issuance, the bank granted Bucks Inc. a one-year grace
period to comply with the working capital agreement. Accordingly, the bank
will not be demanding payment on the loan during the one-year grace period.
67. What is the correct credit to share premium account as a result of the conversion
of bonds on December 31, 2021?
a. 2,228,776 c. 1,114,388
b. 1,671,582 d. 557,194

Page 13 of 20 0915-2303213  resacpareview@gmail.com


AUDITING
ReSA Batch 43 - May 2022 CPALE Batch
08 Feb 2022  6:00 PM to 9:00 PM AUD First Pre-Board Exam
68. What is the carrying value of the remaining bonds payable as of December 31,
2021?
a. 3,028,846 c. 3,056,583
b. 3,033,213 d. 3,064,255
69. What is the correct provision from onerous lease contract as of December 31,
2021?
a. 1,200,000 c. 1,000,000
b. 950,960 d. 900,000
70. How much from the loans payable to the bank should be reported as non-current
liability as of December 31, 2021?
a. 2,000,000 c. 1,000,000
b. 200,000 d. None

- END –

ANSWERS & SOLUTIONS/CLARIFICATIONS


1 A 26 A 51 C
2 B 27 D 52 C
3 B 28 A 53 A
4 A 29 C 54 D
5 B 30 C 55 C
6 D 31 D 56 D
7 B 32 C 57 A
8 C 33 B 58 D
9 D 34 B 59 C
10 D 35 C 60 C
11 B 36 A 61 B
12 C 37 C 62 B
13 B 38 D 63 D
14 A 39 A 64 A
15 A 40 C 65 C
16 A 41 A 66 C
17 C 42 D 67 D
18 C 43 B 68 A
19 B 44 C 69 B
20 A 45 D 70 D
21 D 46 A
22 B 47 D
23 C 48 A
24 D 49 D
25 C 50 B

1. A practitioner's report on agreed-upon procedures should contain the statement that the procedures performed
were those agreed to by the specified parties identified in the report.
4. The audit committee is considered a subgroup ordinarily charged with assisting the board of directors in
fulfilling its oversight responsibilities.
7. According to the IFAC Code of Ethics for Professional Accountant (and AICPA/PCAOB), audit teams are required
to be independent of the audit client during the engagement period and during the period covered by the
financial statements.
8. Audit risk consists of (1) the risks of material misstatement (inherent risk combined with control risk) and (2)
detection risk. The RMMs are the entity's risks, and detection risk is the auditor's risk. Detection risk is the risk
that the procedures performed by the auditor to reduce audit risk to an acceptably low level will not detect a
material misstatement. It is a function of the effectiveness of an audit procedure and its application by the
auditor. Detection risk is the only component of audit risk that can be changed at the auditor's discretion. An
auditor who performs procedures at an interim date should cover the remaining period. The longer the
remaining period, the greater the detection risk resulting from performing procedures at an interim date.
10. The President, not the PRC, has the power to remove from the Board any member whose certificate to practice
has been removed or suspended.
11. PRC Resolution No. 254 Series of 2017 Section 13 – The PIC and BOA accreditation shall be reconciled and
aligned so that CPAs applying for both credentials shall present the same CPD requirements for both applicants.
For individual professionals, the reckoning date of the validity of the PRC license and the BOA accreditation
shall be the date of birth of the individual.

Page 14 of 20 0915-2303213  resacpareview@gmail.com


AUDITING
ReSA Batch 43 - May 2022 CPALE Batch
08 Feb 2022  6:00 PM to 9:00 PM AUD First Pre-Board Exam
12.
GWA GWA No grades Majority of the Status
≥75%? below 65%? subjects ≥75%?
Examinee 1 95.00% Yes Yes N/A Passed
Examinee 2 75.00% Yes Yes N/A Passed
Examinee 3 75.17% Yes Yes N/A Passed
Examinee 4 79.67% Yes No Yes Conditional
Examinee 5 74.50% No Yes No Failed
Examinee 6 74.00% No Yes No Failed
Examinee 7 85.33% Yes No No Failed
Examinee 8 78.50% Yes No Yes Conditional
Examinee 9 73.00% No No Yes Conditional
Examinee 10 78.33% Yes Yes N/A Passed
13. Differences of opinion between the engagement partner and the quality control reviewer should be resolved
by following the firm's policies and procedures.
14. Based on SRC Rule No. 68 (Revised 2019). ISQC 1 / ISQM 1 / ISA 230 requirements is 5 years.
15. Choice “a” is correct. If the accountant agrees with the change, the accountant should issue the review report
without mentioning the change in engagement. Choice “b” is incorrect. The review report should not include a
disclaimer of an audit opinion. Choice “c” is incorrect. The review report should not report the circumstances
that resulted in the change in engagement. Choice “d” is incorrect. The review report should not include a
reference to the original engagement.
16. PSA 210.3. The objective of the auditor is to accept or continue an audit engagement only when the basis
upon which it is to be performed has been agreed, through:
(a) Establishing whether the preconditions for an audit are present; and
(b) Confirming that there is a common understanding between the auditor and management and, where
appropriate, those charged with governance of the terms of the audit engagement.
17. Turnover of the component's board of directors generally would not influence the decision to obtain an
additional engagement letter from the component.
18. A decrease in retained earnings would be an unreasonable expectation because the auditor noticed a significant
increase in net income. Net income closes to retained earnings; therefore, absent any other information, the
auditor would expect there would be an increase in retained earnings.
19. A decrease in the ratio of accounts payable to total current liabilities most likely indicates possible unrecorded
liabilities. In the prior year, there were six accounts payable for every current liability. In the current year,
there are four accounts payable for every current liability. The decrease in accounts payable per current liability
may indicate possible unrecorded payables.
20. An auditor engaging a lawyer to interpret the provisions of a complex contract would be considered to be using
the work of a specialist. PSA defines a expert/specialist as a person or firm with special skills in a field other
than accounting or auditing (e.g., actuaries, appraisers, attorneys, or engineers).
21. When materiality for the financial statements as a whole are revised, the materiality levels for particular classes
of transactions, account balances, or disclosures may also need to be revised.
22. Analytical procedures are evaluations of financial information made by a study of plausible relationships among
financial and nonfinancial data using models that range from simple to complex. The basic premise is that
plausible relationships among data may reasonably be expected to exist and continue in the absence of known
conditions to the contrary. Relationships in stable environments are more predictable than those in unstable
environments, and income statement amounts tend to be more predictable than balance sheet amounts. The
reason is that income statement amounts are based on transactions over a period of time, but balance sheet
amounts are for a moment in time. Also, amounts subject to management discretion tend to be less
predictable.
23. The auditor prepares the summary of unadjusted differences. Management's responses to inquiries cannot be
verified by the auditor by merely taking their statements and filling out a summary sheet of misstatements.
Misstatements recorded on the summary should be supported by additional evidence (e.g., inspection of
documents, observations). Inquiry alone is not sufficient.
24. An appropriate response to an identified business risk associated with plans for a new product line is for the
auditor to analyze the newly identified risk in conjunction with other known business risks and consider whether
there is an immediate consequence for the risk of material misstatement at various levels of the audit. Business
risks often affect risk of material misstatement at the financial statement level, which may affect risks of
material misstatement at the relevant assertion level. For example, an unsuccessful new product may affect
the risks of material misstatement related to the valuation of inventory.
25. The "top-down approach" used during an audit of internal control over financial reporting begins by
understanding the overall risks to internal control over financial reporting at the financial statement level.
26. Manual controls would most likely be more suitable than automated controls for large, unusual, or nonrecurring
transactions.
27. A walkthrough follows a transaction from its origination until it is reflected in the financial statements. This
description "most accurately" describes the process of a walkthrough.
28. Emphasizing ethical behavior through oral communication and by example best demonstrates management
integrity in the absence of a written code of conduct.
29. The classes of transactions in the issuer’s operations that are significant to the issuer’s financial statements are
typically assessed when the auditor is obtaining an understanding of the information and communication
component of internal control.
Page 15 of 20 0915-2303213  resacpareview@gmail.com
AUDITING
ReSA Batch 43 - May 2022 CPALE Batch
08 Feb 2022  6:00 PM to 9:00 PM AUD First Pre-Board Exam
30. Even a well-designed internal control system has its limitations. One example of a limitation of internal control
includes deliberate circumvention of controls by collusion of two or more people, such as when a purchasing
employee and an outside vendor participate in a kickback scheme. Other limitations of internal controls include
human error and management override of control.
31. When planning an engagement to examine the effectiveness of the entity’s internal control, a practitioner
would be least likely to consider the evaluation of the operating effectiveness of the controls. The evaluation
of the operating effectiveness of controls occurs after the planning stage.
32. Overall responses apply to the assessed RMMs at the financial statement level. The following are examples of
overall responses: (1) an emphasis on professional skepticism in evidence gathering and evaluation; (2)
increased supervision; (3) assignment of staff with greater experience or expertise; (4) greater unpredictability
in the choice of further audit procedures; and (5) changing the nature, timing, and extent of audit procedures,
such as modifying the nature of a procedure to obtain more persuasive evidence. At the relevant assertion
level, the response is to change the nature, timing, or extent of further audit procedures.
35. The independent auditor makes all of the judgments about the audit so really A, B, and D are solely the
responsibility of the CPA and not up for discussion. Any discussions are about making the audit as efficient as
possible. Thus, any work such as taking inventory or setting up schedules that will be done by the client or
involve the client needs to be coordinated with the auditors.

PROBLEM 1: SPURS CORP.


36. Collections from customers 3,556,000
Add: Accounts receivable, end 298,000
Sales discounts 92,000
Sales returns (excluding refunds: P149,000-P28,000) 121,000
Write-off of receivables 32,000
Total 4,099,000
Less: Accounts receivable, beg (215,000)
Advances from customers, end (53,000)
Recoveries of previously written-off accounts (19,000)
Gross Sales, Accrual Basis 3,812,000

37. Payments to suppliers 1,876,000


Add: Accounts payable, end 159,000
Purchase discount 74,000
Purchase returns (excluding refunds: P120,000-15,000) 105,000
Less: Accounts payable, beg (184,000)
Gross Purchases, Accrual Basis 2,030,000
Less: Purchase discount (74,000)
Purchas returns (total) (120,000)
Net Purchases 1,836,000
Add: Inventory, beg 154,000
Cost of goods available for sale 1,990,000
Less: Inventory, end (211,000)
Cost of Sales, Accrual Basis 1,779,000

38. Total cash payments for operating expenses 950,000


Add: Accrued expense, end 34,000
Prepaid expense, beg 19,000
Total 1,003,000
Less: Accrued expense, beg (24,000)
Prepaid expense, end (12,000)
Operating expenses, Accrual Basis 967,000
PROBLEM 2: LAKERS INC.
2019 NI 2020 NI 2021 NI 2021 RE, Beg 2021 Assets
Unadjusted balances 987,000 1,259,000 1,980,000
b. Accrued salaries expense, under 2019 (5,000) 5,000
Accrued salaries expense, under 2020 (8,000) 8,000 (8,000)
Accrued salaries expense, under 2021 (12,000)
Accrued utilities expense, under 2019 (14,000) 14,000
Accrued utilities expense, under 2021 (7,000)
Unused office supplies, under 2020 4,000 (4,000) 4,000
Unused office supplies, under 2021 9,000 9,000
Unearned rent income, under 2020 (6,000) 6,000 (6,000)
Unearned rent income, under 2021 (2,000)
Accrued royalty income, under 2019 3,000 (3,000)
Accrued royalty income, under 2021 1,000 1,000
c. Accounts payable/Purchases, under 2020 (25,000) 25,000 (25,000)
Accounts payable/Purchases, under 2021 (32,000)
d. Advances from customers, under 2019 (24,000) 24,000
Advances from customers, under 2021 (41,000)
Page 16 of 20 0915-2303213  resacpareview@gmail.com
AUDITING
ReSA Batch 43 - May 2022 CPALE Batch
08 Feb 2022  6:00 PM to 9:00 PM AUD First Pre-Board Exam
e. Equipment, under 2020 210,000 210,000 150,000
Depreciation, under 2020 (210,000/7years) (30,000) (30,000)
Depreciation, under 2021 (210,000/7years) (30,000)
Adjusted balances 947,000 1,444,000 1,901,000 145,000 160,000
39.Ans. A. 40. Ans. C. 41. Ans. A. 42. Ans. D.

PROBLEM 3: CELTICS CORP.


April 30 (Correct Entry)
Cash 1,800,000
Bonds payable (at FMV: 1M*104%) 1,040,000
Interest expense (1M*12%*4/12) 40,000
Ordinary shares (20,000*25) 500,000
Share premium from ordinary shares 220,000

43. April 30 (Adjusting Entry)


Share premium from ordinary shares 80,000
Bonds payable 40,000
Interest expense 40,000

June 1 (Correct Entry)


Equipment at FMV 520,000
Share premium from treasury shares 22,000
Retained earnings 28,000
Treasury shares (15,000*38) 570,000

44. June 1 (Adjusting Entry)


Share premium from treasury shares 22,000
Retained earnings 28,000
Equipment 50,000

September 1 (Correct Entry)


Ordinary shares (5,000*25) 125,000
Share premium from ordinary shares (2M/250K) 40,000
Retained earnings 25,000
Treasury shares (5,000*38) 190,000

45. September 1 (Adjusting Entry)


Share premium from ordinary shares (2M/250K) 40,000
Retained earnings 40,000

December 31 (Correct Entry)


Memo entry:
255,000 share rights were issued to ordinary shareholders.
Exercise rate is 4 share rights is to 1 ordinary share.
Exercise price is at P40.

46. Assume exercise of 80% of share rights in 2022:


Cash (255,000*80% = 204,000/4 = 51,000*P40) 2,040,000
Ordinary shares (51,000 shares*25) 1,275,000
Share premium from ordinary shares 765,000

47. Assume conversion of 30,000 preference shares to ordinary shares (conversion rate: 1:3)
Preference shares (30,000*P50) 1,500,000
Share premium from pref. shares (3,190,000/110,000) *30K 870,000
Ordinary shares (90,000*P25) 2,250,000
Share premium from ordinary shares 120,000

48. 2019 Year-End Analysis


Projected 2021 Sales based on Actual
2019 (2.5M*130%*130%) 4,225,000
Less: Actual 2019 (Base year) 2,000,000
Total projected increase in sales by 2021 2,225,000
Increase in sales in percentage 111% *Non-market base condition is achievable
Number of options = 200 per employee
Total FMV of services (91employees*200 382,20
= 18,200opt*P21)
Divide by: Vesting period 3
Salaries expense, 2019 127,400
Page 17 of 20 0915-2303213  resacpareview@gmail.com
AUDITING
ReSA Batch 43 - May 2022 CPALE Batch
08 Feb 2022  6:00 PM to 9:00 PM AUD First Pre-Board Exam
2020 Year-End Analysis
Projected 2021 Sales based on Actual
2020 (3.5M*130%) 4,550,000
Less: Actual 2019 (Base year) 2,000,000
Total projected increase in sales by 2021 2,550,000
Increase in sales in percentage 128% *Non-market base condition is achievable
Number of options = 300 per employee
Total FMV of services (90employees*300
= 27,000opt*P21) 567,000
Multiply by: 2 years/ 3 years 2/3
Cumulative salaries expense for 2 years 378,000
Less: Salaries expense in the prior year (127,400)
Salaries expense, 2020 250,600

49. 2021 Year-End Analysis


Actual 2021 Sales 5,100,000
Less: Actual 2019 (Base year) 2,000,000
Total increase in sales by 2021 3,100,000
Increase in sales in percentage 155% *Non-market base condition is achievable
Number of options = 400 per employee
Total FMV of services (85employees*400
= 34,000opt*P21) 714,000
Multiply by: 3 years/ 3 years 3/3
Cumulative salaries expense for 3 years 714,000
Less: Salaries expense in the prior years (378,000)
Salaries expense, 2021 336,000
50. Assuming 40% of the options granted were execised:
Cash (34,000*40% = 13,600/5 = 2,720shares*P120) 326,400
Ordinary share options outstanding (714,000*40%) 285,600
Ordinary shares (2,720shares*P100) 272,000
Share premium from ordinary shares 340,000
PROBLEM 5: JAZZ CORPORATION
52. Net Income, unadjusted 1,760,000
a. Loss on inventory write-off/down (135,000)
e. Gain on early retirement of bonds payable 54,000
g. Impairment loss on equipment (400,000)
e. Effect of the change in inventory costing (FIFO to Ave)
2020 Inventory, end under (444,000-410,000) (34,000)
2021 Inventory, end under (425,000-388,000) 37,000
Adjusted Net Income 1,282,000
54. Retained earnings, beg 5,290,000
c. Understatement in 2020 accrued salaries expense (40,000)
e. Effect of the change in inventory costing (FIFO to Ave)
2020 Inventory, end under (444,000-410,000) 34,000
Retained earnings, beg as restated 5,284,000
b. 15% share dividends (15%*100,000*P120) (1,800,000)
d. Loss on sale of Treasury Share after (APIC from TS) (80,000)
i. Adjusted Net Income 1,282,000
Retained earnings, end 4,686,000
PROBLEM 6: TRAIL BLAZER CO.
2019 Year-end analysis
Average earnings growth rate over the 35% *Services received
vesting period **Number of SARs per employee = 150
Estimated FMV of SARS (180*150=
27,000sars*P15) 405,000
Divide by: Vesting period 3
Salaries expense, 2019 135,000
55. 2020 Year-end analysis
Average earnings growth rate over the 35% *Services received
vesting period **Number of SARs per employee = 150
Estimated FMV of SARS (175*150=
26,250sars*P18) 472,500
Multiply by: 2 years/ 3 years 2/3
Cumulative salaries expense for 2 years 315,000
Less: Salaries expense in the prior year (135,000)
Salaries expense, 2020 180,000

Page 18 of 20 0915-2303213  resacpareview@gmail.com


AUDITING
ReSA Batch 43 - May 2022 CPALE Batch
08 Feb 2022  6:00 PM to 9:00 PM AUD First Pre-Board Exam
56. 2021 Year-end analysis
Actual average earnings growth rate 41% *Services received
(35+40+48)/3 = **Number of SARs per employee = 200
Estimated FMV of SARS (168*200=
33,600sars*P24) 806,400
Multiply by: 3 years/ 3 years 3/3
Cumulative salaries expense for 3 years 806,400
Less: Salaries expense in the prior years (315,000)
Salaries expense, 2021 491,400

PROBLEM 7: SUNS CORPORATION


Accounts Inventory
Payable Dec. 30,
Unadjusted balances 918,600 870,700
December entries:
RR 2519 Goods on consignment (12,900) (12,900)
RR 2520 Unrecorded December purchase 5,500
RR 2522 Purchase in-transit FOB Destination (14,500)
RR 2523 Purchase in-transit FOB Shipping Point 17,400
RR 2524 Not valid December purchase (20,400)
January entries:
RR 2525 Purchase in-transit FOB Seller's Warehouse 11,800 11,800
RR 2526 Purchase in-transit FOB Shipping Point 9,100 9,100
RR 2528 Bill and Hold Agreement in Dec. 8,400 8,400
Adjusted balances 905,600 904,500
57. Ans. A. 58. Ans. D.

59. Estimated warranty expense, 2020 (11,000*60%*45) 297,000


Less: Actual costs paid (74,250)
Warranties payable, end 2020 222,750
Estimated warranty expense, 2021 (15,000*60%*45) 405,000
Less: Actual costs paid (205,200)
Warranties payable, end 2021 422,550

PROBLEM 8: ROCKETS CORP.


61. General Ledger Subsidiary Ledger
Balance per general ledger 459,000 499,000
Check issued to supplier on December 30, dated 23,000
January 4
Check issued to supplier on December 30, dated (19,000)
December 30
Check issued to supplier on January 3, dated 15,000
December 30
Invoice price of goods received on January 5, (20,000)
Purchase in-transit as of Dec. 31, FOB shipping point 25,000
Purchase returns in December, credit memos received (4,000)
in January 5
Debit memos in suppliers account where there is no 10,000
right of offset
Balance per subsidiary ledger 494,000 494,000

62. Total Sales 25,000,000


Allocation:
Sale of goods (25,000/27,500) 22,727,273
Warranty service (2,500/27,500) 2,272,727

Warranty service 2,272,727


Earned revenue (800,000/1,200,000) 1,515,152
Unearned revenue (400,000/1,200,000) 757,576

PROBLEM 9: HEAT CORPORATION


63. Accrued compensated absences, Dec. 31, 2020 1,169,600
Accumulated unused leaves, Dec. 31, 2020
(120 days + 400 + 1,200) 1,720
Salary rate in 2020 680

Accumulated unused leaves, Dec. 31, 2020 1,720


Less: Leaves used/exercised in 2021 (1,380)
Balance 340
Page 19 of 20 0915-2303213  resacpareview@gmail.com
AUDITING
ReSA Batch 43 - May 2022 CPALE Batch
08 Feb 2022  6:00 PM to 9:00 PM AUD First Pre-Board Exam
Leaves earned in 2021 1,200
Accumulated unused leaves 1,540
Less: Expired leaves (From prior to 2019: 120 - 100) (20)
Accumulated unused leaves, balance 1,520
Multiply by: 2021 current salary rate (680*120%) 850
Accrued compensated absences, Dec. 31, 2021 1,292,000

64. B = 15%(NI-B-Tx)
Tx = 30%(NI-B)
B = 15%(NI-B-30%(NI-B))
Bonus per books = 332,579

332,579 = 15% (NI - 332,579 - 30%(NI-332,579)


367,500 = 0.105NI
NI = 3,500,000

NI, unadjusted 3,500,000


Adjustment: Additional salaries expense from accrued compensated abs. (122,400)
NI, adjusted 3,377,600

B = 15% (3,377,600 - B - 30%(3,377,600-B))


1.105B = 354,648
B = 320,948

65. NI before tax and bonus 3,377,600


Less: bonus (320,948)
NI before tax 3,056,652
Income tax expense (3,056,652*30%) (916,996)
Net Income after Tax 2,139,656

PROBLEM 10: BUCKS INC.

66. July 1, 2019 (Correct Entry)


Cash 4,400,000
Bonds payable (at FMV) 4,209,685
APIC - bond conversion option 190,315

FMV of bonds at 4% semi-annual effective interest


Principal: 4,000,000*0.79031 3,161,258 0.79031
Interest: 200,000*4.19371 1,048,427 5.24214
Total FMV 4,209,685

Amortization Table Nominal Int. Effective Int. Amortization Balance


July 1, 2019 Initial Cost 4,209,685
Dec. 31, 2019 Amo 200,000 168,387 31,613 4,178,073
June 30, 2020 Amo 200,000 167,123 32,877 4,145,196
Dec. 31, 2020 Amo 200,000 165,808 34,192 4,111,004
June 30, 2021 Amo 200,000 164,440 35,560 4,075,444
Dec. 31, 2021 Amo 200,000 163,018 36,982 4,038,462
June 30, 2022 Amo 200,000 161,538 38,462 4,000,000

67. Entry upon conversion on December 31, 2021:


Bonds payable (4,038,462*1/4) 1,009,615
APIC-Bond Conversion Option 47,579
Ordinary shares (5,000shares*100) 500,000
Share premium 557,194

68. 3,056,583
Balance of remaining bonds (4,038,462*3/4) 3,028,846

69. Present Value of Noncancellable lease payments


300,000*3.169865 950,960 Lower 0.683013 3.1698654

Lease termination fee 1,000,000

70. While the grace period is for one-year, it was granted by the bank after the balance sheet date.

Page 20 of 20 0915-2303213  resacpareview@gmail.com


ReSA - THE REVIEW SCHOOL OF ACCOUNTANCY
CPA Review Batch 43  May 2022 CPALE  11 Feb 2022  6:00 PM – 9:00 PM

FINANCIAL ACCOUNTING & REPORTING FIRST PRE-BOARD EXAMINATION

INSTRUCTIONS: Select the correct answer for each of the questions. Mark only one
answer for each item by shading the box corresponding to the letter of your choice on
the answer sheet provided. STRICTLY NO ERASURES ALLOWED. Use pencil no. 2 only.

1. On December 31, 2020, the cash account of Gambit Company shows the following
composition:
Petty cash fund, P180,000; Cash in bank (payroll fund), P2,000,000; Interest
and dividend fund, P250,000; Tax fund, P120,000; Cash in bank (current
account), P3,000,000; Certificate of deposit (terms 90 days), P1,000,000;
Certificate of deposit (terms 180 days), P1,500,000; Cash in foreign bank-
restricted, P500,000; Money market fund, (60 days), P500,000; Money market
funds (6 months), P900,000; Customer’s check dated February 15, 2021, P60,000;
Customer’s check dated December 30, 2020 returned for lack of funds, P40,000;
A 30-day BSP treasury bill, P1,000,000; A 3-year BSP treasury bill acquired
three months prior to maturity, P1,200,000; Sinking fund cash,
P800,000;Contingent fund, P900,000 Fund for the acquisition of fixed asset,
P500,000; Travelers’ checks, P60,000; and Cashiers’ checks, P100,000.
What is the correct cash and cash equivalents balance to be reported by Gambit
Company on December 31, 2020?
D a. P 7,810,000
b. P 8,210,000
c. P 8,910,000
d. P 9,410,000
2. The balance sheet of Alaska Company as of December 31, 2021 shows Cash of P17,500.
It was found to include some of the following items:
Postal money orders from customers P 2,400
Notes receivable in the possession of a collection agency 3,200
Receipts for expense advances for the account of credit suppliers 600
Customers’ postdated checks, returned by the bank marked “NSF” 1,800
Traveler’s check 500
Currencies and coins on hand 600
Checks in payment of accounts not yet delivered to payee 6,000
PCF (P160 in currency and P840 in expense receipts) 1,000
What is the correct cash balance?
B a. P 9,660
b. P 11,060
c. P 12,860
d. P 14,260
3. The Petty Cash Fund of GDM Company has an Imprest balance of P10,000. The
following items are found in its drawer on December 31, 2021:
Currencies and coins
10 pcs. @ P 100 ?
8 pcs. @ P 50 ?
30 pcs. @ P20 ?
Unreplenished paid vouchers for expenses and loans dated Dec. 16-31, 2021:
Supplies 1,200
Loan to office director 1,800
Unused supplies 200
Check drawn by office manager dated 12/30/21 2,500
Check drawn by employee dated 1/18/22 1,500
Check drawn by customer dated 12/25/21 1,000
Envelope containing cash donations for the departed
parent of one employee, (no currencies attached) 500
How much is the correct amount of petty cash fund on Dec. 31, 2021?
A a. P 4,000
b. P 2,000
c. P 1,500
d. P 4,500
4. What is the amount of shortage or overage?
C a. P 1,000 short
b. P 1,000 over
c. P 1,500 short
d. P 1,500 over

Page 1 of 14 0915-2303213  resacpareview@gmail.com


FINANCIAL ACCOUNTING & REPORTING
ReSA Batch 43 - May 2022 CPALE Batch
11 Feb 2022  6:00 PM to 9:00 PM FAR First Pre-Board Exam
5. Yoda Corporation’s petty cash fund on December 31, 2020 is composed of the
following:
Coins and Currencies P7,000
Petty Cash Vouchers 13,000
Employee’s checks dated December 25, 2020 8,000
Employee’s check returned by the bank 3,000
Check drawn by the company payable to the petty cash custodian 10,000
Emptied Envelope labeled as contributions of employees for
wedding gift to an officer amounting to 4,000
Employee’s Checks 3,000
Employee’s Postdated Checks 2,000

The petty cash ledger has an Imprest balance of P50,000.


Compute for the amount of the shortage.
A a. P 8,000
b. P 5,000
c. P 30,000
d. P 10,000
6. Fino Company banks with ABC Bank and prepares reconciliation of the bank and
books balances on a regular monthly basis. The December 31, 2021 reconciliation
shows a balance per bank of P581,050, balance per books of P627,000, outstanding
checks of P84,300, deposits in transit of P120,000, interest earned on the bank
balance of P1,250, and service charges of P400. Included in the bank statement
was a cancelled check which the company had failed to record. The check was in
payment of accounts payable.
What is the amount of the unrecorded check issued by the company in payment of
accounts payable?
C a. P 8,600
b. P 11,000
c. P 11,100
d. P 11,900
7. Helium Company is preparing its July 31 bank reconciliation. The ff. data are
available:

July Data Per Bank Per Book


Balance, June 30 P741,400 P719,400
July deposits reflected 476,000* 490,000
July checks reflected (617,000)** (610,000)
Note collected (including P2,000 interest) 202,000
Service charge (1,200)
Balance, July 31 P801,200 P599,400
From the June 30 bank reconciliation:
Deposits in transit, P17,000 Outstanding checks, P39,000
*Erroneously includes a deposit of P25,000 by Hello Corporation.
**Erroneously includes a check drawn by Haller Company for P15,000.
What is the amount of outstanding checks at July 31?
A a. P 47,000
b. P 15,000
c. P 8,000
d. P 7,000
8. What is the amount of deposit in transit at July 31?
D a. P 31,000
b. P 3,000
c. P258,000
d. P 56,000
9. On August 31, 2021, Carbon Company sold goods to C Company. C company signed a
non-interest-bearing note requiring payment of P80,000 annually for five years.
The first payment was made on August 31, 2021. The prevailing rate of interest
for this type of note at the date of issuance was 12%. Information on present
value factors is as follows:

Page 2 of 14 0915-2303213  resacpareview@gmail.com


FINANCIAL ACCOUNTING & REPORTING
ReSA Batch 43 - May 2022 CPALE Batch
11 Feb 2022  6:00 PM to 9:00 PM FAR First Pre-Board Exam

Present value PV of Ordinary


Periods of 1 at 12% annuity of 1 at 12%
4 0.636 3.037
5 0.567 3.605
How much should be reported as sales revenue on August 31, 2021?
B a. P 242,960
b. P 322,960
c. P 288,400
d. P 368,400
10. What is the amount of interest income for the year 2021?
A a. P 9,719
b. P 12,918
c. P 11,536
d. P 14,736
11. Lithium Company uses the net price method of accounting for cash discounts. In
one of its transactions on December 21, 2021, Lithium Company sold merchandise
with a list price of P4,000,000 to a client who was given a trade discount of
20% and 10%. Credit terms given by Lithium Company were 5/10, 3/15, n/30. The
goods were shipped FOB shipping point, freight collect. Total freight charge
paid was P100,000. On January 4, 2022, the client paid his account.
What is the adjusted balance of accounts receivable on Dec. 31, 2021?
C a. P 2,693,600
b. P 2,736,000
c. P 2,793,600
d. P 2,880,000
12. The balances of the selected accounts taken from the December 31, 2019 balance
sheet of TIWALALANG Company are as follows:
Accounts Receivable- P 674,000; Allowance for bad debts- P 24,000
The following transactions (in summary) affecting accounts receivable occurred
during the year ended December 31, 2020:
Sales (all on account terms: 3/10,1/20, n/60) P3,000,000
Cash received from customers 3,200,000
Cash received includes the following:
Customers paying within the 10-day discount period 1,746,000
Customers paying within the 20-day discount period 990,000
Recovery of accounts previously written-off 6,000
Customers paying beyond the discount period ?
Accounts receivable written-off as worthless 22,000
Credit memoranda issued to credit customers for
sales returns and allowances 12,000

An aging of accounts receivable and estimate of uncollectible accounts at


December 31, 2020 revealed the following:
Age Amount Probability of non-collection
Less than 30 days P 150,000 2%
31-90 days 120,000 8%
91-120 days 86,000 15%
More than 120 days ? 30%
How much is the Accounts Receivable ledger balance on December 31, 2020?
A a. P 382,000
b. P 344,500
c. P 348,700
d. P 376,000
13. How much is the Allowance for bad debts on December 31, 2020?
B a. P 25,300
b. P 33,300
c. P 31,500
d. P 23,500

Page 3 of 14 0915-2303213  resacpareview@gmail.com


FINANCIAL ACCOUNTING & REPORTING
ReSA Batch 43 - May 2022 CPALE Batch
11 Feb 2022  6:00 PM to 9:00 PM FAR First Pre-Board Exam
14. On February 1, 2021, MAGI Corporation factored receivables without recourse with
a face amount of P600,000 to NGAT Corporation. MAGI Corporation advances P490,000
to NGAT Corporation. NGAT Corporation’s holdback rate is 5% of the factored
receivables.
What amount of loss should MAGI Corporation recognize as a result of factoring
of receivables?
C a. P 0
b. P 140,000
c. P 80,000
d. P 110,000
15. On October 31, 2020, PARASAPANGARAP Company discounted at the bank a customer’s
P600,000 interest-bearing note, 6-month, 12% note, dated July 1, 2020 with
recourse. The bank discounted the note at 10%.
How much is the gain or loss should the company recognize as a result of
discounting the notes receivable?
D a. P 10,600
b. P 1,400
c. P 24,000
d. P 0
16. On January 1, 2021, Allyssa Company acquired 30% of the voting share capital of
Ruby Company for P5,000,000 which was equal to the book value of interest
acquired. The investee reported net profit of P4,000,000 for 2021 and P6,000,000
for 2022 but paid no dividends during the two-year period. On July 1, 2022,
Alyssa sold half of the investment for P4,500,000. The fair value of the
remaining investment was P4,800,000 on July 1, 2022 and P5,500,000 on December
31, 2022. The remaining investment is to be held at fair value through other
comprehensive income. What amount of gain or loss on reclassification shall be
recognized on July 1, 2022?
B a. P 1,950,000
b. P 1,250,000
c. P 950,000
d. P 700,000
17. October 1, 2021, Michael Company purchased 30,000 shares of Scottie Company at
P180 per share which reflected book value as of that date. At the time of the
purchase, Scottie had 100,000 ordinary shares outstanding. Michael had no
ownership interest in Scottie before the purchase. The nine months ending
September 30, 2021 of Scottie recorded profit of P2,960,000. For the year ended
December 31, 2021, Scottie reported profit of P4,800,000. Scottie paid Michael
dividends of P120,000 on December 31, 2021.

For the year 2022, Scottie reported profit of P2,800,000 and paid dividends of
P1,700,000 to its ordinary shareholders.
On January 2, 2023, Michael sold 20,000 ordinary shares of Scottie for P250 per
share. For year ended December 31, 2023, the reported profit of Scottie was
P4,000,000 and dividends of P40,000 was paid to Michael. Market value of the
remaining shares at this time is P2,300,000. What is the investment carrying
value at December 31, 2022?
A a. P 6,162,000
b. P 5,970,000
c. P 5,832,000
d. P 5,400,000
18. What is the gain (loss) on the sale of 20,000 shares at January 2, 2023?
D a. P 1,400,000
b. P 1,020,000
c. P 1,000,000
d. P 892,000
19. On April 30, 2022, Rabiya Company purchased for cash 18,000 shares of the 60,000
voting shares of Catriona Company for P650,000. This amount exceeded the
underlying equity acquired in the net assets of Catriona Company by P150,000.
The excess is attributable to undervaluation of Catriona’s land and equipment by
P250,000 and P100,000 respectively. At April 30, 2022, the equipment had a
remaining useful life of 5 years. The remaining excess was attributable to
goodwill. During the year 2022, Catriona reported profit of P600,000, of which

Page 4 of 14 0915-2303213  resacpareview@gmail.com


FINANCIAL ACCOUNTING & REPORTING
ReSA Batch 43 - May 2022 CPALE Batch
11 Feb 2022  6:00 PM to 9:00 PM FAR First Pre-Board Exam
P120,000 was earned during January through April. Catriona declared and
distributed a dividend of P4 per share on June 30, 2022. Market price of Catriona
shares at December 31, 2022 is P40 per share. How much goodwill is included in
the carrying amount of investment?
D a. P 150,000
b. P 105,000
c. P 71,250
d. P 45,000
20. Based on same data in no. 19, what is the carrying amount of the investment in
associates at December 31, 2022?
B a. P 708,500
b. P 718,000
c. P 716,000
d. P 720,000
21. KAYAK Company invested 50,000 ordinary shares of the total 200,000 outstanding
ordinary shares and 30,000 preference shares of the total 100,000 outstanding
preference shares with par value of P10 from OTO Corporation on January 1, 2019.
The shares were acquired at P140 per share and P100 per share for ordinary and
preference, respectively.

The following are relevant information of OTO Corporation for the years 2019 and
2020:
2019 2020
Net income for the year P4,000,000 P5,000,000
OTO’s shares outstanding at December 31, 2019 follow:
Ordinary share P4,000,000
5% cumulative preference share ?
During 2019, OTO declared and distributed preference dividends to its preference
shareholders. KAYAK received cash dividend of P10 per ordinary share. OTO did
not declare any dividends during 2020. The share capital of OTO did not change
from 2019 up to 2020. How much is the share in net income to be reported in
profit or loss for the year 2020?
B a. P 1,002,500
b. P 1,237,500
c. P 1,000,000
d. P 1,250,000
22. During 2020, Mark Company acquired 11,000 ordinary shares of Spencer Company’s
200,000 shares that are widely distributed. The shares are not intended to be
traded in the near term and Mark Company does not have the ability to exercise
significant influence over the operating and financial policies of Spencer
Company. The market value of these shares had been changing for the last three
years as follows: December 31, 2020 – P235,000; December 31, 2021 – P241,000;
and December 31, 2022 – P222,000.
These shares were acquired in 2020 for P231,000 plus broker’s commission of
P2,310. How much will be the amount of Equity investments to be reported by Mark
Company in its December 31, 2022 statement of financial position?
B a. P 233,310
b. P 222,000
c. P 241,000
d. P 235,000
23. The Katipunera Corporation bought the shares of Luna Company classified as equity
investments at fair value through other comprehensive income, as follows:
April 17, 2022 1,000 shares at P84
July 16, 2022 2,000 shares at P90
Market value per share of Luna Company shares at December 31, 2022 was P92.
The following were the transactions for 2023:
January 10 Received cash dividend at P4 per share.
June 20 Received 5% bonus issue.
December 10 Sold 1,200 shares at P105 per share.

Page 5 of 14 0915-2303213  resacpareview@gmail.com


FINANCIAL ACCOUNTING & REPORTING
ReSA Batch 43 - May 2022 CPALE Batch
11 Feb 2022  6:00 PM to 9:00 PM FAR First Pre-Board Exam
What amount of dividend revenue should Katipunera Company report for 2023?
B a. P 0
b. P 12,000
c. P 13,800
d. P 25,800
24. Bulaga Company acquired a building on April 1, 2022 for P 18,000,000. The building
is being leased out under operating lease wherein the lessee pays rent on a
quarterly basis amounting to P 30,000. At that date, the building had an estimated
useful life of 30 years and depreciated using the straight-line method. On
December 31, 2017, the fair value of the building was P 19,200,000.
How much is the total net increase/decrease in profit for the year 2022 assuming
the company is using cost model?
D a. P 1,290,000
b. P 1,200,000
c. P 840,000
d. P 360,000
25. Information pertaining to the inventory of Boron Company for the year ended
December 31, 2022 follows:
Beginning inventory P 200,000* Ending Inventory P 300,000**
Total Purchases 830,000 Purchase returns 35,000
Freight-in 10,000 Purchase discounts 5,000
*Includes items A and B costing P20,000 and P30,000, respectively. The net
realizable value of item A is P15,000 and item B is P27,000.
** Includes cost of items A and B still unsold at the end of the year. The net
realizable value of item A is P23,000 and item B is P29,000.
How much is the cost of goods sold under the direct method of accounting for
write-down and reversal of write-down of inventories?
D a. P 700,000
b. P 698,000
c. P 685,000
d. P 693,000
26. How much is the cost of goods sold under allowance method if the policy is to
include any write-down and reversal of inventories as part of cost of goods sold?
D a. P 700,000
b. P 698,000
c. P 685,000
d. P 693,000
27. On January 2, 2022, Chlorine Company decided to convert one of its building into
an investment property that is to be carried at fair value. The building was
previously used as an owner-occupied property. The building was acquired on
January 1, 2020 costing P10,000,000 with an estimated useful life of 8 years
using the straight-line method. The cost of converting the building is
insignificant but as a result of the change in the usage, the fair market value
of the building was reliably valued at P8,000,000.
What amount should be taken directly to equity on the date of transfer?
B a. P 0
b. P 500,000
c. P 750,000
d. P 2,000,000
28. On January 1, 2022 Belgium Corp. acquired 35% of the total 500,000 outstanding
ordinary shares of Brussels Company for P3,500,000. Brussels reported during
2022 a total net income of P4,000,000 and actuarial loss from defined benefit
plan from of P200,000. Belgium received 140,000 ordinary shares during 2022 as
a result of bonus issue distributed by Brussels. The fair value of share at that
time is P15 per share. Brussels distributed total cash dividends at year end of
P1,000,000.
What is the carrying value of Investment in Brussels on December 31, 2022?
A a. P 4,480,000
b. P 4,725,000
c. P 6,580,000
d. P 4,840,000

Page 6 of 14 0915-2303213  resacpareview@gmail.com


FINANCIAL ACCOUNTING & REPORTING
ReSA Batch 43 - May 2022 CPALE Batch
11 Feb 2022  6:00 PM to 9:00 PM FAR First Pre-Board Exam
29. On January 1, 2022, Niger Company classifies a hotel property a non-current asset
held for sale. Immediately before the classification as held for sale, the cost
of the property is P100,000 and accumulated depreciation of P 40,000. The hotel
is depreciated on the straight-line method with a useful life of 10 years. The
estimate of the fair value less cost to sell on this date is P62,000.
Which of the following is true on the date of reclassification?
D a. Impairment loss is recognized amounting to P2,000
b. Realized gain is recognized amounting to P2,000
c. Unrealized gain is recognized amounting to P2,000
d. No amount of gain or loss is recognized
30. On December 30, 2022, Thanos Company classifies a non-current asset as held for
sale. Immediately before the classification as held for sale, the cost of the
property is P500,000 and accumulated depreciation of P200,000. The hotel is
depreciated on the straight-line method with a useful life of 20 years. The
estimated fair value less cost to sell on this date is P270,000. On December 31,
2023, the fair value less cost to sell is P 310,000.
How much is the gain that should be recognized on December 31, 2023?
B a. P 62,500
b. P 30,000
c. P 40,000
d. P 27,500
31. The following information pertains to the living plant and agricultural produce
of Iron Company. On January 1, 2020, the cost of the living plant was P20,000,000
with an estimated useful life of 10 years. The company is using the straight-
line method of depreciation. As of December 31, 2020, Iron Company determines
the following:

Fair value of the fruits before the harvest on Dec. 31, 2020 P5,000,000
Estimated cost to sell of the fruit 100,000
Estimated cost to sell of the living plant 500,000

With the assistance of valuation experts, Iron Company determines that the fair
value of the living plant including the fruit as of December 31, 2020 is
P26,000,000.

How much is the carrying value of the living plant on December 31, 2020 under
PAS 16?
A a. P 18,000,000
b. P 20,000,000
c. P 20,500,000
d. P 25,400,000
32. How much is the carrying value of the living plant on December 31, 2020 under
PAS 41?
D a. P 18,000,000
b. P 20,000,000
c. P 20,500,000
d. P 25,400,000
33. Columbia Company acquired a building on January 1, 2021 for P18,000,000. At that
date, the building had a remaining useful life of 30 years. At December 31, 2021,
the fair value of the building was P19,200,000. The building was classified as
investment property and accounted for under the cost model.
What amount should be reported for this asset in the statement of financial
position on December 31, 2021?
D a. P 19,200,000
b. P 18,560,000
c. P 18,000,000
d. P 17,400,000
34. On January 1, 2017, American Company acquired an equipment worth P2,060,000 for
its operations. The equipment has an estimated useful life of 10 years and an
estimated salvage value of P60,000. It’s the company’s policy to depreciate all
equipment using the straight-line basis. On January 1, 2021, American Company

Page 7 of 14 0915-2303213  resacpareview@gmail.com


FINANCIAL ACCOUNTING & REPORTING
ReSA Batch 43 - May 2022 CPALE Batch
11 Feb 2022  6:00 PM to 9:00 PM FAR First Pre-Board Exam
made a review of the estimated salvage value and determined that the estimated
salvage value of the equipment will be twice the original estimate.
What amount of depreciation expense should the company recognize in 2021?
C a. P 200,000
b. P 194,000
c. P 190,000
d. P 183,333
35. In January 2021, New Zealand Company purchased a mineral mine for P3,600,000
with removal ore estimates by geographical surveys at 2,160,000 tons. The
property has an estimated value of P360,000 after the ore has been extracted.
Newton Company incurred P1,080,000 of development costs preparing the property
for extraction of ore. During 2021, 270,000 tons were extracted and 240,000
tons were sold.
For the year ended December 31, 2021, New Zealand should include what amount of
depletion?
D a. P 360,000
b. P 480,000
c. P 520,000
d. P 540,000
36. For the year ended December 31, 2021, New Zealand’s cost of goods sold is
B a. P 360,000
b. P 480,000
c. P 520,000
d. P 540,000
37. On April 30, the company purchased and installed several furniture and fixtures
items from a local furniture manufacturer and dealer under the terms 5/20, n/30.
The transaction was reflected in the company books as:
4/30 Furniture and fixtures 2,500,000
Accounts payable 2,500,000
5/10 Accounts payable 2,500,000
Cash 2,500,000
In addition, the company incurred freight and installation costs amounting to
P10,000 and P15,000, respectively, which were charged to expense.
How much should the furniture and fixtures be initially recognized?
B a. P 2,375,000
b. P 2,400,000
c. P 2,500,000
d. P 2,525,000
38. In June 2021, Canada Company acquired a machine in exchange for a non-monetary
asset with a cost of P1,200,000 and an accumulated depreciation of P 600,000 and
paid cash difference of P160,000. The market value of the non-monetary asset
was determined to be P 650,000.
If the exchange is lacking in commercial substance, what would be the cost of
the new asset acquired and the amount of the gain to be recognized, respectively?
A a. P 760,000 & P 0
b. P 810,000 & P 0
c. P 810,000 & P 50,000
d. P 810,000 & P 210,000
39. Cathy Company acquired P2,000,000 bonds on May 1, 2021 and its accountant
correctly prepared the following entry.
Investment at Amortized Cost 2,294,416
Cash 2,294,416
These bonds pay interest at a rate of 8% per annum every April 30 and will mature
after 10 years. Market rate of interest for the same bonds was 6%.
How much is the Premium amortization for the year 2021?
D a. P 63,553
b. P 42,369
c. P 22,335
d. P 14,890

Page 8 of 14 0915-2303213  resacpareview@gmail.com


FINANCIAL ACCOUNTING & REPORTING
ReSA Batch 43 - May 2022 CPALE Batch
11 Feb 2022  6:00 PM to 9:00 PM FAR First Pre-Board Exam
40. Connie Company carried out the following transactions in bond investments held
for trading during the current year:
8/1 Purchased 5,000, P1,000, 12% bonds of AAA Company at 104 plus
accrued interest. The bonds pay interest semiannually on May 1 and
November 1
8/31 Purchased 2,000, P1,000 12% bonds of BBB Company at 98 plus accrued
interest. Semiannual payment of interest is on June 30 and December
31.
12/1 Sold 2,000 of the AAA bonds at 102 plus accrued interest. Brokerage
fee of P160,000 was incurred.
12/31 AAA bonds were selling at 98. BBB bonds were selling at 99.
How much is the Gain or Loss on sale of AAA bonds?
C a. P 40,000 loss
b. P 40,000 gain
c. P 200,000 loss
d. P 200,000 gain
41. How much is the total interest income for the year?
B a. P 500,000
b. P 310,000
c. P 270,000
d. P 230,000
42. How much is the unrealized gain (loss) to be reported in the profit or loss
section of statement of comprehensive income for the year?
C a. P 160,000 UG
b. P 300,000 UG
c. P 160,000 UL
d. P 280,000 UL
43. On January 1, 2017, Alvin Corporation purchased 3-year, 10%, 5,000 of P1,000
face value bonds for P4,600,000. In relation to this acquisition, Alvin incurred
P160,000 broker’s commission. Alvin intended to collect contractual cash flows
and to sell the financial asset. The effective rate is 12%.
On June 30, 2019, Alvin sold the bonds at 110 plus interest.
Meanwhile, Alvin determined the following fair values at each yearend:
December 31, 2017 102
December 31, 2018 105
December 31, 2019 104
How much is the unrealized gain/loss to be reported in the equity section of the
statement of financial position on December 31, 2018?
C a. P 0
b. P 70,256
c. P 339,056
d. P 221,200
44. How much is the unrealized gain to be reported as component of other comprehensive
income in the 2018 statement of comprehensive income?
B a. P 0
b. P 70,256
c. P 339,056
d. P 221,200
45. How much is the Gain on sale of the bond on June 30, 2019?
B a. P 250,000
b. P 544,528
c. P 589,056
d. P 794,528
46. A physical count on December 31, 2020 revealed that EARTH Company had inventory
with a cost of P4,400,000. The following items were excluded from this amount:
- Merchandise of P600,000 is held on consignment by EARTH.

- Goods costing P400,000 was shipped by EARTH “Ex-ship” to a customer on December


31, 2020. The customer received the goods on January 3, 2021.

Page 9 of 14 0915-2303213  resacpareview@gmail.com


FINANCIAL ACCOUNTING & REPORTING
ReSA Batch 43 - May 2022 CPALE Batch
11 Feb 2022  6:00 PM to 9:00 PM FAR First Pre-Board Exam

- Goods costing P800,000 shipped by a vendor FOB destination on December 31,


2020 was received by EARTH on January 7, 2021.
- Goods costing P700,000 was shipped by a supplier “CIF” on December 30, 2020
and received by EARTH on January 10, 2021.
What is the correct amount of inventory on December 31, 2020?
C a. P 4,900,000
b. P 5,400,000
c. P 5,500,000
d. P 6,000,000
47. Transactions for the month of June were:
PURCHASES SALES
June 1 (balance) 800 @ P3.20 June 2 600 @ P 5.50
June 3 2,200 @ P3.10 June 6 1,600 @ P 5.50
June 7 1,200 @ P3.30 June 9 1,000 @ P 5.50
June 15 1,800 @ P3.40 June 10 400 @ P 6.00
June 22 500 @ P3.50 June 18 1,400 @ P 6.00
June 25 200 @ P 6.00
Assuming that perpetual inventory records are kept in peso, the ending inventory
on a FIFO basis is
D a. P 4,110
b. P 4,160
c. P 4,290
d. P 4,470
48. Assuming that the perpetual inventory records are kept in units only, the ending
inventory on an average-cost basis, rounded to the nearest peso is
B a. P 4,096
b. P 4,238
c. P 4,290
d. P 4,322
49. The records of Morning Company show the following for the current year:

Cost Retail
Beginning inventory 340,000 640,000
Purchases 4,500,000 7,300,000
Freight in 100,000
Purchase return 150,000 250,000
Purchase allowance 90,000
Departmental transfer in 100,000 160,000
Net markup 150,000
Net markdown 500,000
Sales 6,600,000
Sales allowance 50,000
Sales returns 150,000
Employee discount 100,000
Spoilage and breakage 200,000
What is the amount of estimated cost of ending inventory under the average cost
retail?
C a. P 512,000
b. P 450,000
c. P 480,000
d. P 487,500
50. What is the estimated cost of ending inventory under the FIFO retail? Round off
cost ratio into whole number.
D a. P 512,000
b. P 450,000
c. P 480,000
d. P 487,500

Page 10 of 14 0915-2303213  resacpareview@gmail.com


FINANCIAL ACCOUNTING & REPORTING
ReSA Batch 43 - May 2022 CPALE Batch
11 Feb 2022  6:00 PM to 9:00 PM FAR First Pre-Board Exam
51. Assume an investor purchases bonds at a premium the bonds are to be held as a
long-term investment which of the following statement is true regarding the
amount of bond interest revenue to be reported over life of the bonds?
D a. The periodic amount of bond interest revenue will always be above the
periodic amount of cash received for interest
b. The pattern of the periodic amortization of bond interest revenue is
an increasing amount
c. The periodic amount of bond interest revenue will always be equal to
the periodic amount of cash received
d. The periodic amount of bond interest revenue will always be less than
the periodic amount of cash received
52. When a company uses the perpetual inventory system in accounting for its
merchandise inventory, which of the following is false?
A a. Total cost of goods sold is computed by deducting ending inventory
from total goods available for sale
b. The inventory account is updated after each sale
c. One of the entries to record return of goods is debit inventory and
credit cost of goods sold
d. One of the entries to record the sale of goods is debit cost of goods
sold and credit inventory
53. Which of the following notes receivable shall be reported at face value?
D a. Long term non-interest-bearing notes receivable
b. Long term interest-bearing notes receivable with stated rate higher
than effective rate
c. Long term interest-bearing notes receivable with stated rate lower
than effective rate
d. Long-term interest-bearing notes receivable with stated rate equals
effective rate
54. Which of the following is true with regards to the accrued interest on investment
in debt securities that are sold between interest dates?
D a. The accrued interest is computed using the effective rate
b. The accrued interest will be paid to the seller when the bonds mature
c. The accrued interest is extra income to the buyer and treated as bond
issue cost of the buyer
d. The accrued interest is added to the issue price of the bond to
determine the total cash payments from bond acquisition
55. Debt securities designated initially as FVOCI and reclassified to IAC, interest
income recognized subsequent to reclassification is
D a. Fair value on reclassification date x Effective rate on
reclassification date
b. Fair value on reclassification date x Original effective rate
c. Amortized cost on reclassification date x Effective rate on
reclassification date
d. Amortized cost on reclassification date x Original effective rate
56. Which of the following is true about equity method in accounting for investment
in associate?
B a. Investment should be carried at the balance sheet date at fair value
less cost to sell
b. Impairment loss shall be recognized if recoverable value is lower
than carrying amount
c. Transaction cost should be expensed outright at initial recognition
d. Dividend received is reported in the income statement
57. The following statements are based on PAS 28 (Investment in Associates):
Statement I: An investment in an associate shall be accounted for using the
equity method (benchmark) or cost method (alternative).
Statement II: An investor shall discontinue the use of equity method from the
date when it ceases to have significant influence over an associate and shall
account for the investment in accordance with PFRS 9.
Statement III: On the loss of significant influence, the investor shall measure
at historical cost any investment the investor retains in the former associate.
B a. Only statement I is false
b. Only statement II is true
c. Only statement III is true
d. All of the statements are false

Page 11 of 14 0915-2303213  resacpareview@gmail.com


FINANCIAL ACCOUNTING & REPORTING
ReSA Batch 43 - May 2022 CPALE Batch
11 Feb 2022  6:00 PM to 9:00 PM FAR First Pre-Board Exam

58. In case of an investment in associate reclassified to fair value through OCI due
to loss of significant influence, the gain/loss from cessation that is unrealized
shall
A a. Be reported in profit or loss
b. Be reported in OCI
c. Be directly reported in retained earnings
d. Not be accounted for
59. Which of the following would result to a decrease in accounts receivable of the
seller and decrease in accounts payable of the buyer in a sale of goods on
account?
B a. FOB Destination, freight prepaid
b. FOB Destination, freight collect
c. FOB Shipping point, freight prepaid
d. FOB Shipping point, freight collect
60. A Cash Over and Short account
C a. is not generally accepted.
b. is debited when the petty cash fund proves out over.
c. is debited when the petty cash fund proves out short.
d. is a contra account to Cash.
61. The journal entries for a bank reconciliation
B a. are taken from the "balance per bank" section only.
b. may include a debit to Office Expense for bank service charges.
c. may include a credit to Accounts Receivable for an NSF check.
d. may include a debit to Accounts Payable for an NSF check.
62. When preparing a bank reconciliation, credit memos are
C a. added to the bank statement balance.
b. deducted from the bank statement balance.
c. added to the balance per books.
d. deducted from the balance per books.
63. Which of the following is true regarding the reclassification from investment at
fair value through profit or loss to investment at amortized cost?
A a. Reclassification date is made at the beginning of the next accounting
period
b. The initial cost of investment at amortized cost is the amortized
cost at the date of reclassification
c. The difference between the fair value and the face value is a gain
or loss on reclassification taken to profit or loss
d. The carrying amount and the fair value at the date of reclassification
should be amortized over the remaining life of the bond.
64. Which of the following will result if the current year’s ending inventory amount
is understated?
B a. Cost of goods sold will be understated.
b. Gross profit will be understated.
c. Net income will be overstated.
d. Retained earnings will be overstated.
65. For an investment in debt securities classified as FVOCI, unrealized gain taken
to Equity is
B a. The excess of fair value over the original cost
b. The excess of fair value over the amortized cost
c. The excess of original cost over the fair value
d. The excess of amortized cost over the fair value
66. When the allowance method of recognizing bad debts expense is used, the entry to
write-off a specific customer account would
B a. Decrease in net income
b. Have no effect on net income
c. Have no effect on accounts receivable
d. Increase net income

Page 12 of 14 0915-2303213  resacpareview@gmail.com


FINANCIAL ACCOUNTING & REPORTING
ReSA Batch 43 - May 2022 CPALE Batch
11 Feb 2022  6:00 PM to 9:00 PM FAR First Pre-Board Exam

67. Which of the following statements is not true regarding trade discounts?
C a. Trade discounts are means of converting a catalog list price to the
prices actually charged to the buyer
b. Trade discounts are used to avoid frequent changes in catalogs and
to hide the true invoice price from competitors
c. Trade discounts are recognized for financial accounting purposes
d. Trade discounts are always deducted from the list price in recording
accounts receivable arising from a credit sale transaction
68. After being held for 40 days, a 120-day, 12% interest-bearing note receivable
was discounted with recourse at a bank at the discount rate of 15%. Which of
the following statements is incorrect relating to the discounted notes?
D a. Accrued interest computed at 12% for 40 days is recorded in the
accounts
b. Discount is computed based on the maturity value of the note at 15%
for 80 days
c. Proceeds is equal to the maturity value less discount at 15% for 80
days
d. A loss is recognized equal to the excess of the note and accrued
interest over the proceeds from the discounting
69. The entry debiting accounts receivable and crediting allowance for bad debts
would be made when
C a. A customer pays its accounts receivable
b. A customer defaults on its account
c. A previously defaulted customer pays its outstanding balance
d. Estimated uncollectible receivables are too slow
70. On December 31, 2020 balance sheet, a notes receivable was reported as a
noncurrent asset and its accrued interest for eight months was reported as a
current asset. Which of the following terms would fit the notes receivable?
D a. Both principal and interest amounts are payable on April 30, 2021 and
April 30, 2022
b. Principal and interest are due December 31, 2022
c. Both principal and interest amounts are payable on December 31, 2021
and December 31, 2022
d. Principal is due April 30, 2022 and interest is due April 30, 2021
and April 30, 2022.

- END –

Page 13 of 14 0915-2303213  resacpareview@gmail.com


FINANCIAL ACCOUNTING & REPORTING
ReSA Batch 43 - May 2022 CPALE Batch
11 Feb 2022  6:00 PM to 9:00 PM FAR First Pre-Board Exam

ANSWERS & SOLUTIONS/CLARIFICATIONS


1. 180,000 + 2,000,000 + 250,000 + 120,000 + 3,000,000 + 1,000,000 + 500,000 +1,000,000 +1,200,000
+60,000 +100,000 = 9,410,000
2. 17,500 – 3,200 – 600 -1,800 – 840 = 11,060
3. 2,000 +2,500 – 500 = 4,000
4. (2,000 + 1,200 + 1,800 + 2,500 + 1,500 – 500) – 10,000 = 1,500 short
5. (7,000 + 13,000 + 8,000 + 3,000 + 10,000 + 3,000 + 2,000 – 4,000) – 50,000 = 8,000
6. (581,050 – 84,300 +120,000) – (627,000 +1,250 – 400) = 11,100
7. 39,000 + 610,000 – (617,000 – 15,000) = 47,000
8. 17,000 + 490,000 – (476,000 – 25,000) = 56,000
9. 80,000 + (80,000 x 3.037) = 322,960
10. (80,000 x 3.037) x 12% x 4/12 = 9,719
11. (4,000,000 x .80 x .90) x 97% = 2,793,600
12. 674,000 + 3,000,000 + 6,000 – 1,800,000 – 1,000,000 – 6,000 – 458,000 -22,000 -12,000 = 382,000
13. 3,000 + 9,600 + 12,900 + 7,800 = 33,300
14. (490,000 + 30,000) – 520,000 = 80,000
15. WITH recourse = No gain or loss
16. 4,800,000 – (7,100,000 x 50%) = 1,250,000
17. 5,400,000 + 552,000 – 120,000 + 840,000 – 510,000 = 6,162,000
18. (20,000 x 250) – (6,162,000 x 2/3) = 892,000
19. 150,000 – (250,000 x 30%) – (100,000 x 30%) = 45,000
20. 650,000 + (480,000 x 30%) – (30,000 /5 x 8/12) – (18,000 x P4) = 718,000
21. 25% x (5M -50,000) = 1,237,500
22. FV, 12/31/22 = 222,000
23. 3,000 shares x P4/share = 12,000
24. (30,000 x 3) – (18M / 30 yrs. x 9/12) = 360,000
25. (200,000-8,000) + 800,000 - (300,000 – 1,000) = 693,000
26. Same with direct method
27. 8,000,000 – (10M x 6/8) = 500,000
28. 3,500,000 + (4M x35%) – (200,000 x 35%) – (1M x 35%) = 4,480,000
29. FVCSTS (62,000) vs. CV (60,000) = No gain or loss
30. Increase in FVCTS amounting to 40,000 exceeded the previous impairment of 30,000 (300,000 – 270,000)
31. 20,000,000 – 2,000,000 = 18,000,000
32. 26,000,000 – 600,000 = 25,400,000
33. 18,000,000 x 29/30 = 17,400,000
34. 1,260,000 -120,000 / 6 yrs. = 190,000
35. 270,000 tons x P2.00 per ton = 540,000
36. 240,000 tons x P2.00 per ton = 480,000
37. (2,500,000 x 95%) + 10,000 + 15,000 = 2,400,000
38. (1,200,000 – 600,000) +160,000 = 760,000; No gain or loss since without commercial substance
39. (2,294,416 x 6% x 8/12) – (2,000,0000 x 8% x 8/12) = 14,890
40. (2,000,000 x 1.02 – 160,000) – (2/5 x 5,200,000) = 200,000 loss
41. (5M x12% x 4/12) + (3M x 12% x 1/12) + (2M x 12% x 4/12) = 310,000
42. (2,940,000 + 1,980,000) – (3,120,000 + 1,960,000) = 160,000 UL
43. 5,250,000 – 4,910,944 = 339,056
44. 339,056 – 268,800 = 70,256
45. 5,500,000 – 4,955,472 = 544,528
46. 4,400,000 + 400,000 + 700,000 = 5,500,000
47. (500 x 3.50) + (800 x 3.40) = 4,470
48. 1,300 units x P3.26 = 4,238
49. 750,000 x 64% = 480,000
50. 750,000 x 65% = 487,500

Page 14 of 14 0915-2303213  resacpareview@gmail.com


ReSA - THE REVIEW SCHOOL OF ACCOUNTANCY
CPA Review Batch 43  May 2022 CPALE  10 Feb 2022  6:00 PM – 9:00 PM

REGULATORY FRAMEWORK for BUSINESS TRANSACTIONS FIRST PRE-BOARD EXAMINATION

INSTRUCTIONS: Select the correct answer for each of the questions. Mark only one
answer for each item by shading the box corresponding to the letter of your choice on
the answer sheet provided. STRICTLY NO ERASURES ALLOWED. Use pencil no. 2 only.

1. Offspring of animals are considered as


A a. Natural fruits
b. Industrial fruits
c. Civil fruits
d. Juicy fruits
2. H, I and J are joint debtors. They obliged themselves to deliver a specific
rooster (worth P9,000) to P and Q, joint creditors. Assuming H is the only one
who does not want to comply with the obligation, how much is the total amount
that can be collected from him by the creditors?
C a. P9,000
b. P9,000 plus damages
c. P3,000 plus damages
d. None. He is not forced to comply.
3. A, B and C are solidary debtors for the amount of P45,000 from F. Mr. B has
already paid P20,000 to F. How much can F further collect from B?
A a. P25,000
b. None. It has already exceeded the share of B.
c. Another P20,000
d. P5,000
4. One-peso coins are considered to be legal tender up to
A a. One thousand pesos
b. One hundred pesos
c. One million pesos
d. Up to any amount
5. Five centavo coins are considered to be legal tender up to
B a. One thousand pesos
b. One hundred pesos
c. One million pesos
d. Up to any amount
6. Fifty-peso bills are considered to be legal tender up to
D a. One thousand pesos
b. One hundred pesos
c. One million pesos
d. Up to any amount
7. On January 1, Diego borrowed P20,000 from Cardo. On February 1, the former
borrowed another P10,000 from the latter, evidenced by an interest-bearing
promissory note. Both debts are now due. Upon demand by Cardo, Diego paid
P6,000 without designating the liability on which it would be applied. How will
the P6,000 be applied?
B a. It will be considered as payment for the January 1 debt only
b. It will be considered as payment for the February 1 debt only
c. It will be considered as payment for both debts, in the ratio of
1:1
d. It will be considered as payment for both debts, in the ratio of
2:1 respectively
8. On March 1, Desiree borrowed P100,000 from Cristine. On July 1, the former
borrowed another P500,000 from the latter. Both debts are now due. Upon demand
by Cristine, Desiree paid P40,000 without designating the liability on which it
would be applied. How will the P40,000 be applied?
D a. It will be considered as payment for the March1 debt only
b. It will be considered as payment for the July 1 debt only
c. It will be considered as payment for both debts, in the ratio of 1:1
d. It will be considered as payment for both debts, in the ratio of 1:5
respectively

Page 1 of 12 0915-2303213  resacpareview@gmail.com


REGULATORY FRAMEWORK for BUSINESS TRANSACTIONS
ReSA Batch 43 - May 2022 CPALE Batch
10 Feb 2022  6:00 PM to 9:00 PM RFBT First Pre-Board Exam
9. The contract must bind both contracting parties and its validity or compliance
cannot be left to the will of one of them.
B a. Liberty to contract
b. Mutuality of contracts
c. Relativity of contracts
d. Consensuality of contracts
10. It is a remedy in equity by means of which a written instrument is made or
construed so as to express or conform to the real intention of the parties when
some error or mistake has been committed.
D a. Restitution
b. Rescission
c. Reparation
d. Reformation
11. Which among the following is considered to be the most defective?
B a. Voidable
b. Void
c. Rescissible
d. Unenforceable
12. Gary is the guardian of Monina, a minor. He sold Monina’s car with a fair value
of P 200,000 for just P 180,000. The contract entered into by Gary is
C a. Voidable
b. Void
c. Valid
d. Rescissible
13. Under a contract, Mr. X obligated himself to build a house for Mr. Z. In this
example, the object or the prestation is
D a. Mr. X
b. The house itself
c. Mr. Z
d. The construction of the house
14. An obligation to deliver a cat named Pussy is an example of
D a. Negative personal obligation
b. Generic real obligation
c. Positive personal obligation
d. Specific real obligation
15. During a strong typhoon, Mr. X saved the horses of Mr. Z, who is currently out
of town. Mr. X incurred expenses amounting to P5,000. This can be reimbursed
because of
B a. Solutio indebiti
b. Negotiorum gestio
c. Fortuitous event
d. Natural obligation
16. Juan received P20,000 from Pedro because the latter thought that Juan is his
creditor Juancho. Juan should return the money because of
B a. Contract
b. Solutio indebiti
c. Negotiorum gestio
d. Public policy
17. The civil liability arising from crimes may include the following, except:
C a. Reparation
b. Indemnification for consequential damages
c. Involuntary servitude
d. Restitution
18. After a strong storm, Pedro did not remove the broken branches of his mango tree.
Few days after, a passerby was hit by a branch that fell. Is Pedro liable?
D a. No, it is a fortuitous event.
b. Yes, he is liable for a crime.
c. No, it is a mere accident.
d. Yes, he is liable for a quasi-delict.

Page 2 of 12 0915-2303213  resacpareview@gmail.com


REGULATORY FRAMEWORK for BUSINESS TRANSACTIONS
ReSA Batch 43 - May 2022 CPALE Batch
10 Feb 2022  6:00 PM to 9:00 PM RFBT First Pre-Board Exam
19. “I promise to deliver my dog Bruno to you next week.” The animal contemplated
in this obligation is a
D a. Generic object
b. Specific object
c. Determinate object
d. Both b and c
20. The diligence required from common carriers is
B a. Ordinary diligence
b. Extra ordinary diligence
c. Diligence of a good father of a family
d. No diligence at all
21. Mr. S sold a pig named Peachy to Ms. B for P5,000. No condition or period was
contemplated. After the perfection of the contract but before the delivery of
the pig, Peachy the pig gave birth to 5 piglets. Who is entitled to the 5
piglets?
B a. Mr. S
b. Ms. B
c. Peachy, because she is the mother of the piglets
d. None of the above
22. On December 1, 2018, Mr. S agreed to sell a car worth P1,000,000 to Mr. B. The
car will be delivered to Mr. B on December 31, 2018. Prior to December 31,
2018, what kind of right does Mr. B possess?
A a. Personal right
b. Real right
c. Both a and b
d. None of the above
23. Assume further that delivery has been made to Mr. B. What kind of right does he
now possess?
D a. Personal right binding against Mr. S
b. Real right binding against Mr. S
c. Personal right binding against the whole world
d. Real right binding against the whole world
24. Ownership over a property is transferred during the
B a. Perfection of the contract
b. Delivery of the subject matter
c. Payment of the price
d. All of the above
25. Mr. S obliged himself to deliver to Mr. B 50 sacks of sugar for P20,000. Mr. S
was not able to comply with his undertaking. Mr. B was now forced to find a new
seller of sugar. Mr. B was able to buy 50 sacks of sugar from Mr. T for P20,000
also. Is Mr. S still liable?
C a. No, because there was no damage on the part of Mr. B. The sugar
still costs P20,000.
b. No, because Mr. B was able to get a replacement.
c. Yes, Mr. S incurred a breach of contract.
d. Both a and b.
26. What principle is the basis of the rule that an indeterminate thing cannot be
the object of destruction?
C a. Force majeure
b. Negotiorum gestio
c. Genus nunquam perit
d. Compensatio morae
27. Bobby is contemplating whether to buy Susan’s car for P400,000. To give him
more time, he gave a P20,000 option money to Susan. He finally decided to buy
the subject matter after 2 days. How much should Bobby give to Susan at the
date of payment?
A a. P400,000
b. P380,000
c. P420,000
d. Cannot be computed

Page 3 of 12 0915-2303213  resacpareview@gmail.com


REGULATORY FRAMEWORK for BUSINESS TRANSACTIONS
ReSA Batch 43 - May 2022 CPALE Batch
10 Feb 2022  6:00 PM to 9:00 PM RFBT First Pre-Board Exam
28. Holders of nonvoting shares shall nevertheless be entitled to vote on the
following matters, except:
D a. Amendment of the articles of incorporation
b. Adoption and amendment of bylaws;
c. Sale, lease, exchange, mortgage, pledge, or other disposition of
all or substantially all of the corporate property
d. None of the choices is an exception
29. Holders of nonvoting shares shall nevertheless be entitled to vote on the
following matters, except:
D a. Incurring, creating, or increasing bonded indebtedness
b. Increase or decrease of authorized capital stock
c. Merger or consolidation of the corporation with another corporation
or other corporations
d. None of the choices is an exception
30. No-par value shares must be issued for a consideration of at least
A a. Five pesos per share
b. One peso per share
c. Ten pesos per share
d. One hundred pesos per share
31. For founders’ shares, where the exclusive right to vote and be voted for in the
election of directors is granted, it must be for a limited period not to exceed
____ year/s from the date of incorporation
C a. One
b. Two
c. Five
d. Ten
32. These are shares of stock which have been issued and fully paid for, but
subsequently reacquired by the issuing corporation through purchase,
redemption, donation, or some other lawful means.
B a. Redeemable shares
b. Treasury shares
c. Watered stocks
d. Nominal shares
33. Any person, partnership, association or corporation, singly or jointly with
others but not more than __________ in number, may organize a corporation for
any lawful purpose or purposes:
C a. 5
b. 10
c. 15
d. None of the above
34. Under the Revised Corporation Code, a corporate term for a specific period may
be extended or shortened by amending the articles of incorporation: Provided,
That no extension may be made earlier than ________ years prior to the original
or subsequent expiry date(s) unless there are justifiable reasons for an
earlier extension as may be determined by the Commission:
B a. Two
b. Three
c. Five
d. Ten

35. Unless otherwise prescribed by the Corporation Code or by special law, and for
legitimate purposes, any provision or matter stated in the articles of
incorporation may be amended by a majority vote of the board of directors or
trustees and the vote or written assent of the stockholders representing at
least _______ of the outstanding capital stock, without prejudice to the
appraisal right of dissenting stockholders in accordance with the provisions of
this Code.
C a. 1/3
b. 1/2
c. 2/3
d. 3/4

Page 4 of 12 0915-2303213  resacpareview@gmail.com


REGULATORY FRAMEWORK for BUSINESS TRANSACTIONS
ReSA Batch 43 - May 2022 CPALE Batch
10 Feb 2022  6:00 PM to 9:00 PM RFBT First Pre-Board Exam

36. The due incorporation of any corporation claiming in good faith to be a


corporation under the Corporation Code, and its right to exercise corporate
powers, shall not be inquired into collaterally in any private suit to which
such corporation may be a party. Such inquiry may be made by the Solicitor
General in a ____________ proceeding.
B a. Impeachment
b. Quo warranto
c. Both A & B
d. None of the above
37. Under the Revised Corporation Code, if a corporation does not formally organize
and commence its business within _____ years from the date of its
incorporation, its certificate of incorporation shall be deemed revoked as of
the day following the end of such period
C a. Two
b. Three
c. Five
d. Ten
38. Under the Corporation Code, if a corporation has commenced its business but
subsequently becomes inoperative for a period of at least ________ consecutive
years, the Commission may, after due notice and hearing, place the corporation
under delinquent status.
B a. Three
b. Five
c. Ten
d. Twenty
39. A delinquent corporation shall have a period of __________ year/s to resume
operations and comply with all requirements that the Commission shall
prescribe.
B a. One
b. Two
c. Three
d. Five
40. Directors shall be elected for a term of one (1) year from among the holders of
stocks registered in the corporation’s books, while trustees shall be elected
for a term not exceeding ______ year/s from among the members of the
corporation.
C a. One
b. Two
c. Three
d. Six
41. Any director or trustee of a corporation may be removed from office by a vote
of the stockholders holding or representing at least ___________ of the
outstanding capital stock.
B a. 1/2
b. 2/3
c. 3/4
d. All
42. In the absence of any provision in the bylaws fixing their compensation, the
directors or trustees shall not receive any compensation in their capacity as
such, except for reasonable ________.
C a. Honorariums
b. Talent fee
c. Per diems
d. None of the above

43. Directors or trustees who willfully and knowingly vote for or assent to
patently unlawful acts of the corporation or who are guilty of gross negligence
or bad faith in directing the affairs of the corporation or acquire any
personal or pecuniary interest in conflict with their duty as such directors or
trustees shall be liable ______________ for all damages resulting therefrom
suffered by the corporation, its stockholders or members and other persons.

Page 5 of 12 0915-2303213  resacpareview@gmail.com


REGULATORY FRAMEWORK for BUSINESS TRANSACTIONS
ReSA Batch 43 - May 2022 CPALE Batch
10 Feb 2022  6:00 PM to 9:00 PM RFBT First Pre-Board Exam
B a. Jointly
b. Solidarily
c. Both A & B
d. None of the above
44. Under the Revised Corporation Code, no management contract shall be entered
into for a period longer than _______ years for any one (1) term.
A a. Five
b. Ten
c. Twenty
d. Fifty
45. The owners of _________ of the outstanding capital stock may delegate to the
board of directors the power to amend or repeal the bylaws or adopt new bylaws.
C a. 1/2
b. Majority
c. 2/3
d. 3/4
46. Regular meetings of stockholders or members shall be held annually on a date
fixed in the bylaws, or if not so fixed, on any date after ______ of every year
as determined by the board of directors or trustees.
C a. January 15
b. March 15
c. April 15
d. December 15
47. Special meetings of stockholders or members shall be held at any time deemed
necessary or as provided in the bylaws: Provided, however, that at least ______
week/s written notice shall be sent to all stockholders or members, unless a
different period is provided in the bylaws, law or regulation.
A a. One
b. Two
c. Three
d. Ten
48. Each incorporator of a stock corporation must own or be a subscriber to at
least ______ share/s of the capital stock.
A a. One
b. Five
c. Ten
d. One hundred
49. A corporation shall have perpetual existence unless its ________________
provides otherwise.
A a. Articles of Incorporation
b. Stock Transfer Book
c. Treasurer’s Affidavit
d. None on the above

50. The board of corporations vested with public interest shall have independent
directors constituting at least _______________ of such board.
B a. 10%
b. 20%
c. 30%
d. 50%
51. A person shall be disqualified from being a director, trustee or officer of any
corporation if, within five years prior to the election or appointment as such,
the person was convicted by final judgment of an offense punishable by
imprisonment for a period exceeding _________ years.
B a. Five
b. Six
c. One
d. Ten

Page 6 of 12 0915-2303213  resacpareview@gmail.com


REGULATORY FRAMEWORK for BUSINESS TRANSACTIONS
ReSA Batch 43 - May 2022 CPALE Batch
10 Feb 2022  6:00 PM to 9:00 PM RFBT First Pre-Board Exam
52. In no case shall the total yearly compensation of directors exceed ______ of
the net income before income tax of the corporation during the preceding year.
B a. 5%
b. 10%
c. 15%
d. 20%
53. A contract of the corporation with (1) one or more of its directors, trustees,
officers or their spouses and relatives within the fourth civil degree of
consanguinity or affinity is ____________, at the option of such corporation.
B a. Void
b. Voidable
c. Unenforceable
d. Rescissible
54. Stockholdings exceeding ______________ of the outstanding capital stock shall
be considered substantial for purposes of interlocking directors.
C a. 10%
b. 15%
c. 20%
d. 30%
55. If the bylaws so provide, the board may create an executive committee composed
of at least _________ directors.
B a. Two
b. Three
c. Five
d. Ten
56. A private corporation may extend or shorten its term as stated in the articles
of incorporation when approved by a __________ vote of the board of directors
or trustees, and ratified at a meeting by the stockholders or members
representing at least two-thirds (2/3) of the outstanding capital stock or of
its members.
C a. 1/3
b. 1/2
c. Majority
d. 2/2
57. Stock corporations are prohibited from retaining surplus profits in excess of
______________ of their paid-in capital stock.
D a. 5%
b. 10%
c. 20%
d. 100%
58. For the adoption of bylaws by the corporation, the affirmative vote of the
stockholders representing at least ______________of the outstanding capital
stock, shall be necessary
B a. 1/2
b. Majority
c. 1/3
d. 2/3
59. ABC Corporation has a principal office in Pacific Star Building, Makati City.
What is the proper venue of the stockholder’s meeting under the Revised
Corporation Code as a general rule?
A a. Pacific Star Building
b. Any venue in Makati City only
c. Any venue in Metro Manila
d. Any venue in Luzon
60. DEF Corporation has a principal office in Lancaster Tower Building, Pasig City.
What is the proper venue of the stockholder’s meeting in case it is impractical
to conduct the meeting in Lancaster Tower Building?
C a. Lancaster Tower Building only
b. Any venue in Pasig City
c. Any venue in NCR
d. Any venue in the Philippines

Page 7 of 12 0915-2303213  resacpareview@gmail.com


REGULATORY FRAMEWORK for BUSINESS TRANSACTIONS
ReSA Batch 43 - May 2022 CPALE Batch
10 Feb 2022  6:00 PM to 9:00 PM RFBT First Pre-Board Exam
61. GHI Corporation has a principal office in Amparo Building, Kawit, Cavite. What
is the possible venue of the stockholder’s meeting in case it is impractical to
conduct the meeting in Amparo Building?
B a. Amparo Building only
b. Any venue in Kawit, Cavite
c. Any venue in the Cavite Province
d. Any venue in Calabarzon area
62. The chairman or, in his absence, the ________________ shall preside at all
meetings of the directors or trustees as well as of the stockholders or
members, unless the bylaws provide otherwise.
A a. President
b. Corporate Secretary
c. Treasurer
d. Both B&C
63. A subscription of shares in a corporation still to be formed shall be
irrevocable for a period of at least ________ month/s from the date of
subscription, unless all of the other subscribers consent to the revocation, or
the corporation fails to incorporate within the same period or within a longer
period stipulated in the contract of subscription.
D a. One
b. Two
c. Three
d. Six
64. A partnership is considered nominate because
B a. It is perfected by mere consent
b. It has a designation in law and specific rules applicable to it
c. It is entered into by two or more persons
d. Each of the parties is required to give something
65. A partnership is considered to be onerous because
D a. It is perfected by mere consent
b. It has a special name or designation in law
c. It is entered into by two or more persons
d. Each of the parties is required to give something
66. It means “choice of the person”. Due to this principle, no one can be a member
of the partnership association without the consent of all the other associates.
C a. Solutio indebiti
b. Negotiorum gestio
c. Delectus personae
d. Personae non grata
67. Who among the following can enter in a contract of partnership?
C a. Unemancipated minors
b. Insane or demented persons
c. Deaf mutes who know how to write
d. Persons who are suffering from civil interdiction
68. It is a kind of partnership where no time is specified and is not formed for a
particular undertaking
A a. Partnership at will
b. Partnership with a fixed term
c. De jure partnership
d. De facto partnership
69. One wherein the existence of certain persons as partners is not avowed or known
A a. Secret partnership
b. Open partnership
c. Notorious partnership
d. Commercial partnership

Page 8 of 12 0915-2303213  resacpareview@gmail.com


REGULATORY FRAMEWORK for BUSINESS TRANSACTIONS
ReSA Batch 43 - May 2022 CPALE Batch
10 Feb 2022  6:00 PM to 9:00 PM RFBT First Pre-Board Exam
70. One whose liability to third persons is limited to his capital contribution
B a. General partner
b. Limited partner
c. Both A&B
d. None of the above
71. One who does not take an active part in the business and is not known or held
out as a partner
C a. Silent partner
b. Secret partner
c. Dormant partner
d. Both B and C
72. If the industrial partner violates the prohibition to engage in a business for
himself, the capitalist partners may
C a. Exclude him from the firm plus damages
b. Avail themselves of the benefits plus damages
c. Either A or B, but not both at the same time
d. Both A and B at the same time
73. As regards capitalist partners, the prohibition may extend to operation which
is of
A a. The same kind of business in which the partnership is engaged
b. A different kind of business in which the partnership is engaged
c. Both A and B
d. None of the above
74. J is the managing partner in JKL Partnership. Mr X, a third person, owe P10,000
from J personally. Likewise, Mr. X also owe P20,000 from the JKL Partnership.
One day, Mr. X paid P6,000 to Partner J. The managing partner issued a receipt
for Mr. X’s personal liability to him. How much of the P6,000 should go to the
partnership?
D a. P6,000
b. 0
c. P2,000
d. P4,000
75. J is the managing partner in JKL Partnership. Mr X, a third person, owe P10,000
from J personally. Likewise, Mr. X also owe P20,000 from the JKL Partnership.
One day, Mr. X paid P6,000 to Partner J. The managing partner issued a receipt
for Mr. X’s liability to the JKL Partnership. How much of the P6,000 should go
to Partner J?
B a. P6,000
b. 0
c. P2,000
d. P4,000
76. Three brothers inherited a condo unit from their deceased mother. It
continuously gave them rental income every month. As a result, a
_______________ was established among the partners.
B a. Partnership
b. Co-ownership
c. Suretyship
d. Friendship
77. Three sisters inherited a 4 door apartment from their deceased father. Due to
the fact that it continuously gave them rental income every month, they decided
to upgrade such to an 8 door apartment. As a result, a _______________ was
established among the partners.
A a. Partnership
b. Co-ownership
c. Suretyship
d. Friendship

Page 9 of 12 0915-2303213  resacpareview@gmail.com


REGULATORY FRAMEWORK for BUSINESS TRANSACTIONS
ReSA Batch 43 - May 2022 CPALE Batch
10 Feb 2022  6:00 PM to 9:00 PM RFBT First Pre-Board Exam
78. A partnership, as a rule, may be constituted ________________.
C a. Orally
b. In writing
c. Both a & b
d. None of the above
79. Persons who consent to others representing him as a partner in an existing
partnership is liable as a _____________.
C a. Silent partner
b. Secret partner
c. Partner by estoppel
d. Dormant partner
80. The contributions of a limited partner may be _________
D a. Cash
b. Other property
c. Services or Industry
d. Both A & B only
81. The surname of a limited partner shall not appear in the partnership name
unless
C a. It is also the surname of a general partner
b. Prior to the time when the limited partner became such, the
business has been carried on under a name in which his surname
appeared.
c. Both a & b
d. None of the above
82. The losses and profits shall be distributed _______________
A a. in conformity with the agreement of the partners
b. equally
c. proportionately
d. none of the above
83. Partners P, Q, R and S are partners in a taxi business. On a stormy night,
Partner S (the only industrial partner) was negligent in driving their cab. His
negligence resulted to the death of Mr. Boyet (a passenger) and actual damages
of P40,000 was incurred for his funeral and medical expenses. How much can the
family of Mr. Boyet collect from innocent partner P in this case?
A a. P40,000
b. P10,000
c. Zero
d. P30,000
84. Which of the following has the least priority when it comes to the liquidation
of a limited partnership?
B a. Those to general partners in respect to profits
b. Those to general partners in respect to capital
c. Those to limited partners, other than capital or profits
d. Those to outside creditors
85. Which of the following is the top priority when it comes to the liquidation of
a limited partnership?
D a. Those owing to general partners in respect to profits
b. Those owing to general partners in respect to capital
c. Those owing to limited partners, in respect to capital
d. Those owing to outside creditors
86. Which of the following has the least priority when it comes to assets of a
general partnership?
A a. Those owing to general partners in respect to profits
b. Those owing to general partners in respect to capital
c. Those owing to general partners, other than capital or profits
d. Those to outside creditors

Page 10 of 12 0915-2303213  resacpareview@gmail.com


REGULATORY FRAMEWORK for BUSINESS TRANSACTIONS
ReSA Batch 43 - May 2022 CPALE Batch
10 Feb 2022  6:00 PM to 9:00 PM RFBT First Pre-Board Exam
87. It is a corporation that was formed for a religious purpose.
A a. Ecclesiastical corporation
b. Lay corporation
c. Eleemosynary corporation
d. Civil corporation
88. It is a corporation organized for public charity.
C a. Ecclesiastical corporation
b. Lay corporation
c. Eleemosynary corporation
d. Civil corporation
89. These shares entitle the holder to payment not only of current dividend but
also those in arrears, when dividends are declared.
C a. Participating preferred stock
b. Non-participating preferred stock
c. Cumulative preferred stock
d. Non-cumulative preferred stock
90. These are shares that were issued without consideration or consideration below
par or stated value.
C a. Treasury stock
b. Delinquent stock
c. Watered stock
d. No par value stock
91. The voting rights of common shares may be limited by the existence of
C a. Preferred shares
b. Treasury shares
c. Founders’ Shares
d. Redeemable shares
92. What is the vote required in an amendment of the articles of incorporation?
C a. Majority vote of the BOD + Majority vote of the outstanding capital stock
b. 2/3 vote of the BOD + 2/3 vote of the outstanding capital stock
c. Majority vote of the BOD + 2/3 vote of the outstanding capital stock
d. None of the choices is correct
93. Under the Articles of Incorporation of DEF Corporation, it was stated that it
should have 15 directors. In order to have a quorum in their next meeting, what
is the minimum number of directors that should be able to attend?
B a. 7
b. 8
c. 10
d. 15
94. In relation to the preceding number, assuming 10 directors were present in the
BOD meeting of DEF Corporation, what is the minimum number of affirmative votes
that should be garnered if the corporation would like to push through with the
purchase of a certain asset?
B a. 5
b. 6
c. 8
d. 10
95. There are nine directors in GHI Corporation. One of them was validly removed as
a director. Who has the power to fill such vacancy?
A a. The stockholders only
b. The remaining eight directors
c. The remaining eight directors and the majority of the stockholders
d. The remaining eight directors and 2/3 of the stockholders
96. There are nine directors in JKL Corporation. Two of the directors died in a car
accident. Who has the power to fill such vacancies?
B a. The stockholders only
b. The remaining seven directors
c. The remaining seven directors and the majority of the stockholders
d. The remaining seven directors and 2/3 of the stockholders

Page 11 of 12 0915-2303213  resacpareview@gmail.com


REGULATORY FRAMEWORK for BUSINESS TRANSACTIONS
ReSA Batch 43 - May 2022 CPALE Batch
10 Feb 2022  6:00 PM to 9:00 PM RFBT First Pre-Board Exam
97. There are ten directors in MNO Corporation. Five of them resigned due to an
internal conflict. Who has the power to fill such vacancies?
A a. The stockholders only
b. The remaining five directors
c. The remaining five directors and the majority of the stockholders
d. The remaining five directors and 2/3 of the stockholders
98. The amount allocated to the community development fund of the cooperative shall
not be more than __ of the net surplus.
C a. 7%
b. 10%
c. 3%
d. 5%
99. In case of voluntary dissolution where creditors are affected, the petition for
dissolution shall be filed with:
A a. Cooperative Development Authority
b. Securities and Exchange Commission
c. Regional Trial Court
d. Municipal Trial Court
100. The share capital of a primary cooperative shall have a par value fixed to any
figure not more than:
B a. P500
b. P1,000
c. P5,000
d. P10,000

- END –

Page 12 of 12 0915-2303213  resacpareview@gmail.com


ReSA - THE REVIEW SCHOOL OF ACCOUNTANCY
CPA Review Batch 43  May 2022 CPALE  12 Feb 2022  6:00 PM – 9:00 PM

ADVANCED FINANCIAL ACCOUNTING & REPORTING FIRST PRE-BOARD EXAMINATION

INSTRUCTIONS: Select the correct answer for each of the questions. Mark only one
answer for each item by shading the box corresponding to the letter of your choice on
the answer sheet provided. STRICTLY NO ERASURES ALLOWED. Use pencil no. 2 only.

Items 1 to 5 are based on the following information:


In 2022, DJD Builders Construction began work on a three-year construction project to
build a new performing arts complex (the PAC). The PAC contract price is P150 million.
DJD Builders recognizes revenue on this contract over time according to percentage of
completion. At the end of 2022, the following financial statement information indicates
the results to date for the PAC (missing items denoted by letter):
Income Statement/Statement of Comprehensive Income:
Revenue P (w) million
Cost of construction __ 35 million
Gross profit P (x) million

Balance Sheet/Statement of Financial Position:


Accounts receivable from construction billings P 14 million
Construction in progress P 50 million
Less: Billings on construction __ (y)million
Net billings in excess of construction in
progress P (z)million

Statement of Cash Flow:


Cash collections P 46 million

Compute the following:


1. Total revenue recognized during 2022 (w):
B a. P 35 million
b. P 50 million
c. P100 million
d. P150 million

2. Gross profit recognized during 2022 (x):


A a. P 15 million
b. P 35 million
c. P 46 million
d. P 50 million

3. Billings on construction (y):


D a. P 14 million
b. P 46 million
c. P 50 million
d. P 60 million

4. Net billings in excess of construction in progress (z):


A a. P 10 million
b. P 15 million
c. P 50 million
d. P 60 million

5. Calculate the percentage of PAC that was completed during 2019:


B a. Zero
b. 33 1/3%
c. 65 %
d. 76 1/3%

Items 6 and 7 are based on the following information:


On May 15, 2022, Atlas Sales Company received a shipment of merchandise with a selling
price of P15,000 from Philco Company. The consignment agreement provided for a sale of
merchandise with a credit with terms of 2/10 n/30. The commission of 15% was to be
based on the accounts receivable
collected by the consignee. Cash discounts taken by customers, expenses applicable to
goods on consignment and any cash advanced to the consignor were deductible from the
remittance by the consignee.

Page 1 of 22 0915-2303213  resacpareview@gmail.com


ADVANCED FINANCIAL ACCOUNTING & REPORTING
ReSA Batch 43 - May 2022 CPALE Batch
12 Feb 2022  6:00 PM to 9:00 PM AFAR First Pre-Board Exam

Atlas Sales Company advanced P6,000 to Philco Company upon receipt of the shipment.
Expenses of P800 were paid by Atlas. By June, 2022, 70% of the shipment had been sold,
and 80% of the resulting accounts receivable had been collected, all within the discount
period. Remittance of the amount due was made on June 30, 2022.
The consigned goods cost Philco Company P10,000 and freight charges of P120 had been
paid to ship it to Atlas Sales Company.

6. The cash remitted by Atlas Sales Company


A a. P172
b. P340
c. P2,230
d. P2,340

7. The cost of inventory on consignment amounted to:


C a. P1,500
b. P3,000
c. P3,036
d. P3,186

Use the following information for 8 to 10:


Netcom acquires all the assets of P570,000,000 and liabilities amounting to
P100,000,000 of Unicom by issuing 25,000,00 shares of no-par common stock valued at
P400,000,000 plus cash of P50,000,000 and records the acquisition as a statutory merger
acquisition. Included in the agreement is a contingency guaranteeing the former
shareholders of Unicom that Netcom's shares will be worth at least P350,000,000 after
one year. If not, Unicom will issue additional shares to bring the total value of
shares issued to P350,000,000. This contingency is valued at P20,000,000 at the date
of acquisition. At the end of the first year following the acquisition, the 25,000,000
shares of Netcom's stock held by the former shareholders of Unicom are worth P12/share.

8. The Netcom’s journal entry to initially record the acquisition.


B a. Investment in S. . . . . . . . . . . . . . . 470,000,000
Common stock . . . . . . . . . . . . . . 400,000,000
Cash . . . . . . . . . . . . . . . . . . 50,000,000
PIC – stock contingency. . . . . . . . . 20,000,000
b. Assets. . . . . . . . . . . . . . . . . . . 570,000,000
Liabilities . . . . . . . . . . . . . . 100,000,000
Common stock . . . . . . . . . . . . . . 400,000,000
Cash . . . . . . . . . . . . . . . . . . 50,000,000
PIC – stock contingency . . . . . . . . 20,000,000
c. Loss on stock contingency . . . . . . . . . 470,000,000
Common stock .. . . . . . . . . . . . . 400,000,000
Cash . . . . . . . . . . . . . . . . . . 50,000,000
PIC – stock contingency . . . . . . . . 20,000,000
d. No entry required.

9. How many additional shares must Netcom subsequently issue to the former
shareholders of Unicom?
B a. P25,000,000
b. P 4,166,667
c. 2,083,333
d. No additional shares

10. The Netcom’s journal entry to record the issuance of the additional shares in
the previous number should be:
C a. Loss on stock contingency. . . . . . . . . . 50,000,000
Common stock . . . . . . . . . . . . . . 50,000,000
b. PIC – stock contingency . . . . . . . . . . 20,000,000
Loss on stock contingency. . . . . . . . . . 30,000,000
Common stock . . . . . . . . . . . . . . 50,000,000
c. PIC – stock contingency. . . . . . . . . . . 20,000,000
PIC – others. . . . . . . . . . . . . . . . 30,000,000
Common Stock . . . . . . . . . . . . . . 50,000,000
d. No entry required.

Page 2 of 22 0915-2303213  resacpareview@gmail.com


ADVANCED FINANCIAL ACCOUNTING & REPORTING
ReSA Batch 43 - May 2022 CPALE Batch
12 Feb 2022  6:00 PM to 9:00 PM AFAR First Pre-Board Exam
11. The partnership of Dana, Elsie, Fe, and Gloria is being liquidated over the first
few months of 2022. The trial balance at January 1, 2022 is as follows:

Debits Credits
Cash P 200,000
Accounts receivable 56,000
Inventory 142,000
Equipment – net 300,000
Land 150,000
Loan to Dana 20,000
Accounts payable P 400,000
Dana, capital – 20% 170,000
Elsie, capital – 10% 80,000
Fe, capital – 50% 140,000
Gloria, capital – 20% _________ ___78,000
P 868,000 P 868,000

Additional information:
1. The partners agree to retain P20,000 cash on hand for contingencies and
distribute the rest of the available cash at the end of each month.
2. In January, half of the receivables were collected. Inventory that cost
P75,000 was liquidated for P45,000. The land was sold for P250,000.
3. The accounts payable was liquidated.

How much will each partner receive for the month of January 2022?
C a. Dana, P68,000; Elsie, P39,000; Fe, P -0-; Gloria, P -0-
b. Dana, P81,000; Elsie, P45,500; Fe, P -0-; Gloria, P 9,000
c. Dana, P65,333; Elsie, P37,667; Fe, P -0-; Gloria, P -0-
d. Dana, P103,000; Elsie, P -0-; Fe, P -0-; Gloria, P -0-

12. On July 1, 2022, Sayonara Company has the following balance sheet:
Assets Liabilities and Capital
Cash……………………………… P 20,400 Accounts Payable………………………………… P 38,400
Other Assets………… 219,600 Due to Palmer………………………………………… 14,400
Other liabilities……………………………… 84,000
Palmer, capital–50%……………………… 28,800
Larsen – 50%…………………………………………… 74,400
As of July 1, 2022, the partners have personal net worth as follows:
Palmer Larsen
Assets………………………………………… P 62,400 P 91,200
Liabilities…………………………… 56,400 122,400

The personal net worth of each partner does not include amounts due to or from the
partnership. Assume the other assets are sold for P123,600 after incurring
liquidation expenses of P4,800. How much should Larsen receive?
B a. P -0-
b. 22,800
c. 24,000
d. 16,800

13. Lipton Company had an agency in Antipolo. For the period just ended, the agency
transactions showed the following:
Receipt from sales . . . . . . . . . . . . . . . P350,000
Disbursements:
Purchases . . . . . . . . . . . . . . . . . 400,000
Salaries and commissions . . . . . . . . . . 70,000
Rent . . . . . . . . . . . . . . . . . . . . 20,000
Advertising supplies . . . . . . . . . . . . 10,000
Other expenses . . . . . . . . . . . . . . . 5,000
The agency had P 100,000 receivables and P 50,000 payables as of the end of the
period. Also, there were inventories on hand of P 90,000 and unused advertising
supplies of P 6,000. The agency was set up as an experiment for one period and
would be closed if losses were incurred. The agency should:

Page 3 of 22 0915-2303213  resacpareview@gmail.com


ADVANCED FINANCIAL ACCOUNTING & REPORTING
ReSA Batch 43 - May 2022 CPALE Batch
12 Feb 2022  6:00 PM to 9:00 PM AFAR First Pre-Board Exam
C a. Review again because it was a break even operation.
b. Close with the period’s operational loss of P 155,000.
c. Close with the period’s operational loss of P 9,000.
d. Continue with the period’s profit of P 25,000.
14. Amounts related to the statement of affairs of Windup Company, in bankruptcy
liquidation as of April 1, 2022, were as follows:
Assets pledged for fully secured liabilities……………………………P 80,000
Assets pledged for partially secured liabilities………………… 50,000
Free assets…………………………………………………………………………………………………………………… 272,000
Fully secured liabilities……………………………………………………………………………… 60,000
Partially secured liabilities…………………………………………………………………… 80,000
Unsecured liabilities with priority………………………………………………… 40,000
Unsecured liabilities without priority…………………………………………… 330,000
Compute the total estimated deficiency to unsecured creditors:
C a. P78,000
b. P90,000
c. P108,000
d. P158,000
15. Using the same information in No. 14, compute the cost per peso that unsecured
creditors may expect to receive from Windup Company:
B a. P.61
b. P.70
c. P.76
d. P.81
Items 16 through 18 are based on the following information:
Rome Corporation has one branch office, named Timber Branch. Rome is performing the
end-of-the-period reconciliation. The following items are unsettled at the end of the
accounting period (you may assume that the item has been reflected in the accounts of
the underlined entity):
(1) Rome has agreed to remove P750 of excess freight charges charged to Timber when
Rome shipped twice as much inventory as Timber requested.
(2) Timber mailed a check for P11,000 to Rome as a payment for merchandise shipped
from Rome to Timber. Rome has not yet received the check.
(3) Timber returned defective merchandise to Rome. The merchandise was billed to
Timber at P4,000 when its actual cost was P3,000.
(4) Advertising expenses attributable to the branch office was paid for by the home
office in the amount of P5,000.
16. Which of the following statements is correct:
D a. The Home Office account in Timber’s books is decreased for the P11,000
of cash in transit and is decreased for the P750 of excess freight
charges.
b. The Home Office account in Timber’s books is decreased for the P5,000
of advertising expense and decreased for the P750 of excess freight
charges.
c. The Home Office account in Timber’s books is increased for the P11,000
of cash payment to Rome and decreased by P4,000 for the billed cost
of the defective merchandise inventory.
d. The Home Office account in Timber’s books is decreased for the P750
of excessive freight charges and increased by the P5,000 of the
advertising expenditure.
17. Which of the following statements is correct:
C a. The Timber Branch account on Rome’s books is decreased for the P11,000
of cash in transit and is decreased for the P750 of excess freight
charges.
b. The Timber Branch account on Rome’s books is decreased for the P11,000
of cash in transit and is decreased for the P750 of excess freight
charges.
c. The Timber Branch account on Rome’s books is decreased for the P11,000
of cash in transit and is decreased for the P4,000 billed cost of the
defective merchandise.
d. The Timber Branch account on Rome’s books is increased for the P11,000
of cash in transit and is decreased for the P5,000 of allocated
advertising costs

Page 4 of 22 0915-2303213  resacpareview@gmail.com


ADVANCED FINANCIAL ACCOUNTING & REPORTING
ReSA Batch 43 - May 2022 CPALE Batch
12 Feb 2022  6:00 PM to 9:00 PM AFAR First Pre-Board Exam
18. If the adjusted balances for the Timber Branch Account and the Rome Home Office
accounts is P500,000, what unadjusted balance was listed in Rome’s Timber Branch
Account?
A a. P515,000
b. P510,250
c. P514,000
d. P504,000

19. Partners DD, EE, FF, and GG share profits 50%, 30%, 10% and 10%, respectively.
Accounts maintained with partners just prior to liquidation were as follows:

Advances Loans Capitals


(Dr. Balances) (Cr. Balances) (Cr. Balances)
DD P 5,000 P 40,000
EE 10,000 30,000
FF P 4,500 15,000
GG 2,500 25,000

At this point, cash of P18,000 is available for distribution to the partners.


How much of the P18,000 cash should be distributed to each partner?
D a. DD, P9,000; EE, P 5,400; FF, P1,800; GG, P 1,800
b. DD, P -0- ; EE, P18,000; FF, -0- ; GG, P -0-
c. DD, P -0- ; EE, -0-; FF, -0- ; GG, P 18,000
d. DD, P -0- ; EE, P 6,625; FF, -0- ; GG, P 11,375

20. The partnership of Monte and Carlo has the following provisions:
1. Monte, who is primarily responsible for obtaining new clients, is to receive
a 30% bonus on revenues in excess of P200,000.
2. Carlo, who is primarily responsible for administration, is to receive a 30%
bonus on profits in excess of 50% of revenues, as reflected in the general
ledger.
3. All remaining profits or losses are to be divided equally.
Revenues for the year P280,000
Operating expenses 120,000

The share of partner Monte in the net income should be:


B a. P92,000
b. P89,000
c. P71,000
d. P57,000

21. Cord and Stringer are partners who share profits and losses in the ratio of 3:2,
respectively. On August 31, 2022 their capital accounts are as follows:

Cord P70,000
Stringer 60,000

On that date, they agree to admit Twiner as a partner with a one-third interest
in the capital and profits and losses, for an investment of P50,000. The new
partnership will begin with a total capital of P180,000.

The capital balance of Cord after the admission of Twiner should be:
B a. P56,000
b. P64,000
c. P70,000
d. P76,000

22. On June 30, 2022, the balance sheet of the Oakley, Pine, and Woods partnership,
together with their respective profits and loss ratios was as follows:

Assets, at cost P180,000


Oakley, loan 9,000
Capital, Oakley (20%) 42,000
Capital, Pine (20%) 39,000
Capital, Woods (60%) 90,000

Page 5 of 22 0915-2303213  resacpareview@gmail.com


ADVANCED FINANCIAL ACCOUNTING & REPORTING
ReSA Batch 43 - May 2022 CPALE Batch
12 Feb 2022  6:00 PM to 9:00 PM AFAR First Pre-Board Exam
Oakley has decided to retire from the partnership. By mutual agreement, the
assets are to be adjusted to their current value of P216,000 and the partnership
is to pay Oakley P61,200 for her partnership interest, including her loan, which
is to be repaid in full.
Assuming that goodwill is not to be recorded on the partnership’s books, the
capital balance of Pine after the retirement of Oakley should be:
C a. P36,750
b. P38,250
c. P45,450
d. P49,200

Use the following for items 23, 24, and 25:


At the end of its fiscal year on June 30, 2022, the Ritz, Sally, and Tracy Partnership
had account balances as follows:
Cash P 20,000 Accounts payable P 35,000
Accounts receivable 30,000 Loan from Sally 25,000
Inventories 70,000 Ritz, capital (20%) 70,000
Plant assets, net 60,000 Sally, capital (30%) 50,000
Loan to Ritz __30,000 Tracy, capital (50%) __30,000
Total Assets P210,000 Total Liab. & Equity P210,000
The percentages shown are the residual profit sharing ratios. Tracy also gets a P12,000
annual salary allowance. The partners dissolved the partnership on July 1, 2022 and
began the liquidation process. During July the following events occurred:
• Receivables of P15,000 were collected.
• The inventory was sold for P20,000.
• All available cash was distributed on July 31, except for P10,000 of expected
expenses.

23. The book value of the partnership equity/interest (i.e., total equity/interest
of the partners) on June 30, 2022 is:
C a. P210,000
b. P150,000
c. P145,000
d. P120,000

24. The cash available for distribution to partners on July 31, 2022 is:
D a. P55,000
b. P35,000
c. P20,000
d. P10,000

25. Without bias on your part, assume that the cash available for distribution to
partners on July 31, 2022 is P10,000. Under this assumption Sally should receive:
A a. P10,000
b. P 6,000
c. P 3,000
d. Amount cannot be determined

26. Ann, Bee, and Kay are in the process of liquidating their partnership. Kay has
agreed to accept the inventories as part of her settlement. The inventories have
a fair value of P60,000 and a book value of P80,000. Account balances and profit
and loss sharing ratios are summarized as follows:
Cash P 198,000 Accounts payable P 149,000
Inventories 80,000 Ann, capital (40%) 79,000
Plant assets, net 230,000 Bee, capital (40%) 140,000
_________ Kay, capital (20%) 140,000
Total Assets P 508,000 Total Liab. & Equity P 508,000

If the partners agree to distribute the available cash:


A a. Kay will receive P23,000 of the cash distribution.
b. Bee will receive P40,667 of the cash distribution.
c. Immediately after the cash distribution of cash and inventory items,
Kay’s capital account balance will be P59,000.
d. Immediately after the cash distribution of cash and inventory items,
Kay’s capital account balance will be P30,000.
Page 6 of 22 0915-2303213  resacpareview@gmail.com
ADVANCED FINANCIAL ACCOUNTING & REPORTING
ReSA Batch 43 - May 2022 CPALE Batch
12 Feb 2022  6:00 PM to 9:00 PM AFAR First Pre-Board Exam

27. Which of the following procedures are acceptable for dealing with the negative
balance in a partner’s capital account during liquidation:
D a. The partner with the negative capital balance can contribute assets
to the partnership sufficient to bring the capital account up to
zero.
b. If the partner with the negative capital balance is personally
insolvent; the negative capital balance may be absorbed by those
partners having a positive capital balance according to the profit
and loss sharing ratios applying to all the partners.
c. If the partner with the negative capital balance is personally
insolvent, the negative capital balance may be absorbed by those
partners having a positive capital according to the profit and loss
sharing ratios applying to those partners having positive balances.
d. A and C are acceptable choices.
e. A, B, and C are acceptable.

28. What is the difference between the terms “the partners’ equity (or interest)
balances” and the partners’ capital account balances?”
D a. There is no difference in the terms; they can be used interchangeably.
b. The term “partners’ equity (or interest)” is the sum of the individual
partner’s capital accounts.
c. The term “partners’ equity (or interest)” is the sum of the individual
partner’s capital accounts as increased by partnership loans made to
the partner and reduced by a partner’s loans to the partnership.
d. The term “partners’ equity (or interest)” is the sum of the individual
partner’s capital accounts as decreased by partnership loans made to
the partner and increased by a partner’s loans to the partnership.

29. Patterson Company acquiring the net assets of Sheila Company by issuing 100,000
of its P1 par value shares of common stock. The shares have fair value of P15
each. Just prior to the acquisition, Sheila’s balance sheet was as follows:
Sheila Company
Balance Sheet
January 1, 2022

Assets Liabilities and Equity


Accounts receivable P100,000 Current liabilities . . P80,000
Inventory . . . . . 210,000 Bonds payable . . . . . 200,000 P280,000
Equipment (net) . . 100,000 Stockholders' equity:
Land . . . . . . . 200,000 Common stock (P1 par) P10,000
Building (net) . . 300,000 Retained earnings . . 620,000 630,000
Total liabilities and
Total assets . . . P910,000 equity . . . . . P910,000
Fair values agree with book value except for the building, which is appraised at
P450,000. The following additional information is available:
• The equipment will be sold for an estimated price of P80,000. A 10% commission
will be paid to a broker.
• A major R&D project is underway. The accumulated costs are P56,000, and the
estimated value of the work is P90,000.
• A warranty attaches to products sold in the past. The estimated future repair
costs under the warranty are P30,000.
• Sheila has a customer list that has value. It is estimated that the asset such
as this is valued at a 20% rate of return. The present value amounted to P210,650.

Determine the excess of total cost over fair value of net assets acquired (or
goodwill):
C a. P -0-
b. P469,350
c. P477,350
d. P519,350

Page 7 of 22 0915-2303213  resacpareview@gmail.com


ADVANCED FINANCIAL ACCOUNTING & REPORTING
ReSA Batch 43 - May 2022 CPALE Batch
12 Feb 2022  6:00 PM to 9:00 PM AFAR First Pre-Board Exam
30. Norton Corporation has agreed to acquire the net assets of Payco Corporation.
Just prior to the acquisition, Payco’s balance sheet was as follows:

Assets Liabilities and Equity


Accounts receivable P200,000 Current liabilities . P80,000
Inventory . . . . . 270,000 Bonds payable. . . . . 250,000 P330,000
Equipment (net) . . 100,000 Stockholders' equity:
Common stock (P1
par). . . . . P100,000
Retained earnings . . 140,000 240,000
Total liabilities and
Total assets . . . P570,000 equity. . . . . P570,000

Fair values agree with book values except for the equipment, which has an estimated
fair value of P40,000. Also, it has been determined that brand-name copyrights have
an estimated value of P15,000. Norton Corporation paid P25,000 in acquisition costs
to consummate the transaction.

The cash paid to acquire Payco Corporation was P160,000.

The gain on acquisition amounted to:


A a. P35,000
b. P20,000
c. P10,000
d. No gain

31. Trial balances for the home office and the branch of the Tony Co. show the
following accounts before adjustment, on December 31, 2022. The home office
policy of billing the branch for merchandise is 20% above cost.
Home
office Branch
Unrealized intercompany inventory profit P 10,800
Shipments to branch 24,000
Purchase (outsiders) P 7,500
Shipments from home office 28,800
Merchandise inventory, December 1, 2021 45,000

What part of the branch inventory as of December 1, 2022 represent purchases from
outsiders and what part represents goods acquired from the home office?
Outsiders Home Office
D a. P 12,000 P33,000
b. 16,500 28,500
c. 15,000 30,000
d. 9,000 36,000

Use the following information for 32 and 33:


Ping Company acquires all of Sun Corp. in an asset acquisition. Ping paid P1,000,000
more than Sun's book value, and this excess was attributed entirely to goodwill, as
all of Sun's assets and liabilities were carried at amounts equivalent to fair value.
At the time of the combination, a lawsuit was pending against Sun, which Sun had not
recorded on its books. It was felt at the time that Sun would win the lawsuit, so no
provision for it was made when Ping recorded the asset acquisition.

32. Six months after the acquisition, new information reveals that the expected
value of the lawsuit at the date of acquisition was P400,000. The appropriate
entry on Ping's books to record this new information.
C a. Retained earnings. . . . . . . . . . . . . . 400,000
Estimated lawsuit liability. . . . . . . 400,000
b. Loss on lawsuit. . . . . . . . . . . . . . . 400,000
Estimated lawsuit liability. . . . . . . 400,000
c. Goodwill. . . . . . . . . . . . . . . . . . 400,000
Estimated lawsuit liability. . . . . . . 400,000
d. No entry required.

Page 8 of 22 0915-2303213  resacpareview@gmail.com


ADVANCED FINANCIAL ACCOUNTING & REPORTING
ReSA Batch 43 - May 2022 CPALE Batch
12 Feb 2022  6:00 PM to 9:00 PM AFAR First Pre-Board Exam
33. Assume the same information as above, except that the value change is a result
of events occurring subsequent to acquisition. The appropriate entry on Ping's
books to record the new information.
B a. Retained earnings. . . . . . . . . . . . . . 400,000
Estimated lawsuit liability. . . . . . . 400,000
b. Loss on lawsuit. . . . . . . . . . . . . . . 400,000
Estimated lawsuit liability. . . . . . . 400,000
c. Goodwill. . . . . . . . . . . . . . . . . . 400,000
Estimated lawsuit liability. . . . . . . 400,000
d. No entry required.
Items 34 through 36 are based on the following information:
DJ pays P5,000,000 in cash and issues 50,000 shares of stock with a par value of
P10/share and fair value of P40/share to acquire Builder’s assets and liabilities on
January 1, 2022. Balance sheets just prior to the acquisition are as follows:
DJ Builders
Book value Book value Fair value
Current assets P 7,000,000 P 1,000,000 P2,100,000
Property, plant & equipment, net 55,000,000 5,000,000 3,000,000
Identifiable intangible assets ____400,000 __2,000,000 7,000,000
Total assets P62,400,000 P 8,000,000

Current liabilities P 6,500,000 P 800,000 P1,000,000


Long-term debt 30,000,000 6,000,000 5,800,000
Common stock, par value 200,000 700,000
Additional paid-in capital 22,000,000 1,800,000
Retained earnings 4,000,000 4,000,000
Accumulated other comprehensive income 100,000 ( 500,000)
Treasury stock (_400,000) (4,800,000)
Total liabilities & equity P62,400,000 P 8,000,000
DJ’s consultants find these items that are not reported on Builder’s balance sheet:
Fair value
Potential contracts with new customers P 300,000
Advanced production technology 170,000
Future cost savings 100,000
Contractual obligations – long-term warranties 280,000
Non-competition agreements 70,000
Customer relationships – not contractual 100,000
Customer contract 50,000
Recent favorable press reports on a consulting firm 30,000
Long-time customer relationships 40,000
Outside consultants are paid P400,000 in cash, and registration fees to issue the new
stock are P200,000. To sweeten the deal, DJ agrees to pay the former shareholders of
Builders an additional cash amount of P4,000,000 million at the end of 2023 that
depends on the 2022-23 reported net income of Builders. DJ believes there is a 25%
chance that this payment will have to be made, and the appropriate discount rate for
the earn-out is 20% (present value factor of P1 @ 20% = .69444).

34. The total assets after the acquisition:


C a. P70,400,000
b. P71,284,440
c. P71,574,440
d. None of the above

35. The total liabilities after the acquisition:


B a. P43,300,000
b. P44,274,440
c. P44,554,440
d. None of the above
36. The stockholders’/shareholders/equity holders of DJ after the acquisition:
B a. P25,900,000
b. P27,300,000
c. P27,900,000
d. None of the above

Page 9 of 22 0915-2303213  resacpareview@gmail.com


ADVANCED FINANCIAL ACCOUNTING & REPORTING
ReSA Batch 43 - May 2022 CPALE Batch
12 Feb 2022  6:00 PM to 9:00 PM AFAR First Pre-Board Exam
37. Treadway Corporation acquires Hooker, Inc. The parent pays more for it than the
fair value of the subsidiary’s net assets. On the acquisition date, Treadway has
equipment with a book value of P420,000 and a fair value of P530,000. Hooker has
equipment with a book value of P330,000 and a fair value of P390,000. Hooker is
going to use push-down accounting. Immediately after the acquisition, what
amounts in the Equipment account appear on Hooker’s separate balance sheet and
on the consolidated balance sheet?
C a. P330,000 and P750,000
b. P330,000 and P860,000
c. P390,000 and P810,000
d. P390,000 and P920,000
Use the following information for questions 38 to 42:
On January 1, 2022, Park Corporation and Strand Corporation and their condensed balance
sheet are as follows:
Park Strand
Corp. Corp.
Current Assets . . . . . . . . . . . . . .P . 70,000 P 20,000
Non-current Assets . . . . . . . . . . . . . 90,000 40,000
Total Assets . . . . . . . . . . . . . . .P .160,000 P 60,000

Current Liabilities . . . . . . . . . . . P. 30,000 P 10,000


Long-term Debt . . . . . . . . . . . . . . . 50,000 -
Stockholders’ Equity . . . . . . . . . . . . 80,000 50,000
Total Liabilities and Equities . . . . . .P .160,000 P 60,000

On January 2, 2022, Park Corporation borrowed P60,000 and used the proceeds to obtain
80% of the outstanding common shares of Strand Corporation. The P60,000 debt is payable
in 10 equal annual principal payments, plus interest, beginning December 31, 2022. The
excess fair value of the investment over the underlying book value of the acquired net
assets is allocated to inventory (60%) and to goodwill (40%).
On a consolidated balance sheet as of January 2, 2022, what should be the amount for
each of the following?

38. The amount of goodwill using (1) proportionate basis (partial) and (2) full
fair value (full/gross-up) basis:
B a. (1) P 0; (2) P0
b. (1) P 8,000; (2) P10,000
c. (1) P10,000; (2) P 8,000
d. (1) P 0; (2) P20,000

39. Assuming NCI is measured at fair value, total Current assets should be:
A a. P105,000
b. P102,000
c. P100,000
d. P 90,000

40. Non-current asset using (1) proportionate basis (partial) and (2) full fair
value basis (full/gross-up) in computing goodwill should be:
C a. (1) P130,000; (2) P130,000
b. (1) P138,000; (2) P138,000
c. (1) P138,000; (2) P140,000
d. (1) P140,000; (2) P140,000

41. (1) Current liabilities and (2) non-current liabilities should be:
B a. (1) P50,000; (2) P110,000
b. (1) P46,000; (2) P104,000
c. (1) P40,000; (2) P104,000
d. (1) P46,000; (2) P 90,000

42. Stockholders’ equity using (1) proportionate basis (partial) and (2) full fair
value basis (full/gross-up) basis of determine non-controlling interest should
be:
B a. (1) P80,000; (2) P 95,000
b. (1) P93,000; (2) P 95,000
c. (1) P93,000; (2) P93,000
d. (1) P95,000; (2) P95,000
Page 10 of 22 0915-2303213  resacpareview@gmail.com
ADVANCED FINANCIAL ACCOUNTING & REPORTING
ReSA Batch 43 - May 2022 CPALE Batch
12 Feb 2022  6:00 PM to 9:00 PM AFAR First Pre-Board Exam

43. Anselmo Company operates retail hobby shops from the main store and a branch
store. Merchandise is shipped from the main store and to the branch and billed
to the branch at an arbitrary 10% markup. Trial balances of the main store and
branch as of December 31, 2022 are as follows:
Main Store Branch
Debits:
Cash . . . . . . . . . . . . . . . . . . . .P . 1,500 P 1,000
Accounts receivable – net . . . . . . . . . . 200 -
Inventory, December 31, 2022 . . . . . . . . . 3,500 2,500
Building – net . . . . . . . . . . . . . . . . 60,000 18,000
Equipment – net . . . . . . . . . . . . . . . 30,000 12,000
Branch store . . . . . . . . . . . . . . . . . 32,300 -
Purchases . . . . . . . . . . . . . . . . . . 240,000 11,000
Shipments from home office . . . . . . . . . . - 99,000
Other expenses . . . . . . . . . . . . . . . . 15,000 7,000
Total debits . . . . . . . . . . . . . . . . P. 382,500 P 150,500

Credits:
Accounts payable . . . . . . . . . . . .P 15,000 P 500
Unrealized inventory profit . . . . . . . 9,200 -
Main Store . . . . . . . . . . . . . . . - 30,000
Capital stock . . . . . . . . . . . . . . 50,000 -
Retained earnings . . . . . . . . . . . . 16,000 -
Sales . . . . . . . . . . . . . . . . . . 200,000 120,000
Shipments to branch . . . . . . . . . . . 90,000 -
Profit from branch . . . . . . . . . . . ____2,300 _________
Total credits . . . . . . . . . . . . . .P 382,500 P 150,500

Inventories on hand at December 31, 2022 at the main store and branch are P3,000
and P1,800, respectively. The December 31, 2021 branch inventory includes
merchandise purchased from outsiders of P300, and the December 31, 2022 branch
inventory includes P150 of merchandise purchased from outsiders. The combined
cost of goods sold amounted to:
D a. P 261,200
b. P 252,200
c. P 243,150
d. P 252,150

Items 44 and 45 are based on the following information:


On January 1, 2022, Lesley Benjamin signed an agreement (covering 5 years) to operate
as a franchisee of Campbell Inc. for an initial franchise fee of P50,000. The amount
of P10,000 was paid when the agreement was signed, and the balance is payable in five
annual payments of P8,000 each, beginning January 1, 2023. The agreement provides that
the down payment is non-refundable and that no future services are required of the
franchisor once the franchise commences operations on April 1, 2022. Lesley Benjamin’s
credit rating indicates that she can borrow money at 11% for a loan of this type.
For Campbell’s 2022-related revenue for this franchise arrangement, assuming that in
addition to the franchise rights, Campbell also provides 1 year of operational
consulting and training services, beginning on the signing date. These services have
a value of P3,600.

44. The amount of franchise revenue on January 1, 2022 assuming that Campbell must
provide services to Benjamin throughout the franchise period to maintain the
franchise value.
A a. Zero.
b. P10,433
c. P39,567
d. P50,000

45. In relation to No. 44, the amount of franchise revenue on December 31, 2022:
B a. Zero.
b. P7,913
c. P39,567
d. P50,000

Page 11 of 22 0915-2303213  resacpareview@gmail.com


ADVANCED FINANCIAL ACCOUNTING & REPORTING
ReSA Batch 43 - May 2022 CPALE Batch
12 Feb 2022  6:00 PM to 9:00 PM AFAR First Pre-Board Exam
46. On March 1, 2022, Giordano Company enters into a contract to transfer a product
to Hotter on July 31, 2022. The contract is structured such that Warmer is
required to pay the full contract price of P57,000 on August 31, 2022. The cost
of the goods transferred is P34,200. Giordano delivers the product to Hotter on
July 31, 2022. The contract exists on:
B a. March 1, 2022
b. July 31, 2022
c. August 31, 2022
d. Incomplete data

47. Meyer & Smith is a full-service technology company. They provide equipment, and
installation services as well as training. Customers can purchase any product or
service separately or as a bundled package. Container Corporation purchased
computer equipment, installation and training for a total cost of P120,000 on
March 15, 2022. Estimated standalone fair values of the equipment, installation,
and training are P75,000, P50,000, and P25,000 respectively. The transaction
price allocated to equipment, installation and training:
D a. P75,000, P50,000, P25,000 respectively
b. P40,000, P40,000, P40,000 respectively
c. P120,000 for the entire bundle
d. P60,000, P40,000 and P20,000 respectively

48. On July 31, O’Malley Company contracted to have two products built by Taylor
Manufacturing for a total of P185,000. The contract specifies that payment will
only occur after both products have been transferred to O’Malley Company.
O’Malley determines that the standalone prices are P100,000 for Product 1 and
P85,000 for Product 2. On August 1, when Product 1 has been transferred, the
journal entry to record this event include a:
D a. debit to Accounts Receivable for P100,000
b. debit to Accounts Receivable for P85,000
c. debit to Contract Assets for P85,000
d. debit to Contract Assets for P100,000

49. On January 1, Joey enters into a contract with Althea for the sale of an excavator
with unique specifications. Joey and Althea develop the specifications and Joey
contracts with a construction equipment manufacturer to produce the equipment.
The manufacturer will deliver the equipment to Althea when it is completed Joey
agrees to pay the manufacturer P42,000,000 upon delivery of the excavator to
Althea. Anderson and Althea agree to a selling price of P46,200,000 that will be
paid by Althea to Joey. Joey’s profit is P4,200,000 Joey’s contract with Althea
requires Althea to seek remedies for defects from the manufacturer, but Joey is
responsible for any corrections due to errors in specifications. The role of
Joey is a:
B a. Customer
b. Principal
c. Agent
d. No agreement at all
50. Maybelle Paulino Computers manufactures and sells computers that include a
warranty to make good on any defect in its computers for 150 days (often referred
to as an assurance warranty). In addition, it sells separately an extended
warranty, which provides protection from defects for three years beyond the 150
days (often referred to as a service warranty). How many performance obligations
are in the contract?
C a. 0
b. 1
c. 2
d. 3

Use the following information for questions 51 and 52: Variable Consideration
Billy Biotech enters into a licensing agreement with Paul Pharmaceutical for a drug
under development. Billy will receive a payment of P20,000,000 if the drug receives a
regulatory approval. Based on prior experience in the drug-approval process, Billy
determines it is 90% likely that the drug will gain approval and a 10% chance of
denial. Assuming that regulatory approval was granted on December 20, 2022, and that
Billy received the payment from Paul on January 15, 2023.

Page 12 of 22 0915-2303213  resacpareview@gmail.com


ADVANCED FINANCIAL ACCOUNTING & REPORTING
ReSA Batch 43 - May 2022 CPALE Batch
12 Feb 2022  6:00 PM to 9:00 PM AFAR First Pre-Board Exam
51. Determine the transaction price of the arrangement for Billy Biotech:
C a. Nil
b. P18,000,000
c. P20,000,000
d. No transaction at all
52. On December 20, 2022, license revenue amounted to
A a. Nil
b. P18,000,000
c. P20,000,000
d. No transaction at all
53. Horizontal business combinations occur when one entity purchases which of the
following?
C a. A supplier
b. A customer
c. A competitor
d. None of the above

54. Under PFRS 3:


B a. both direct and indirect costs are to be capitalized.
b. both direct and indirect costs are to be expensed.
c. direct costs are to be capitalized and indirect costs are to be
expensed.
d. indirect costs are to be capitalized and direct costs are to be
expensed.

55. A business combination is accounted for properly as an acquisition. Which of


the following expenses related to effecting the business combination should enter
into the determination of net income of the combined corporation for the period
in which the expenses are incurred?
Security issue costs Overhead allocated to the merger
C a. Yes Yes
b. Yes No
c. No Yes
d. No No

56. Parental Company and Sub Company were combined in an acquisition transaction.
Parental was able to acquire Sub at a bargain price. The sum of the fair values
of identifiable assets acquired less the fair value of liabilities assumed
exceeded the cost to Parental. After eliminating previously recorded goodwill,
there was still some "negative goodwill." Proper accounting treatment by
Parental is to report the amount as
C a. paid-in capital.
b. a deferred credit, which is amortized.
c. an ordinary gain.
d. an extraordinary gain.
57. What is the theoretically preferred method of presenting a non-controlling
interest in a consolidated balance sheet?
D a. As a separate item within the liability section.
b. As a deduction from (contra to) goodwill from consolidation, if any.
c. By means of notes or footnotes to the balance sheet.
d. As a separate item within the stockholders’ equity section.

58. The investment in a subsidiary should be recorded on the parent's books at the
C a. underlying book value of the subsidiary’s net assets.
b. fair value of the subsidiary’s net identifiable assets.
c. fair value of the consideration given.
d. fair value of the consideration given plus an estimated value for
goodwill.

59. The second step in the process for revenue recognition is to


D a. allocate transaction price to the separate performance obligations.
b. determine the transaction price.
c. identify the contract with customers.
d. identify the separate performance obligations in the contract.

Page 13 of 22 0915-2303213  resacpareview@gmail.com


ADVANCED FINANCIAL ACCOUNTING & REPORTING
ReSA Batch 43 - May 2022 CPALE Batch
12 Feb 2022  6:00 PM to 9:00 PM AFAR First Pre-Board Exam
60. Partial satisfaction of a multiple performance obligation is reported on the
statement of financial position as
C a. contract liability.
b. receivable.
c. contract asset.
d. unearned service revenue.

61. Contract liability is a company’s obligations to transfer goods or services to


a customer for which the company has received consideration from the customer.
An example of a contract liability is
B a. Prepaid subscription.
b. Unearned magazine subscription.
c. Mortgage Payable.
d. Service Revenue.

62. If both the home office and the branch of a business enterprise use the perpetual
inventory system, a Shipment to Branch ledger account appears in the accounting
records of:
D a. The home office only
b. The branch only
c. Both the home office and the branch
d. Neither the home office nor the branch

63. In preparing the financial statements of the home office and its various branches:
D a. Nonreciprocal accounts are eliminated but reciprocal accounts are
combined
b. Both reciprocal and nonreciprocal accounts are eliminated
c. Both reciprocal and nonreciprocal accounts are combined
d. Reciprocal accounts are eliminated and nonreciprocal accounts are
combined

64. In the year end general ledger closing procedures,which accounts are closed in
arriving at Cost of Sales?
Purchases Sent to Branch Purchases from Home Office
A a. Yes Yes
b. No Yes
c. No No
d. Yes NO

65. How are anticipated administrative expenses reported on a statement of financial


affairs?
B a. As a footnote until actually incurred.
b. As a liability with priority
c. As a partially secured liability.
d. As an unsecured liability

66. The Statement of Realization and Liquidation differs from the Statement of
Affairs because
D a. The Statement of Realization and Affairs reports estimated realizable
values rather than actual liquidation results
b. The Statement of Realization and Affairs is a summary of secured debt
activity only
c. The Statement of Realization and Affairs is prepared only at final
completion of the liquidation process
d. The Statement of Realization and Affairs reports actual liquidation
results rather than estimated realizable values

67. Which of the following are recognized each period under the cost-recovery (point-
in-time) method?
C a. Costs only.
b. Revenues only.
c. Both costs and revenues.
d. None of these.

Page 14 of 22 0915-2303213  resacpareview@gmail.com


ADVANCED FINANCIAL ACCOUNTING & REPORTING
ReSA Batch 43 - May 2022 CPALE Batch
12 Feb 2022  6:00 PM to 9:00 PM AFAR First Pre-Board Exam
68. If the percentage-of-completion (overtime) method is used, what is the basis for
determining the gross profit to be recognized in the second year of a three-year
contract?
C a. Cumulative actual costs incurred only.
b. Incremental cost for the second year only.
c. Cumulative actual costs and estimated costs to complete.
d. No gross profit would be recognized in year 2.

69. All revenue for franchise companies is derived from


C a. assistance for site selection and negotiating lease.
b. bookkeeping and advisory services.
c. sale of initial franchise and continuing fees.
d. advertising and promotion.

70. Goods on consignment should be included in the inventory of:


A a. the consignor but not the consignee
b. both the consignor and the consignee
c. the consignee but not the consignor
d. neither the consignor nor the consignee

- END –
ANSWERS & SOLUTIONS/CLARIFICATIONS
1. Total revenue recognized during 2019 (w): P 50 million
CIP contains cost + gross profit* = revenue, so W = P50
* Note that the Income statement is in gross profit position, therefore, entries
recorded under CIP account pertain to both actual costs incurred and the RGP
(alternatively, the amount pertains to the amount debited to AR)
2. Gross profit recognized during 2022 (x): P50M Revenue - P35M cost = P15M GP P 15 million
3. Billings on construction (y):
AR billed ending balance of P14M + AR billed collected in 2022 P46M = P 60 million
P60M Recorded AR billed for 2022
4. Net billings in excess of construction in progress (z): P10 million
Billings of P60M – CIP of P50M (alternatively, this pertains to the net billed AR
(i.e., billed AR is higher than the amount of revenue recorded as unbilled AR)
5. Calculate the percentage of PAC that was completed during 2022: 33.33%
P50M revenue recognized/P150M Contract Price = 33.33%

6. Gross collection (P15,000 x 70% x 80%) P 8,400


Less: Cash discount taken by customer __168
(P8,400 x 2%)
Net collection P 8,232
Less Charges:
Expenses P 800
Commission (P8,400 x 15%) _1,260 __2,060
Due to Consignor P 6,172
Less: Advances _6,000
Amount remitted P 172

7. Charges Related to
Total Consignment Inventory on
Charges Sales Consignment
(100%) (70%) (30%)
Consignor’s charges:
Cost P10,000 P 7,000 P 3,000
Freight 120 84 36
Consignee’s charges:
Expenses 800 800
Commission (15% x 1,575 1,575
P10,500)
Cash discount (P10,500 x 168 168 ______
80% x 2%)
Total P12,663 P 9,627 P 3,036
Sales price (70% x P15,000) _10,500_
Profit on Consignment P 873
Page 15 of 22 0915-2303213  resacpareview@gmail.com
ADVANCED FINANCIAL ACCOUNTING & REPORTING
ReSA Batch 43 - May 2022 CPALE Batch
12 Feb 2022  6:00 PM to 9:00 PM AFAR First Pre-Board Exam
8. Assets 570,000,000
Liabilities 100,000,000
Capital stock 400,000,000
Cash 50,000,000
PIC-stock contingency 20,000,000

9. P350,000,000 – (P12 x 25,000,000) = P50,000,000/P12 = 4,166,667 additional shares

10. The contingency was originally recorded in equity at the amount of P20,000,000. However, changes in the
value of stock price contingencies do not affect the acquisition price or income. Any changes in value are
adjustments in equity.
PIC- stock contingency 20,000,000
PIC-other 30,000,000
Common stock 50,000,000

11. Dana Elsie Fe Gloria Total


Capital balances P 170, 000 P 80, 000 P 140, 000 P 78,000 P 468, 000
Loan to Dana (20, 000) (20, 000)
Total interest P 150, 000 P 80, 000 P 140,000 P 78,000 P 448, 000
Total reduction in interest (69,000) (34,500) (172,500) (69,000) (345,000)
CAFD P 81,000 P 45,500 P (32,500) P 9, 000 *P 103, 000
Possible loss (2:1:2) (13, 000) (6, 500) 32, 500 (13, 000) -
P 68, 000 P 39, 000 P (4, 000) P 103, 000
Possible loss (2:1) (2, 667) (1, 333) 4, 000 -
Cash received/distributed P 65, 333 P 37,667 P 103, 000

Payment to partners:
Cash, beginning…………………………………………………………………………… P 200,000
Add: Proceeds from -
Receivables (1/2 x P56, 000)…………………………………………………… 28,000
Inventory…………………………………………………………………………………. 45,000
Land ……………………………………………………………………………………….. 250,000
Less: Cash withheld………………………………………………………………. ( 20,000)
Payment of accounts payable…………………………………………………… (400,000)
Cash available for distribution (CAFD)………………………………. P 103,000

12. Palmer Larsen Total


Due to (from) P 14,400 P -0- P 14,400
Capital balances 28,800 74,400 103,200
Total Interests P 43,200 P 74,400 P117,600
Reduction in interests ( 50,400) ( 50,400) (100,800)
Cash available for distribution (CAFD) P( 7,200) P 24,000 P 16,800*
Additional investment (P62,400 – P56,400) 6,000 -0- 6,000
Balances P ( 1,200) P 24,000 P 22,800
Additional loss 1,200 ( 1,200) -0-
Payment P -0- P 22,800 P 22,800

Cash beginning……………………………………………………………………………….. P 20,400


Add: Proceeds…………………………………………………………………………………123,600
Less: Payment of liquidation expenses………………………………………………. (4,800)
Payment of liabilities:
Accounts Payable……………………………………………………………… ( 38,400)
Other liabilities………………………………………………………………… ( 84,000)
Cash available for distribution (CAFD)………………………………P 16,800*

13. Sales (P350,000 + P100,000)……………………………………………………… P 450,000


Less: Cost of goods sold:
Purchases (P400,000 + P50,000)………………………………P 450,000
Less: Inventory, end…………………………………………………… 90,000 360,000
Gross profit………………………………………………………………………………………….P 90,000
Less: Expenses –
Salaries and commissions…………………………………………P 70,000
Rent……………………………………………………………………………………20,000
Advertising supplies (P10,000 – P6,000)……… 4,000
Other expenses…………………………………………………………… 5,000 99,000
Net Loss………………………………………………………………………………………………P( 9,000)
Page 16 of 22 0915-2303213  resacpareview@gmail.com
ADVANCED FINANCIAL ACCOUNTING & REPORTING
ReSA Batch 43 - May 2022 CPALE Batch
12 Feb 2022  6:00 PM to 9:00 PM AFAR First Pre-Board Exam
14. Estimated deficiency to unsecured creditors:
Free Assets:
Assets pledged to fully secured liabilities (P80,000 – P60,000)……P 20,000
Free Assets…………………………………………………………………………… 272,000
Total Free Assets………………………………………………………………………… …… P292,000
Less: Unsecured liabilities with priority………………………………………… …….. 40,000
Net Free Assets.…………………………………………………………………………………….P252,000
Less: Unsecured Liabilities without priority:
Partially secured liabilities (P80,000 – P50,000)……………P 30,000
Add: Unsecured liabilities without priority……………………. 330,000 360,000
Estimated deficiency to unsecured liabilities..…………………………………….P108,000
15. Expected Recovery % of Unsecured Liabilities:
Net Free Assets / Unsecured liabilities without priority
P252,000/P360,000……………………………………………………………………70% or P.70
16, 17 & 18
Investment in Home Office
Branch
Unadjusted 515,000 495,750
Excess freight (750)
Remittance (11,000)
Returns (4,000)
Expense allocation 5,000
Adjusted 500,000 500,000

19. D E F G Total
Advances P( 4,500) P( 2,500) P( 7,000)
Loans P 5,000 P 10,000 15,000
Capitals 40,000 30,000 15,000 25,000 110,000
Total Interests P 45,000 P 40,000 P 10,500 P 22,500 P 118,000
Reduction in Interest (50,000) (30,000) (10,000) (10,000) ( 100,000)
(5:3:1:1)
CAFD P( 5,000) P 10,000 P 500 P 12,500 P 18,000*
Additional loss for possible
insolvency (3:1:1) 5,000 ( 3,000) ( 1,000) ( 1,000) -0-
Balances P 7,000 P( 500) P 11,500 P 18,000
Additional loss for possible
insolvency ( 3:1) ( 375) 500 ( 125) -0-
Balances P 6,625 P 11,375 P 18,000

20. Allocated to
Total Monte Carlo
Bonus:
30% of (P280,000 – P200,000) ..................... P 24,000 P24,000
30% of [P160,000 – (50% of P280,000)] ............... 6,000 P 6,000
Residual profit ..................................... P 30,000
Allocate 50:50 ..................................... 130,000 65,000 65,000
P160,000 P89,000 P71,000

21. Contributed Agreed


Capital Bonus Capital
Cord (3) P70,000 (6,000) 64,000
Stringer (2) 60,000 (4,000) 54,000
Twiner (new partner) 50,000 10,000 60,000*
Total P180,000 P -0- P180,000
* P180,000 x 1/3
** P10,000 x 3/5
*** P10,000 x 2/5
Agreed capital of Cord: P70,000 – P6,000 = P64,000

Page 17 of 22 0915-2303213  resacpareview@gmail.com


ADVANCED FINANCIAL ACCOUNTING & REPORTING
ReSA Batch 43 - May 2022 CPALE Batch
12 Feb 2022  6:00 PM to 9:00 PM AFAR First Pre-Board Exam
22. P45,450
A B C D=A+B
+/ (-) C
CC Revaluation Bonus AC
Allocated Allocated based
based on P/L on P/L ratio
ratio
Pine (20%) P39,000 7,200 (750)***** 45,450
Woods (60%) 90,000 21,600 (2,250)***** 109,350
Oakley (20%) – retiring partner) 51,000* 7,200 3,000**** 61,200***
Total P179,000 P36,000** P -0- P180,000
* P42,000 + P9,000
** P216,000 – 180,000
*** amount paid represents the agreed capital of the retiring partner
**** balancing figure
***** P3,000 x 2/8 and P3,000 x 6/8

Alternatively:
Amount paid P 61,200
Less: Book value of interest of Oakley (20%) * 58,200
Bonus to retiring partner P 3,000

*Total Interest of Oakley


Loans P 9,000
Capital P 42,000
Add: Share in adjustment of asset
(P216,000 – P180,000) x 20% 7,200 49,200
P58,200

Therefore, the capital balance of Pine should be:


= P39,000 + (P126,000 – P180,000) x 20% - (P3,000 x 2/8)
= P45,450

23. P145,000 = P70,000 + P50,000 + P30,000 + P25,000 – P30,000

24. P10,000* = P20,000 + P15,000 + P20,000 – P35,000 – P10,000

25. Ritz(20%) Sally(30%) Tracy (50%) Total


Total Interest 40,000 75,000 30,000 145,000
Reduction (27,000) (40,500) (67,500) (135,000)
Balance 13,000 34,500 (37,500) *10,000
Poss. Loss (2:3) (15,000) (22,500) 37,500 -0-
( 2,000) 12,000 -0- 10,000
Poss. Loss 2,000 ( 2,000) -0-
10,000 10,000

* refer to No. 24 for computation.

26. Ann (40%) Bee (40%) Kay (20%) Total


Total Interest 79,000 140,000 140,000 359,000
Share in inventory (8,000) (8,000) (4,000) (20,000)
revaluation decrease
(P60,000-P80,000)
Distribute inventory to (60,000)
Kay at fair value (60,000)
Adjusted Equity interest 71,000 132,000 76,000 279,000
Reduction (92,000) (92,000) (46,000) (230,000)
Balance ( 21,000) 40,000 90,000 *49,000
Poss. Loss (4:2) 21,000 (14,000) (7,000) -0-
0 16,000 23,000 49,000
*P198,000 – P149,000 = P49,000

Page 18 of 22 0915-2303213  resacpareview@gmail.com


ADVANCED FINANCIAL ACCOUNTING & REPORTING
ReSA Batch 43 - May 2022 CPALE Batch
12 Feb 2022  6:00 PM to 9:00 PM AFAR First Pre-Board Exam
29. Consideration Transferred:
Shares: Common stock (100,000 shares × P15) P1,500,000
Less: Fair value of net assets acquired:
Accounts receivable P 100,000
Inventory 210,000
Equipment for resale (P80,000 less 10%) 72,000
Land 200,000
Building 450,000
R&D project 90,000
Customer list (P100,000 payment
discounted 3 years at 20%) 210,650
Current liabilities ( 80,000)
Bonds payable (200,000)
Estimated liability under warranty (30,000)
Value of net identifiable assets acquired 1,022,650
Excess of total cost over FV of net assets (goodwill) P 477,350

30. Consideration Transferred:


Cash ..................................................................................... P160,000
Less: Fair value of net assets acquired:
Accounts receivable........................................................ P 200,000
Inventory ...................................................................... 270,000
Equipment .................................................................... 40,000
Brand-name copyright .................................................... 15,000
Current liabilities ............................................................ (80,000)
Mortgage payable .......................................................... (250,000)
Value of net identifiable assets acquired ................................. 195,000
Excess of total FV over cost of net assets (gain) ............................. P(35,000)
31. Unrealized Intercompany Inventory Profit/Allowance
for overvaluation of branch inventory before adjustments P10,800
Less: Allowance for overvaluation of shipments
(P28,800 - P24,000) (4,800)
Allow. for overvaluation of beginning inventory P6,000
Divided by: Mark-up on cost 20%
Merchandise inventory at cost, December 1, 2022 P30,000
Add: Allowance for overvaluation of beginning branch
Inventory 6,000
Merchandise inventory at BP, December 1, 2022 P36,000

Merch. inventory, December 1, 2022 per branch books P45,000


Less: Merchandise inventory at billed price 36,000
Merchandise inventory, December 1, 2022 -- outsiders P 9,000

32. note on the term “expected value (i.e., PV of the expected cash flow) …at the date…”,
“expected…at the” means it exists on the date of acquisition.

Goodwill 400,000
Estimated Lawsuit liability 400,000

33. note on the word “occurring subsequent”, it means it does not exist on the date of acquisition.
Loss on lawsuit 400,000
Estimated Lawsuit liability 400,000

34. Assets:
DJ: P62,400,000 + P2,100,000 + P3,000,0000 + P7,000,000 + P170,000 + P70,000
+ P50,000 + P2,384,440 – P400,000 (direct costs) – P200,000 (costs to
issue) – P5,000,000 (cash consideration) + ( unreported intangibles:
P170,000 + P70,000, + P50,000) – P290,000 cash paid for the unreported intangibles P71,574,440

Page 19 of 22 0915-2303213  resacpareview@gmail.com


ADVANCED FINANCIAL ACCOUNTING & REPORTING
ReSA Batch 43 - May 2022 CPALE Batch
12 Feb 2022  6:00 PM to 9:00 PM AFAR First Pre-Board Exam

Goodwill computation:
Consideration transferred:
Cash to former shareholders P 5,000,000
Market value of stock to former shareholders 2,000,000
Earnout (Prob. PV of Cash Contingent P4,000,000 x .25 x .69444 _____694,440
Consideration)
Consideration transferred/Total acquisition cost P 7,694,440
Less: MV
Current assets P 2,100,000
Property, plant & equipment 3,000,000
Identifiable intangibles on DJ’s books 7,000,000
Previously unreported intangibles- advanced
production technology 170,000
Noncompetition agreements 70,000
Customer contract 50,000
Current liabilities ( 1,000,000)
Long-term debt ( 5,800,000)
Previously unreported warranty contractual
Obligations ( 280,000) __5,310,000
Positive Excess: Goodwill P 2,384,440

35. Liabilities:
DJ: P6,500,000 + P30,000,000 + P1,000,000 + P5,800,000 + P280,000, contractual
Obligations + P694,440, PV of Cash Contingent Consideration P44,274,440

36. Stockholders’ Equity:


Common stock: (P200,000 + (50,000 shares x P10, par)] P 700,000
APIC (P22,000,000 + {50,000 shares x (P40 – P10]}) - P200,000, costs to issue 23,300,000
RE [P4,000,000 – P400,000, direct costs] 3,600,000
AOCI 100,000
Treasury stock ( 400,000)
P27,300,000
Therefore:
Asset (No. 36) P71,574,440
Liabilities (No. 37) P44.274,440
SHE (No. 38) 27,300,000
Total Liabilities and SHE = Total Assets P71,574,440
37. P60,000 allocation to equipment is "pushed-down" to subsidiary and increases balance from P330,000 to
P390,000. Consolidated balance is P420,000 plus P390,000.
38.
(1)
Consideration transferred .......................................................................... P60,000
Less: Strand's book value (P50,000 x 80%) .................................................. (40,000)
Fair value in excess of book value ............................................................... P20,000
Excess assigned to inventory (60%) ............................................... P12,000
Excess assigned to goodwill (40%) ................................................ P 8,000
(2)
Consideration transferred (P60,000 ÷ 80%) ................................................. P75,000
Less: Strand's book value .......................................................................... (50,000)
Fair value in excess of book value ............................................................... P25,000
Excess assigned to inventory (60%) ............................................... P15,000
Excess assigned to goodwill (40%) ................................................ P10,000

39. Park current assets ................................................................................ P 70,000


Strand current assets ............................................................................. 20,000
Excess inventory fair value ..................................................................................... 15,000
Consolidated current assets.................................................................................... P105,000

Page 20 of 22 0915-2303213  resacpareview@gmail.com


ADVANCED FINANCIAL ACCOUNTING & REPORTING
ReSA Batch 43 - May 2022 CPALE Batch
12 Feb 2022  6:00 PM to 9:00 PM AFAR First Pre-Board Exam
40.
(1)
Park noncurrent assets ............................................................................. P 90,000
Strand noncurrent assets .......................................................................... 40,000
Excess fair value to goodwill (partial) ......................................................... ___8,000
Consolidated noncurrent assets ................................................................. P138,000
(2)
Park noncurrent assets .............................................................................. P 90,000
Strand noncurrent assets ........................................................................... 40,000
Excess fair value to goodwill (full) ............................................................... __10,000
Consolidated noncurrent assets .................................................................. P140,000
41.
(1) Add the two book values and include 10% (the P6,000 current portion) of the loan taken out by Park
to acquire Strand.
(2) Add the two book values and include 90% (the P54,000 noncurrent portion) of the loan taken out by
Polk to acquire Strand.
42.
(1)
Park stockholders' equity .......................................................................... P80,000
NCI (partial):
BV of SHE – S ……………………………………………………………..P50,000
Adjustments to reflect fair value (inventory)……………….15,000
FV of SHE – S……………………………………………………………… P65,000
x: Multiplied by: NCI%................................................ 20% 13,000
Total stockholders' equity ......................................................................... P93,000

(2)
Park stockholders' equity ............................................................... …………. P80,000
NCI (full):
BV of SHE – S ……………………………………………………………..P50,000
Adjustments to reflect fair value (inventory)……………. 15,000
FV of SHE – S……………………………………………………………………….P65,000
x: Multiplied by: NCI%............................................... 20%
NCI (partial)………………………………………………………………...P13,000
Add: NCI on full-goodwill (P10,,000 – P8,000)…………. 2,000
Non-controlling interest at fair value (20% × P75,000)………… 15,000
Total stockholders' equity P95,000
43. Combined Cost of Goods Sold:
Merchandise Inventory, 1/1/20X5:
Home Office, cost……………………………………………… P 3,500
Branch: Outsiders, ……………………………...........................P 300
From Home Office (P2,500 – P300)/110%.............. 2,000 2,300 P 5,800
Add Purchases (P240,000 + P11,000)…………………………….. 251,000
COGAS………………………………………………………………… P 256,800
Less: Merchandise Inventory, 12/31/20X5
Home Office, cost………………………………………………. P 3,000
Branch: Outsiders…………………………………………………………...P 150
From Home Office (P1,800 – P150)/110%............ 1,500 1,650 4,650
Cost of Goods Sold………………………………………………… P252,150
44. refer to AFAR-04 on discussion of Contract Liability
January 1, 20x5:
Cash……………………………………………………………………………………………………………………….10,000
Notes Receivable……………………………………………………………………………………………………40,000
Unearned Interest Income/Discount on Notes Receivable………………… 10,433
Contract Liability (P10,000 + P29,567)…………………………………………… 39,567*
*Down payment made on 4/1/x5…………………………………………..P10,000.00
Present value of an ordinary annuity (P8,000 x 3.69590).. 29,567.20
Total revenue recorded by Campbell and total
acquisition cost recorded by Lesley Benjamin……………..P39,567.20
45. December 31, 20x5: (P39,567 ÷ 5) = P7,913
46. No entry is required on March 1, 2022, because neither party has performed on the contract. That is, neither
party has an unconditional right as of March 1, 2022. On July 31, 2022, Giordano delivers the product and
therefore should recognize revenue as it received an unconditional right to consideration on that date. In
addition, Giordano satisfies its performance obligation by delivering the product to Hotter.

No entry – neither party has performed on March 1, 2022.

Page 21 of 22 0915-2303213  resacpareview@gmail.com


ADVANCED FINANCIAL ACCOUNTING & REPORTING
ReSA Batch 43 - May 2022 CPALE Batch
12 Feb 2022  6:00 PM to 9:00 PM AFAR First Pre-Board Exam
The entry on July 31, 2022, to record the sale and related cost of goods sold is as follows:
Accounts receivable. . . . . . . . . . . . . . . . . . . . . . . . . . . . . . . . . . .57,000.
Sales. . . . . . . . . . . . . . . . . . . . . . . . . . . . . . . . . . . . . . . . . . . 57,000

Cost of sales. . . . . . . . . . . . . . . . . . . . . . . . . . . . . . . . . . . . . . . .34,200


.
Inventory . . . . . . . . . . . . . . . . . . . . . . . . . . . . . . . . . . . . . . . . 34,200

The entry to record the receipt of cash on August 31, 2022 is a follows:
Cash. . . . . . . . . . . . . . . . . . . . . . . . . . . . . . . . . . . . . . . . . . . . .57,000
.
Accounts receivable . . . . . . . . . . . . . . . . . . . . . . . . . . . . . . . . . 57,000
A key attribute of the revenue arrangement is that the signing of the contract by the two parties is not
recorded until one or both of the parties perform under the contract. Until performance occurs, no net
asset or net liability occurs.
47. P75,000 + P50,000 + P25,000 = P150,000
P75,000/ P150,000  P120,000 = P60,000
P50,000/ P150,000  P120,000 = P40,000
P25,000/ P150,000  P120,000 = P20,000.
49. Joey is acting as principal in the contract based on the following indicators:
• Joey is responsible for fulfilling the contract because it is responsible for ensuring that the excavator
meets specifications
• Joey has inventory risk because it is responsible for correcting error in specifications, even though the
manufacturer has inventory risk during production
• Joey has discretion in establishing the selling pric
• Joey’s consideration is in the form of profit, not commission
• Joey has credit risk for the P46,200,000 receivable from Tanner
50. In this case, two performance obligations exist:
1. one related to the sale of the computer and the assurance warranty (it should be noted that quality-
assurance warranty is part of performance obligation), and

2. the other to the extended warranty (service warranty).

The sale of the computer and related assurance warranty (quality-assurance) are one performance
obligation as they are interdependent and interrelated with each other.

However, the extended warranty is separately sold and is not interdependent (or not connected).
51. Because the arrangement only has two possible outcomes (regulatory approval is achieved or not), Bai
determines the transaction price based on the most likely approach. Thus, the best measure for the
transaction price is P20,000,000.

52. December 20, 2022


No entry-neither party has performed.
January 15, 2023
Cash .................................................................10,000,000
License Revenue ......................................................... 10,000,000

**Don’t think that there’s so much darkness, that it’s no use to have a small light, because even one
candle can be seen a mile away when it’s dark.**
**When all else is lost, the future still remains.**
**The greatest mistake you can make is to continually fear making mistakes.**
We are never given guarantees in life. We are only given the opportunities and it is up to us to make the
BEST out of it.
The most difficult secret of a man to keep is the opinion he has of himself.
Nothing great was ever achieved without determination.
Don’t be discouraged; everyone who got where he is, started where he was.
Impossibilities vanish when a man and his GOD confront a mountain.

Page 22 of 22 0915-2303213  resacpareview@gmail.com

You might also like